test 4 cox, Final

Ace your homework & exams now with Quizwiz!

What is creatinism?

Baby is missing palmar grasp.

Fibrocystic breast condition

Breast pain and tender lumps; the lumps are rubbery, ill defined, and commonly found in the upper outer quadrant of the breast caused by an imbalance in the normal estrogen-to-progesterone ratio. Onset late teens and 20s; usually subsides after menopause

When you are mixing regular and NPH insulin what gets drawn up first?

Clear before cloudy (regular before NPH).

The client is admitted to the hospital with a suspected diagnosis of Hodgkin's disease. Which assessment finding would the nurse expect to note specifically in the client? "a) fatigue b) weakness c) weight gain d) enlarged lymph nodes"

D - Hodgkin's disease is a chronic progressive neoplastic disorder of lymphoid tissue characterized by the painless enlargement of lymph nodes with progression to extralymphatic sites, such as the spleen and liver. Weight loss is most likely to be noted. Fatigue and weakness may occur but are not related significantly to the disease."

In the client with a subdural hematoma, which of the following if assessed would indicate Increased ICP? A. Tachycardia B. Constricted Pupils C. Elevated diastolic pressure D. Decreased level of consciousness

D. Decreased level of consciousness

Which of the following is not a general side effect of chemotherapy treatment? A. Alopecia B. Bone marrow suprress C. Nausea and vomiting D. Urinary Retention

D. Urinary Retention

What electrolytes/ values is Addison low in?

Everything except K (Na, H20, BP, and Blood Sugar)

Adrenal cortex

Glucocorticoids (cortisol) Mineralocorticoids (aldosterone)

Nursing considerations for hormone diagnostic tests - Growth hormone (GH) lab study - Growth hormone (GH) stimulation (insulin tolerance test) - ADH stimulation (water deprivation test)

Growth hormone lab study - Note fasting status and recent activity level Growth hormone stimulation (insulin tolerance test) - NPO after midnight, water is permitted - Keep 50% dextrose and 5% dextrose IV at bedside - Monitor hypoglycemia and hypotension ADH stimulation (water deprivation test) - NPO for test - Serum Na is normal and urine osmolality < 300 - Hourly urine specific gravity check - Q2h Serum Na and osmolality check - Discontinue test and rehydrate if weight drops more than 2kg at anytime

Ductal ectasia

Hard, irregular mass or masses with nipple discharge, enlarged axillary nodes, redness, and edema; difficult to distinguish from cancer Women approaching menopause

What can cause diabetes insipidus?

Head trauma, brain tumor, punched in nose, or infection of the brain.

What are the S&S of thyroid storm?

Heart palpitations, anxiety, dyspnea, diaphoretic, tremors, bulging eyes, high fever (104*).

Diverticulosis

Herniation of intestinal mucosa

What are complications with hyperpituitary-acromegaly?

Increased ICP, DM, and meningitis

What does a cushings syndrome diet look like?

Increased protein & K but decreased calories, carbs, Na.

Weight loss with increased appetite may indicate which gland dysfunction?

Hyperthyroidism

major endocrine glands

Hypothalamus (a neuroendocrine gland) • Pituitary gland • Adrenal glands • Thyroid gland • Islet cells of the pancreas • Parathyroid glands • Gonads

What should you have at the bedside in case adrenal insuffiency arises?

Inderal (lowers BP/stress).

What is the treatment for hyperthyroidism (graves disease)?

Diet: 6-8000 cal/day, propylthiaracil (blocks thyroid hormone), if propythiaracil doesn't work take i131 test (destroys gland so become myxedema pt on synthroid).

What should you remember when testing someone's blood glucose at the bedside?

Don't squeeze puncture site (dilutes w tissue fluid), choose outer edge of fingertip or earlobe for adult (heel for infant), hold pts finger in a dependent position and gently massage finger twd puncture site, clean with alcohol swab.

What is pheochromocytoma? S&S? check all that apply

It is a tumor on the adrenal gland. S&S= HTN, tachycardia, palpitations, sweating, fever, and headaches.

What are the S&S of hyperglycemia? check all that apply

Like being drunk! Extreme thirst, FREQUENT URINATION, dry skin, HUNGER, BLURRED VISION, drowsiness, and nausea.

What are the S&S of hypoglycemia? check all that apply

Like hangover. SHAKING, SWEATING, HUNGER, WEAKNESS/FATIGUE, HA, IRRITABLE, anxious, dizziness, fast heartbeat, and impaired vision.

Diverticulitis has pain where

LlQ abdominal pain Blood in stools

Nephropathy

Microvascular complication associated with damage to small vessels of the kidney - Prevention with HTN and glucose control

Fibroadenoma of breast

Most common benign lesion; solid mass of connective tissue that is unattached to the surrounding tissue

Can you mix lantus?

NO!!!!

hormone secreting portion of pancreas

Islets of Langerhans

Diabetic retinopathy treatment

Prevent hemorrhaging and resulting scarring (laser photo coagulation) and/or removal of cloudy vitreous humor due to hemorrhaging (vitrectomy)

Insulin

Produced by Pancreases (Islet) Decrease blood glucose - Facilitates glucose transport into cells

Oxytocin

Production stimulated by touch receptor in nipples - Stimulates milk secretion - Uterine contractility

What is elevated with hyperpituitary/acromegaly?

Serum somatotrophin

Melanocyte-stimulating hormone (MSH)

Target Pineal gland - Increase melanin production - melanocytes to make skin darker - Pineal gland also help regulate circadian rhythms and reproductive system at onset of puberty

A 26-year-old patient with diabetes rides a bicycle to and from work every day. Which site should the nurse teach the patient to administer the morning insulin? a. thigh. b. buttock. c. abdomen. d. upper arm.

ANS: C Patients should be taught not to administer insulin into a site that will be exercised because exercise will increase the rate of absorption. The thigh, buttock, and arm are all exercised by riding a bicycle

factors in development of type 2 diabetes

-insulin resistance -decrease ability of pancreas to produce insulin -inappropriate glucose production -altered production of hormones and adiopokines -metabolic syndrome

The client is diagnosed as having a bowel tumor and several diagnostic tests are prescribed. The nurse understands that which test will confirm the diagnosis of malignancy? 1) Biopsy of the tumor. 2) Abdominal ultrasound. 3) Magnetic resonance imaging. 4) Computed tomography scan

1) Biopsy of the tumor.

5 components of Metabolic syndrome (risk factor for diabetes II)

1) Elevated glucose levels 2) Abdominal obesity 3) Elevated blood pressure 4) High triglycerides 5) Decreased high-density lipoproteins (HDLs)

Infection in diabetes

- Diabetics are more susceptible to infections because of a defect in mobilization of WBCs and impaired phagocytosis - Neuropathy may delay the detection of infection - Persistent glycosuria may predispose patient to bladder infection

Drug alert Metformin

- Do not use with kidney, liver, or heart disease - IV contrast media can exacerbate metformin-induced lactic acidosis - Patients undergoing surgery or radiologic procedure that involves contrast medium are instructed to temporarily d/c metformin before surgery and not resume until 48 hours after.

Diabetes exchange lists

- the individual is given a meal plan with specific numbers of helpings from a list of exchanges for each meal and snack. - The exchanges are starches, fruits, milk, meat, vegetables, fats, and free foods.

A client is scheduled for an outpatient electroencephalogram (EEG). A nurse instructs the client to prepare for the test by: 1. removing all hair pins. 2. avoiding eating or drinking 6 hours prior to the test 3. Being prepared to have some of the scalp shaved 4. Having blood drawn for a glucose level 2H before the test

1. removing all hair pins.

Impaired fasting glucose (IFG)

100-125 mg/dL

Target BP for diabetic:

130/80

insulin per carb

1U/15g 2U/25g

Latent autoimmune diabetes in adults (LADA)

a condition in which type 1 diabetes develops in adults

Roux-en-Y gastric bypass (bypass)

The patient's stomach, duodenum, and part of the jejunum are bypassed so fewer calories can be absorbed

Thyroid

Triiodothyronine (T3 ) Thyroxine (T4 ) Calcitonin

Can Addison's pts have steroids?

Yes because they increase Blood Sugar

Which signs/symptoms should make the nurse suspect the client is experiencing a thyroid storm? 2. Hyperpyrexia and extreme tachycardia.

2. Hyperpyrexia (high fever) and heart rate above 130 beats per minute are signs of thyroid storm,

The client is admitted to rule out Cushing's syndrome. Which laboratory tests should the nurse anticipate being ordered? 2. Plasma levels of ACTH and cortisol.

2. The adrenal gland secretes cortisol and the pituitary gland secretes adrenocorticotropic hormone (ACTH), a hormone used by the body to stimulate the production of cortisol.

nutrition breakdown

20-35 calories/kg of body weight per day (20-25 cal/kg to lose weight; 25-30 cal/kg to maintain; 30-35 cal/kg to gain weight) Carbs 45-65% total calories; Fats 20-35% total calories; Proteins 10-35% calories per day. Recommended daily intake of PROTEIN 0.8-1g/kg of body weight

how many grams of carbonhydrates are recommanded

25 to 30g/day

When teaching a patient with chronic SIADH about long-term management of the disorder, the nurse determines that additional instruction is needed when the patient says, a. "I need to shop for foods that are low in sodium and avoid adding salt to foods."

a. "I need to shop for foods that are low in sodium and avoid adding salt to foods." Rationale: Pts with SIADH are at risk for hyponatremia, and a sodium supplement may be prescribed.

peritonitis

Abdominal pain. Rebound tenderness. Shallow breaths, fever, distention, vomiting depending on severity Diagnostics: CBC, peritoneal aspiration, xray of abdomen may show paralytic ileus (dilated loops of bowel), free air or fluid. US and CT for ascites and abscesses. Medical management: abx, NG suction, pain control, IVF. Surgery to drain purulent fluid and repair damage, investigate cause. Drains. Nursing care: IV access, pain control (positioned knees flexed), I+O and electrolyte monitoring for replacement therapy. NPO and NGT to suction (decrease gastric distension and further leakage of bowel contents into peritoneum. may need low flow O2 prn.

Acanthosis nigrians

Acanthosis nigrians - Velvety light brown to black skin thickening, predominantly seen on flexures, axillae, and the neck Necrobiosis lipoidica diabeticorum - Red yellow lesions, with atrophic skin that becomes shiny and transparent revealing tiny blood vessels under the surface.

treatment for dawn phenomenon

Add or increase the at-bedtime dose of insulin. "Dawn is rising" or adjust dose

Hyperpigmentation, or "bronzing" of the skin (particularly on knuckles, elbows, knees, genitalia, and palmar creases) is a classic finding in what disease and what hormone deficiency?

Addison disease - Deficient cortisol and steroid hormones

What angle should be used to give insulin?

45* SQ

While hospitalized for a fractured femur, a 68-year-old pt is diagnosed with hypothyroidism. Which of these medications on the original admission orders will the nurse need to consult with the HCP about before it is administered? select all a. Docusate (Colace) b. Percocet (oxycodone/acetamiophen) c. Diazepam (Valium) d. Ibuprofen (Motrin)

B and C R: Worsening of mental status and myxedema coma can be precipitated by the use of sedatives, especially in older adults. The nurse should discuss the diazepam with the health care provider before administration. The other medications may safely be given to the pt.

The neurologic assessment of a client who had a brain abscess and is post craniotomy includes the Glasgow Coma scale What does the nurse evaluates to assess the clients score on the Glascow Coma scale 1. Ability of the clients pupils to react to light 2. Degree of purposeful movements by the client 3. Appropriateness of the clients verbal responses 4. Stimulus necessary to cause the clients eyes to open 5. Symmetry of muscle strength of the clients extremities A. 1, 2,4 B. 2, 3, 4 c. 2, 4, 5 D. 1, 3, 5

B. 2, 3, 4

What should you suspect with a pregnant woman who is craving ice cubes?

Blood sugar imbalance (thirsty). Possibly gestational diabetes.

What organs are affected by metabolic syndrome?

Brain, hear, and pancreas.

A client receiving external radiation to the left thorax to treat lung cancer has a nursing diagnosis of Risk for impaired skin integrity. Which intervention should be part of this client's plan of care? A. Avoiding using soap on the irradiated areas B. Applying talcum powder to the irritated areas daily after bathing C. Wearing a lead apron during direct contact with the client D. Removing thoracic sin markings after each radiation treatment

A. Avoiding using soap on the irradiated areas

Of the following which are appropriate in caring for stomatis? A. Avoid spicy or acidic food because they will irritate the condition B. Use soft bristle toothbrush C. Local anesthetics D. Rinse frequently with commercial mouth washes

A. Avoid spicy or acidic food because they will irritate the condition B. Use soft bristle toothbrush C. Local anesthetics

What does the pituitary gland secrete?

ADH, Thyroid stimulating hormone, sex hormones, growth hormone, and ACTH.

The nurse is caring for a patient with increased intracranial pressure (ICP). Which procedure is contraindicated in this case? A. EEG B. Skull X-rays C. Lumbar Puncture D. CT Scan

C. Lumbar Puncture

hypopituritarism

affects growth hormone thyroid stimulating hormone, andtidiuretic hormone

for people with no risk factors for diabetes screening should begin at

age 45

A client phones a nurse after having three basal cell carcinoma lesions excised the day before and is concerned that the wounds are draining a small amount of serosanguanous fluid and that the small dressing is leaking. Which action should the nurse recommend? A. Apply ice to the dressing B. Contact the physician C. Change the dressing D. Take medication for pain

C. Change the dressing

If an unconscious client demonstrates decorticate posturing the nurse will observe A. Flexion of both upper and lower extremities B. Extension of elbows and knees, plantar flexion of feet and flexion of wrists C. Flexion of elbows, extension of knees, and plantar flexion D. Extension of upper extremities, flexion of lower extremities

C. Flexion of elbows, extension of knees, and plantar flexion

what to do for hypoglycemia complications

Check glucose if possible. If < 70 mg/dL, give glucose immediately (15 g simple carbohydrate if conscious). Recheck in 15 min. Repeat. Complex carbohydrate after recovery. ( 15 g every 15 min until improvement is seen)

Which finding should a nurse identify as requiring further investigation? a)White blood cell (WBC) count of 7,000/?l b)Platelet count of 115,000/ c)Red blood cell (RBC) count of 4.9 million/?l d)Hematocrit of 45%

b)Platelet count of 115,000/

A patient receives aspart (NovoLog) insulin at 8:00 AM. Which time will it be most important for the nurse to monitor for symptoms of hypoglycemia? a. 9:00 AM b. 11:30 AM c. 4:00 PM d. 8:00 PM

ANS: A 9:00AM The rapid-acting insulins peak in 30min to 3 hours. The patient is not at a high risk for hypoglycemia at the other listed times, although hypoglycemia may occur.

The nurse is assessing a client diagnosed with acute myeloid leukemia. Which assessment data support this diagnosis? "1.) Fever and infections. 2.) Nausea and vomiting. 3.) Excessive energy and high platelet counts. 4.) Cervical lymph node enlargement and positive acid-fast bacillus."

Correct: 1. 1. Fever and infection are hallmark symptoms of leukemia. They occur because the bone marrow is unable to produce WBCs of the number and maturity needed to fight infection (CORRECT). 2. Nausea and vomiting are symptoms related to the treatment of cancer but not to the diagnosis of leukemia (omit #2). 3. The clients are frequently fatigued and have low platelet counts. The platelet count is low as a result of the inability of the bone marrow to produce the needed cells (omit #3). 4. Cervical lymph node enlargement is associated with Hodgkin's lymphoma, and positive acid-fast bacillus is diagnostic for tuberculosis (omit #4)."

Which of follwoing diagnostic tests should be preformed annually after age 50 to screen for colon cancer? A. Abdominal CT Scan B. Abdomnal X-ray C. Colonoscopy D. Fecal occult blood test

D. Fecal occult blood test

the colony stimulating agent oprelvekin (Neumega) is used A. To stimulate RBC production B. To stimulate WBC production C. To inhibit WBC production D. To stimulate platelet production

D. To stimulate platelet production

Complications of feet and lower extremities

Diabetics have high risk for foot ulcerations and lower extremities amputations - Sensory and autonomic neuropathy decreases protective sensation and increase risk of injury and infection - PAD reduces blood flow and nutrition to LE, delayed wound healing, causes ischemic digits

effect of iodine on thyroid

Dietary intake of protein and iodine is needed to produce thyroid hormones. Iodine is absorbed from the intestinal tract as iodide. The thyroid gland draws iodide from the blood and concentrates it. After iodide is in the thyroid, it combines with the amino acid tyrosine to form T4 and T3 . These hormones bind to thyroglobulin and are stored in thyroid follicular cells. When stimulated, T4 and T3 are released into the blood. They

Drug therapy for hyperthyroidism

Drug Therapy - Also used to prepare patient for thyroidectomy - Antithyroid drugs (PTU and Tapazole) + PTU needs TID, Tapazole needs QD doses + Improvement in 1-2 wks, good result in 4-8 wks + Therapy for 6-15 months to allow for remission - Iodine + Large rapid doses inhibit T3/T4 + Maximal effect seen in 1-2 weeks - B-adrenergic blocks used for symptom relief

Osmotic Diuresis in DKA

Elevated Serum BG raises serum osmolality --> H2O shifts from cells to ECF --> Osmotic diuresis occurs with significant loss of H2O, Na, and K Hyperglycemia is exacerbated by the osmotic diuresis induced pre-renal uremia --> impairs urinary glucose excretion.

Diabetic Ketoacidosis Collaborative care

Emergency - Maintain patent airway - Establish IV access - Begin fluid and electrolyte replacement - IV administration of insulin (AFTER K+ levels checked and replaced) - B.G. reduction of 36-54 mg/dL/hr is appropriate

Exercise and Diabetes

Exercise lowers the blood glucose level, encourages weight loss, reduces cardiovascular risks, improves circulation and muscle tone, decreases total cholesterol and triglyceride levels, and decreases insulin resistance and glucose intolerance.

What are the S&S of hyperthyroidism (Graves disease)?

FAST AND HIGH, intolerance to heat, fine straight hair, facial flushing, tachycardia, weight loss, diarrhea, finger clubbing, amenorrhea, BULGING EYES, localized edema.

How is diabetes diagnosed?

Fasting BS, 5 hour glucose tolerance test (fast 8 hrs drink sugar, check urine for sugar and BS hourly- It normally elevates @ 2 hrs then starts to decrease but diabetics keeps going up. Never Give This Test To Kids), sugar in urine, and glycosylated hemoglobin.

Prolactin

Female: Stimulate milk production in lactating women. Increases response of follicles to LH and FSH. Male: Stimulates testicular function in men

A client is being discharged after undergoing a thyroidectomy which of the following discharge instructions would be appropriate for this client. a. Report signs and systems of hypoglycemia b. Take thyroid replacement medication as ordered c. Watch for changes in body functioning such as lethargy restlessness d. Avoid all over the counter medications e. Carry injectable dexamethasone at all times

b. Take thyroid replacement medication as ordered c. Watch for changes in body functioning such as lethargy restlessness

A pt is suspected of having a pituitary tumor causing panhypopituitarism. During assessment of the pt, the nurse would expect to find b. changes in secondary sex characteristics.

b. changes in secondary sex characteristics. Rationale: Changes in secondary sex characteristics are associated with decreases in FSH and LH.

What test helps evaluate your average blood sugar over a 3 month period?

Hemoglobin A1C (blood sugar attached to red blood cells-live for about 3 months)

How to prevent somogyi effect

bed time snack reduction of insulin dose or both

complications: hypoglycemia

blood glucose level falls below 60 mg/dl caused by too much insulin or oral hypoglycemic agents client should carry some forme of fasht acting simple carbohydrate with them ( high fat foods not recommended )

Glycogenolysis

breakdown of glycogen to glucose

What are the S&S of SIADH?

Increased ADH (wet as a watermelon), too much water reabsorbed from kidneys, dec output, inc SG>1.030, increased urine osmolality, fluid retention =decreased serum Na=decreased serum osmolality<280 mOs/kg.

What are the S&S of metabolic syndrome?

HTN, Obesity (waist greater than 40" in men 35" women), sedentary lifestyle, poor diet, family history, Blood pressure 130/85 or higher, glucose 100 or higher, tiredness, and difficulty losing weight.

What are clinical manifestations of hyperpituitary/acromegaly?

HYPERTROPHY OF SOFT TISSUE, PROTRUDING JAW, ENLARGEMENT OF SMALL BONE IN HAND AND FEET, Goiter, visual changes, coarse facial features, cardiomyopathy.

What values are Cushing's pts low in?

K only everything else is high (Na, H20, BP, BS)

Chronic complications of diabetes mellitus

Macrovascular complications (earlier onset) - Cerebrovascular, cardiovascular and peripheral vascular diseases. Microvascular complications (specific to diabetes) - Thickening of vessel membranes in capillaries and arterioles in response to conditions of hyperglycemia - Common: retinopathy, nephropathy, dermopathy

What should you do immediately for someone experiencing thyroid storm?

O2, IV placement for fluids, propythiouracil (dec thyroid), lower body temp wtemp blanket and acetaminophen.

Who gets type I diabetes? What is characteristic of type I?

ONLY Kids, was called juvenile diabetes, it is an autoimmune disorder where no insulin is produced (insulin dependent). It occurs most often before age 15 and can be familial. It is a lifelong disease.

aspiration pneumonia

Observe for increasing temperature and pulse and for other signs of dehydration such as dry mucous membranes and decreased urinary output. Auscultate lungs every 4 to 8 hours to check for diminishing breath sounds, especially in lower lobes. Patients may become short of breath and report chest discomfort. A chest x-ray confirms this diagnosis, and treatment with antibiotics is started.

What is important about an insulin pump?

Only regular insulin used, administer every time eat, and change the needle every 3 days.

Major reason why patient with pituitary problem seek treatment

Pituitary gland compressing optic nerve

A female client is undergoing tests for multiple myeloma. Diagnostic study findings in multiple myeloma include: A a low serum protein level. B hypocalcemia C Bence Jones protein in the urine. D a decreased serum creatinine leve

Presence of Bence Jones protein in the urine almost always confirms the disease, but absence doesn't rule it out. Serum calcium levels are elevated because calcium is lost from the bone and reabsorbed in the serum. Serum protein electrophoresis shows elevated globulin spike. The serum creatinine level may also be increased.

Glucagon

Produced by Pancreas Increase blood glucose - Stimulate glycogenolysis, gluconeogenesis, and ketogenesis

Parathyroid hormone (PTH)

Produced by Parathyroid gland (stimulated by low calcium) Helps calcium go bone to blood - Increases calcium blood levels Decrease renal excretion of Ca - Increase calcium blood levels Increases renal excretion of phosphorus

Thyroxine (T4) and T3

Produced by Thyroid gland (stimulated by TSH) - Increases metabolic rate of all cells and processes of cell growth and tissue differentiation

Diabetic Ketoacidosis pathophysiology clinical manifestations

Profound deficiency in insulin and id characterized by hyperglycemia, ketosis, acidosis, and dehydration 1) When theres not enough insulin, glucose can't be used for energy 2) Body compensates by using fat for energy 3) Ketones are acidic by-product of fat metabolism Clinical manifestations - Dehydration s/s - Abdominal pain, nausea and vomiting - Kussmal respirations - Acetone in breath (fruity odor)

Growth hormone (GH), or somatotropin

Promotes protein anabolism - Promote growth and tissue repair Lipid mobilization and catabolisms

Insulin types onset, peak, duration Rapid acting - lispro (Humalog) - aspart (NovaLog) Short acting - Regular (Humulin R, Novolin R) Intermediate acting - NPH (Humulin N, Novolin N) Long acting - glargine (Lantus) - detemir (Levemir)

Rapid acting - Onset: 15 minute - Peak: 1-2 hours - Duration: 3-5 hour Short acting - Onset: 30-60 minutes - Peak: 2-4 hour - Duration: 6-10 hour Intermediate acting - Onset: 1-2 hour - Peak: 4-12 hours - Duration: 12-18 hours Long acting - Onset: 1-2 hours - Peak: No peak - Duration: 24 hours

Adrenocortical Insufficiency (Addison's disease)

Reduced corticosteroids Etiology - Most common is autoimmune disease - Autosomal recessive trait - Primary: All corticosteroids are reduced - Secondary: pituitary problem Clinical manifestations - Not evident until 90% adrenal cortex destroyed - Slow progressive weakness, fatigue, weight loss, anorexia - Bronze-colored skin hyperpigmentation to sun-exposed areas of the body, pressure points, joints Complications - adrenal insufficiency (addisonian crisis) Diagnostic Studies - Depressed serum and urinary cortisol levels - Increased ACTH levels in Primary - Decreased ACTH levels in Secondary

What are the three types of rapid acting insulins?

Regular, Novolin R, and Humulin (onset 15-30 mins, peak 2-5 hrs, duration 2-8 hrs, adverse reaction occurs midmorning-trembling/weakness).

What is a transphenoidal hypophysectomy?

Removal of pituitary gland by approach through the nose.

What if the medical tx for cushings is unsuccessful?

Remove adrenal glands and turn into addisons patient (can control addisons with steroids)

what should be reviewed to determine nutritional status

Review of the nutrition history • Food and fluid intake record • Laboratory data • Food-drug interactions • Health history and physical assessment • Anthropometric measurements • Psychosocial assessment

Neuropathy Care

Medication - capsaicin, TCAs, SSRI, antiseizure,

Commonly used medications that can induce diabetes in some people Conditions that can cause diabetes

Medications - Corticosteroids (prednisone) - Thiazides - Phenytoin (Dilantin) - Atypical antipsychotics (e.g. clozapine [Clozaril]) Conditions that can cause diabetes - Cushing syndrome - Hyperthyroidism - Recurrent pancreatitis - Cystic fibrosis - Hemochromatosis - Parenteral nutrition

What are two complication of transphenoidal hypophysectomy?

Meningitis (check for clear fluid from nose, decrease stimulation, seizure precautions) and DIABETES INSIPIDUS (decrease ADH=decreased concentration=decreas SG<1.010).

Adrenocorticotropic hormone (ACTH)

Stimulate adrenal cortex - Stimulates secretion of corticosteroids

Thyroid-stimulating hormone (TSH) or thyrotropin

Stimulates thyroid gland - Stimulate thyroid hormone release

insulin can be given

Sub Q in -abdomen(best site) ----arm, thigh, butt avoid areas to be exercised (increased circulation= increased and rapid absorption= hypoglycemia)

diabetes put individual at higher risk of

cardiovasvular disease stroke reduced immunity eye and vision complications neuropathy

too much ADH: SIADH

causes water intoxication and hyponatremia

how to determine if somogyi effect?

check BG @ 2 to 4am for hypoglycemia

polyphagia

consequence of cellular malnourishment when insulin deficiency prevents utilization of glucose for energy body turns to other sources(protein and fat) for energy (weigth loss)

insulin pump

continuous subcutaneous insulin infusion administered by external work device. continuous basal rate ofinsulin is infused in additonto bolus of insulin befor emeals

hypothalamus

controls body temp, sleep , and appetite

thyroid gland

controls the rate of body metabolism produces thyroxine (t4) triiodothyronine (T3 ).

Disorders of posterior pituitary gland SIADH

Syndrome of inappropriate antidiuretic hormone (SIADH) - too much ADH - Fluid retention - Low urine output - Dilutional Hyponatremia (vomiting, cramps, muscle twitching, seizures, headaches, lethargy) Collaborative Care - Fluid restriction to 1000 if mild, 500 if severe - HOB flat or <10 degree to enhance venous return to the heart (increase BP) = reduce release of ADH - Seizure precautions - If severe: IV hypertonic saline solution, Lasix - demeclocycline (Declomycin) - block ADH effect - Ice chips/sugarless gum to relief thirst

What is the treatment for hypothyroidism?

Synthroid, avoid stimulants (coffee, tea), need to know if med is working.

causes of diabetes mellitus

genetic autoimmune environmental(virus, obesity)

adrenal glands

regulates sodium and electrolyte balance (aldosterone) fat, protein, and carb metabolism

glucagon

released by pancreatic alpha cells in response to low BG glucagon increases BG

repeat test for

to confirm repeat A1C, FPG, OGTT

rifaximin

used for travelers constipation

metabolic syndrome diagnosed if 3 or more conditions

waist circumference triglycerides HDL BP fasting blood glucose

insulin increases

when food is ingested

A patient with Grave's asks the nurse what caused the disorder. The best response by the nurse is d. "In genetically susceptible persons antibodies form that attack thyroid tissue and stimulate overproduction of thyroid hormones."

d. "In genetically susceptible persons antibodies form that attack thyroid tissue and stimulate overproduction of thyroid hormones."

signs of peritonitis

guarding of abdomen, pallor, progressive abdominal pain and distention, restlessness, tachycardia and tachypnea, fever and chills

symptoms of addison crisis

headache severe abdominal, leg, and lower back pain generalized weakness irritbility and confusion severe hypotension hyponatremia, hyperkalemia, hypoglycemia,

fat

healthy fats: olives, nuts, avocados limit trans fat

CRP (C-reactive protein)

high levels seen in diabetic pts

interventions of hypopituitary

hormone replacement family/patient support

activation of insulin from

increased blood glucose

Where is excess glucose stored?

liver and muscles

alpha cells of pancreas

make and secrete glucagon

assessment of hypopituitarism

mild to moderate obesity reduced cardiac output sexual dysfunciton fatigue, low bp

High risk for gestational diabetes

obese older family hx of diabetes

type 2 diabetes has to do with

pancreas liver adipose tissue muscle

early stages of insulin resistance

pancreas responds to high BG by producing great amounts of insulin temporary state of hyperinsulinemia that co-exists with hyperglycemia

glitazones are contraindicated in

patients symptomatic for heart failure or cardiovascular disease

if BG is over 250 in person while exercise with type 1 diabetes and ketones are present then

delay activity drink fluids exercise can worsen hyperglycemia

if BG is less than 100 while exercise then

eat 15g after 15 to 30mins recheck if BG is under 100 delay exercise

avoiding hypoglycemia when exercising

exercise 1 hr after meal or take 15g carb snack can eat carbs q 30min during exercise

glucogeneogenesis

formation of glucose

insulin is not taken orally due to

gastric acids inactivate

Treating hypoglycemia Treating hyperglycemia

Treating hypoglycemia: Rule of 15 1) First check blood glucose 2) If under 70, give 15g of simple carbohydrates - (4-6oz of juice or soft drink) 3) Check glucose 15 minutes later; repeat 2-3 times - *In acute setting, it can be treated with 20-50mL of 50% dextrose IV push* - Can also give 1 mg of glucagon by IM or subcut.

What should you suspect if a 13 year old is wetting bed, eating alot, and not gaining weight?

Type 1 diabetes.

What type of DM get Diabetic Ketoacidosis?

Type I

What type of DM is ketosis prone?

Type I

What type of DM get Hyperglycemic Hyperosmolar Nonketotic syndrome (HHNK)?

Type II

Clinical manifestations of hypoglycemia

*Cold and clammy give some candy* Release of epinephrine - shakiness, tremors, palpitations - nervousness, diaphoresis, anxiety, pallor Insufficient glucose to the brain - Faintness, dizziness, headache - Unsteady gait, slurred speech, difficulty speaking - *stupor, confusion, seizures, coma* Other clinical manifestations - *Numbness of fingers, toes, and mouth*

Clinical manifestations of hyperglycemia

*Hot and dry, surgar high* - *Polyuria, Polyphagia* - Weakness, fatigue - Blurred vision - Headache - Glycosuria - *Nausea and vomiting* - Abdominal cramps

Diagnostic Studies of Hyperthyroidism

- Decreased TSH levels and elevated free T4 levels - Radioactive Iodine Uptake (RAIU) test to differentiate Graves' disease from other forms of thyroiditis. (Graves's disease will show 35-95% uptake, compare to <2% in thyroiditis)

Indications for corticosteroids

- Pain - Inflammation (e.g. RA) - Organ transplant - Respiratory distress

Clinical manifestations of Diabetes

- Polyuria, polydipsia, polyphagia - Weakness and fatigue (RBCs are sugar-coated, not effective at carrying O2) - Ketoacidosis - Recurrent infections (sugar is good medium for bacteria) - Prolonged wound healing (Low circulation) - Visual changes (Retinopathy)

Antidiuretic hormone (ADH), or vasopressin

- Promotes reabsorption of water - Vasoconstriction

Treatment for hypopituitarism

- Somatropin for low GH - Surgery, radiation for tumors - Hormone therapy

The client diagnosed with Cushing's disease has developed 1 peripheral edema. Theclient has received intravenous fluids at 100 mL/hr via IV pump for the past 79 hours. The client received IVPB medication in 50 mL of fluid every 6 hours for 15 doses. How many mL of fluid did the client receive? ________

The client has received 8,650 mL of intravenous fluid.

Causes of HHS

UTI, pneumonia, sepsis, acute illness, newly diagnosed type 2 diabetes 60yrs > and type 2

What should you teach the patient about a i131 test?

Used to test thyroid function, can't have an iodine allergy, stop thyroid meds 1 week before test, can't be pregnant, MAKE SURE DONE BEFORE OTHER IODINE TESTS!

posterior pituitary

Vasopressin (antidiuretic hormone [ADH]) Oxytocin

Can an insulin pump be delegated to an LPN?

Yes

Effects of aging on endocrine system

1) Decreased hormone production and secretion 2) Altered hormone metabolism and biologic activity 3) Decreased responsiveness of target tissues to hormone 4) Alterations in circadian rhythms

prediabetes

a condition in which the blood sugar level is higher than normal, but not high enough to be classified as type 2 diabetes

gestational diabetes

a form of diabetes mellitus that occurs during some pregnancies 4.6 to 9.2% of pregnancies tested using OGTT 24 to 28 weeks

diabetes mellitus

a group of metabolic disorders characterized by hyperglycemia defects in insulin secretion, impaired insulin utilization, or both

The nurse is teaching the client diagnosed with hyperthyroidism. Which information should be taught to the client? Select all that apply. 1. Notify the HCP if a three (3)-pound weight loss occurs in two (2) days. 2. Discuss ways to cope with the emotional lability. 3. Notify the HCP if taking over-the-counter medication. 4. Carry a medical identification card or bracelet.

1. Notify the HCP if a three (3)-pound weight loss occurs in two (2) days. 2. Discuss ways to cope with the emotional lability. 3. Notify the HCP if taking over-the-counter medication. 4. Carry a medical identification card or bracelet.

acute care with hypoglycemia

20 to 50ml of 50% dextrose if pt cannot swallow 1mg of glucagon by IM or subQ(deltoid)

A client is undergoing a left modified radical mastectomy for breast cancer. Postoperatively, blood pressure should be obtained from the client's right arm, and the left arm and hand should be elevated as much as possible to prevent which condition? a. Lymphedema b. Trousseau's sign c. IV infusion infiltration d. Muscle atrophy

a. Lymphedema

The nurse would expect the following lab results for a client diagnosed with DI. a. Urine specific gravity of 1.001 and serum sodium of 158mEq/L b. Urine specific gravity of 1.028 and serum sodium of 128 mEqL c. Urine osmolality of 300 mOsm/kg and serum sodium of 145 mEq/L d. Serum osmolality of 200mOsm/kg

a. Urine specific gravity of 1.001 and serum sodium of 158mEq/L

RN observes a nursing assistant (NA) caring for a patient after a hypophysectomy. Which action by the NA requires that the RN intervene? a. The NA lowers the HOB to the flat position.

a. The NA lowers the HOB to the flat position. R: HOB should be elevated about 30 degrees to decrease pressure on the sella turcica and avoid headaches.

After neck surgery, a patient develops hypoparathyroidism. The nurse should plan to teach the patient about a. calcium supplementation to normalize serum calcium levels.

a. calcium supplementation to normalize serum calcium levels.

A patient with acromegaly is treated with a transphenoidal hypophysectomy. Postoperatively, the nurse a. ensures that any clear nasal drainage is tested for glucose

a. ensures that any clear nasal drainage is tested for glucose

Signs and Symptoms of hyperparathyroidism may include a. hypercalcemia, hypophosphatemia, osteopenia, pancreatitis b. Hypercalcemia, hypophosphatemia, osteoarthritis, osteopenia c. hypercalcemia, Hypernatremia, and increased susceptibility of fractures d. Hyponatremia, hypercalcemia, muscle atrophy

a. hypercalcemia, hypophosphatemia, osteopenia, pancreatitis

A patient with symptoms of DI is admitted to the hospital for evaluation and treatment of the condition. An appropriate nursing diagnosis for the patient is a. insomnia related to waking at night to void.

a. insomnia related to waking at night to void.

The nurse determines that the patient in acute adrenal insufficiency is responding favorably to treatment when a. the patient appears alert and oriented

a. the patient appears alert and oriented rationale- confusion, irritability, disorientation, or depression is often present in the patient with Addison's

A patient with hyperthyroidism is treated with radioactive iodine (RAI) at a clinic. Before the patient is discharged, the nurse instructs the pt a. to monitor for symptoms of hypothyroidism, such as easy bruising and cold intolerance.

a. to monitor for symptoms of hypothyroidism, such as easy bruising and cold intolerance.

Patients with excessive cortisol levels would be expected to have which of the following signs and symptoms Select all that apply a. weight gain b. Round or moon face appearance c. Feeling of well being d. Excessive hair growth e. Fragile Skin f. Orthostatic hypertension

a. weight gain b. Round or moon face appearance d. Excessive hair growth e. Fragile Skin

Dawn phenomenon

an increase in blood glucose in the early morning, most likely due to increased glucose production in the liver after an overnight fast growth hormone and cortisol

pt teaching for bariatric

avoid alcohol, high protein foods, and foods high in sugar and fat eat slowly and chew food well take nutritional supplements monitor and report complications

how to prevent dumping syndrome

avoid, sugar, salt, and milk eat high protein, high fat, and low carb eat small meals and avoid consuming fluids with meals, lie down after meals take antispasmodic med to delay gastric emptying

A few weeks after an 82-year-old with a new diagnosis of type 2 diabetes has been placed on metformin (Glucophage) therapy and taught about appropriate diet and exercise, the home health nurse makes a visit. Which finding by the nurse is most important to discuss with the health care provider? a. Hemoglobin A1C level is 7.9%. b. Last eye exam was 18 months ago. c. Glomerular filtration rate is decreased. d. Patient has questions about the prescribed diet.

ANS: C The decrease in renal function may indicate a need to adjust the dose of metformin or change to a different medication. In older patients, the goal for A1C may be higher in order to avoid complications associated with hypoglycemia. The nurse will plan on scheduling the patient for an eye exam and addressing the questions about diet, but the biggest concern is the patient's decreased renal function

A 45 year old female suffering from progressive multiple sclerosis for the past 7 years has been on a long term corticosteroid therapy for management of symptoms and has developed Cushings Disease. Which type of acid base balance disturbance would you expect to find on this client's ABG results? a. Metabolic acidosis b. Metabolic alkalosis c. Respiratory acidosis d. Respiratory alkalosis

b. Metabolic alkalosis

Following a thyroidectomy, a patient develops carpal spasm while the nurse is taking a blood pressure on the left arm. Which action by the nurse is appropriate? b. Have the patient rebreathe using a paper bag.

b. Have the patient rebreathe using a paper bag.

The nurse writes a problem of "altered body image" for a 34-year-old client diagnosed with Cushing's disease. Which intervention should be implemented? 3. Use therapeutic communication to allow the client to discuss feelings.

3. Allowing the client to ventilate feelings about the altered body image is the most appropriate intervention.

peripheral arterial disease (PAD)

blockage of arteries carrying blood to the legs, arms, kidneys and other organs reduced blood flow

A client with PTH excess would most likely experience abnormal serum levels of a. Sodium and Chloride b. Potassium and glucose c. Urea and uric acid d. Calcium and phosphorus

d. Calcium and phosphorus

When providing discharge instructions to a patient following a subtotal thyroidectomy, the nurse advises the patient to c. avoid eating foods such as soybeans, turnips, and rutabagas

c. avoid eating foods such as soybeans, turnips, and rutabagas Rationale- the patient should avoid goitrogens, foods that inhibit thyroid, such as soybeans, turnips, rutabagas, and peanut skins.

A patient with Cushing syndrome returns to the surgical unit following an adrenalectomy. During the initial postoperative period, the nurse gives the highest priority to c. maintaining fluid and electrolyte status.

c. maintaining fluid and electrolyte status.

During assessment of the patient with acromegaly, the nurse would expect the patient to report c. undesirable changes in appearance

c. undesirable changes in appearance Rationale- the increased production of growth hormone in acromegaly causes an increase in thickness and width of bones and enlargement of soft tissues, resulting in marked changes in facial features, oily and coarse skin, and speech difficulties.

Which information obtained when caring for a pt who has just been admitted for evaluation of DI will be of greatest concern to the nurse? d. The patient is confused and lethargic.

d. The patient is confused and lethargic.

A pt is admitted with possible SIADH. Which information obtained by nurse is most important to communicate rapidly to health care provider? d. The pt has a serum sodium level of 119 mEq/L.

d. The pt has a serum sodium level of 119 mEq/L.

Following a transsphenoidal resection of a pituitary tumor, an important N assessment is a. monitoring hourly urine output. b. checking the dressings for serous drainage. c. palpating for dependent pitting edema. d. obtaining continuous pulse oximetry.

A R: After pituitary surgery, the pt is at risk for diabetes insipidus caused by cerebral edema and monitoring of urine output and urine specific gravity is essential. There will be no dressing when transsphenoidal approach is used. The pt is at risk for dehydration, not volume overload. The pt is not at high risk for problems with oxygenation, and continuous pulse oximetry is not needed.

Idiopathic diabetes

A form of type 1 diabetes that is strongly inherited and not related to autoimmunity. (hispanic, asian, africans)

What does a fruity breath indicate?

A high blood sugar

insulin is

A hormone produced by the pancreas or taken as a medication by many diabetic type 1 NEEDS type 2 sometimes needs or during periods of stress, illness or surgery

insulin

A hormone produced by the pancreas or taken as a medication by many diabetics

The client is admitted to the intensive care department diagnosed with myxedema coma. Which assessment data warrant immediate intervention by the nurse? 2. Pulse oximeter reading of 90%..

A pulse oximeter reading of less than 93% is significant.

fasting blood glucose MS

>110

What test is used to evaluate thyroid function?

i131

counterregulatory hormones

increase blood glucose by: stimulating glucose production and release by liver decreasing movement of glucose into cells

alcohol

inhibits gluconeogenesis (breakdown of glycogen to glucose) by liver can cause severe hypoglycemia 1 drink for women 2 drinks for men always consume carbs when drinking

three majors types of glucose lowering agents

insulin oral agents noninsulin injectable

hyperpituitarism

leads to acromegaly and cushings

fats recommended intake

less than 200mg/day of cholestrol

fructosamine test

measures average glucose levels over the past 3 weeks

HDL MS

men <40 women <50

risk factors for macrovascular

obesity **smoking HTN high fat intake lifestyle can be decreased

type 1 diabetes has to do with

pancreas -autoimmune destruction of b-cells -insuff production of insulin

major distinction between type 1 and type2

presence of endogenous insulin

mammography

recommends that women at average risk of breast cancer begin annual screening mammography at age 45. Women ages 40 to 45 should have the choice to start annual mammograms after the risks and potential benefits have been explained. Women age 55 and older may switch to mammograms every 2 years.

intervention siadh

reduce fluid intake admin diuretic admin isotonic or hypertonic solution admin demeclocyline

Carbohydrates

source of energy -whole grains -fruits -vegetables -low fat diary

oral meds that encourage the release of insulin when the patient is still able to produce insulin

sulfonylurea agents meglitinide

insulin resistance

the inability of the cells to respond to insulin pancreas compensates by secreting more insulin =hyperinsulinemia

Cyclic feeding schedule

the same as continuous feeding except that the infusion is stopped for a specified time in each 24-hour period, usually 6 hours or longer ("down time"). Down time typically occurs in the morning to allow bathing, treatments, and other activities.

Microvascular complications result from

thickening of vessel membranes in capillaries and arterioles

function of endocrine

to control overall body function and regulation, including metabolism, NUTRITION, ELIMINATION, temperature, FLUID AND ELECTROLYTE BALANCE, growth, and reproduction.

causes of hyperglycemia

too much food, ill, too little insulin, decreased activity, infection, stress, corticosteroids

causes of hypoglycemia

too much insulin,no food, too much/intense exercise, alcohol intake w/o food

screening for neuropathy

type 1: 5 years after 1st diagnosed then annually type 2: immediatly

basal insulin

type of insulin that controls glucose production between meals and overnight, is about 50% of daily needs, nearly constant levels

autonomic neuropathy

damage to nerves supplying the internal body structures that regulate functions such as blood pressure, heart rate, bowel and bladder emptying, and digestion

incarcerated hernia

descending portion of the bowel becomes tightly caught in hernial sac, compromising blood supply require surgical repair abdominal distension, tenderness, difficulty defacating

brain liver and blood cells

do not depend on insulin

dawn phenomenon

drop in insulin between 5-8 am patient will experience prebreakfast hyperglycemia treatment: giving higher dose of interediate acting insulin at 10 pm

complications of bypass

dumping syndrome (dumps too rapidly into small intestine overwhelming ability to digest, avoid sugary foods).. Must take multiple supplements for the rest of their lives (iron, calcium, cobalamin). Must eat in small amounts, and separate fluids from food when eating.

obesity contributes to what diseases

early onset of many chronic illnesses, such as osteoarthritis, diabetes mellitus, hypertension, coronary artery disease, pulmonary problems, delayed wound healing, and infection.

Somogyi effect

early-morning hyperglycemia that occurs as a result of nighttime hypoglycemic episodes due to high dose of insulin

common type 2 manifestations

fatigue recurrent infections recurrent vaginal yeast or candidal infections prolonged wound healing vision changes

All of the following are appropriate teaching points for someone infectious of mononucleosis except A. Complete bedrest is essential until the infection is gone B. NSAIDS should be administered to relieve symptoms C. Avoid contact sport especially if splenomegaly is present D. Drink plenty of fluids to avoid hydration

A. Complete bedrest is essential until the infection is gone

A client begins to cry uncontrollably after learning that a tumor removed from the lung is cancerous. how should the nurse deal with this situation A. Remain with the client and allow the client to cry B. Leave the room and find out if the client can have a sedative C. Ask the client exactly what the physician said D. Explain that its important for the client not to cry now.

A. Remain with the client and allow the client to cry

A nurse is teaching a client with pain about the common side effects of Percocet including Select All A. Renal Toxicity B. GI Upset C. Constipation D. Shock E. Bradycardia

A. Renal Toxicity B. GI Upset C. Constipation E. Bradycardia

When assessing the patient experiencing the onset of symptoms of type 1 diabetes, which question is most appropriate for the nurse to ask? a. "Have you lost any weight lately?" b. "How long have you felt anorexic?" c. "Is your urine unusually dark colored?" d. "Do you crave fluids containing sugar?"

ANS: A "Have you lost any weight lately?" Weight loss occurs because the body is no longer able to absorb glucose and starts to break down protein and fat for energy. The patient is thirsty but does not necessarily crave sugar-containing fluids. Increased appetite is a classic symptom of type 1 diabetes. With the classic symptom of polyuria, urine will be very dilute.

The nurse obtains the following information about a patient before administration of metformin (Glucophage). Which finding indicates a need to contact the health care provider before giving the metformin? a. The patient's blood glucose level is 166 mg/dL. b. The patient's blood urea nitrogen (BUN) level is 60 mg/dL. c. The patient is scheduled for a chest x-ray in an hour. d. The patient has gained 2 lb (0.9 kg) since yesterday.

ANS: B The patient's blood urea nitrogen (BUN) level is 60 mg/dL. The BUN indicates impending renal failure and metformin should not be used in patients with renal failure. The other findings are not contraindications to the use of metformin.

The client has developed iatrogenic Cushing's disease. Which statement is the scientific rationale for the development of this diagnosis? 2. The client has been taking steroid medications for an extended period for another disease process

"Iatrogenic" means a problem has been caused by a medical treatment or procedure

Corticosteroids (e.g. prednisone) possible negative effects when taken long-term

- Increase blood glucose level - Bone loss, thining bone - Decrease immune system - Thin skin, bruising, lower wound healing

How does exercise affect diabetes

- It lowers blood glucose levels - It decreases insulin resistance - Weight loss = decrease insulin resistance - Lower triglycerides and LDLs, raises HDLs - It can cause counter-regulatory hormones, temporary elevation of glucose; therefore, vigorous activities should be avoided if glucose > 250 and ketones are present in urine - It can lead to hypoglycemia

storage of insulin

-Do not heat/freeze. -In-use vials may be left at room temperature up to 4 weeks. -Extra insulin should be refrigerated. -Avoid exposure to direct sunlight. prefilled insulin stored with needle up to avoid clumping

type 1 patho

-Results from a severe, absolute lack of insulin caused by a reduction in beta-cell mass. -Islet cell destruction is believed due to: genetic susceptibility, autoimmunity, and an environmental insult. 80 to 90% pancreatic cells destroyed

The nurse is planning the care of a client diagnosed with Addison's disease. Which intervention should be included? 1. Administer steroid medications.

1 . Clients diagnosed with Addison's disease have adrenal gland hypofunction.

The nurse identifies the client problem "risk for imbalanced body temperature" for the client diagnosed with hypothyroidism. Which intervention should be included in the plan of care? 1. Discourage the use of an electric blanket.

1 . External heat sources (heating pads,electric or warming blankets) should be discouraged because they increase the risk of peripheral vasodilation and vascular collapse.

The client diagnosed with Addison's disease is admitted to the emergency department after a day at the lake. The client is lethargic, forgetful, and weak. Which intervention should the nurse implement? 1. Start an IV with an 18-gauge needle and infuse NS rapidly.

1 . The client was exposed to wind and sun at the lake during the hours prior to being admitted to the emergency department.

how to know if treatment and modifications to reduce DM are working

1 A1c less than 7% 2. premeal glucose of 70-130 mg/dl peak after meal blood glucose of less than 180 mg

How many x per day should lantus be given?

1 bc it lasts for 24 hours.

Recommended indications for pancreas transplantation when renal failure isn't present

1) History of frequent, acute, and severe metabolic complications (e.g. hypo/hyperglycemic, ketoacidosis) that requires medical attention 2) Clinical and emotional problems with use of exogenous insulin that is so severe as to incapacitating 3) Consistent failure of insulin-based management to prevent acute complications

The nurse is reviewing the laboratory results of a client diagnosed with multiple myeloma. Which of the following would the nurse expect to note specifically in this disorder? 1) Increased calcium level. 2) Increased white blood cells. 3) Decreased blood urea nitrogen level. 4) Decreased number of plasma cells in the one marrow.

1) Increased calcium level.

assessment of breast cancer

1.breast self examination 2.mass felt in upper out quadrant, beneath the nipple, or in axilla bloody or clear nipple discahrge skin dimpling skin edema or peau d orange

plasma samples or venous samples are

10 to 12% higher for BG

exercise

150min/week 30min/5days type 2: resistance training x3

carb counting

15g of carbohydrate = 1 carb serving amount of carbs per day depends on BG, age, weight, activity

Nursing considerations for hormone diagnostic tests - 17-ketosteroids - Cortisol (free) - Vanillylmandelic acid (VMA) - Fasting blood glucose - Oral glucose tolerance test (OGTT)

17-ketosteroids - 24-hr urine collection. Specimen refrigerated Cortisol (free) - 24-hr urine collection. Avoid stressful situation Vanillylmandelic acid (VMA) - 24-hr urine collection at pH < 3.0 with HCL acid preservative. Keep on ice. - Discuss d/c drugs 3 days before urine collection Fasting blood glucose - Fast 8-12 hours, water is permitted, norm: 70-99 Oral glucose tolerance test (OGTT) - Fast 8-12 hours - Pt. drinks 75g glucose, sample at 30,60,120min - Norm: <100 base, <200 at 30-60m, <140 at 120min - >200 at 120 min indicates diabetes mellitus

The client diagnosed with a pituitary tumor developed syndrome of inappropriate antidiuretic hormone (SIADH). Which interventions should the nurse implement? 2. Assess for nausea and vomiting and weigh daily.

2. Early signs and symptoms are nausea and vomiting.

The nurse is admitting a client to the neurological intensive care unit who is postoperative transsphenoidal hypophysectomy. Which data warrant immediate intervention? 2. The client has an output of 2,500 mL since surgery and an intake of 1,000 mL.

2. The output is more than double the intake in a short time.

Which question should the nurse ask when assessing the client for an endocrine dysfunction? 2. "Have you had any unexplained weight loss?"

2. Weight loss with normal appetite may indicate hyperthyroidism.

Which sign/symptom indicates to the nurse the client is experiencing hyperparathyroidism? 2. A positive Chvostek's sign.

2. When a sharp tapping over the facial nerve elicits a spasm or twitching of the mouth, nose, or eyes, the client is hypocalcemic

A nurse is monitoring a client for signs and symptoms related to superior vena cava syndrome. Which of the following is an early sign of this oncological emergency? 1) Cyanosis. 2) Arm edema. 3) Periorbital edema. 4) Mental status change.

3) Periorbital edema.

The nurse is admitting a client diagnosed with primary adrenal cortex insufficiency (Addison's disease). Which clinical manifestations should the nurse expect to assess? 3. Bronze pigmentation, hypotension, and anorexia.

3. Bronze pigmentation of the skin, particularly of the knuckles and other areas of skin creases, occurs in Addison's disease.

The client diagnosed with Cushing's disease has undergone a unilateral adrenalectomy. Which discharge instructions should the nurse discuss with the client? 3. Explain the signs and symptoms of infection and when to call the health-care provider.

3. Notifying the HCP if signs/symptoms of infection develop is an instruction given to all surgical clients on discharge.

The nurse is discussing the endocrine system with the client. Which endocrine gland secretes epinephrine and norepinephrine? 3. The adrenal medulla.

3. The adrenal medulla secretes the catecholamines epinephrine and norepinephrine.

The client diagnosed with hypothyroidism is prescribed the thyroid hormone levothyroxine (Synthroid). Which assessment data indicate the medication has been effective? 3. The client's temperature is WNL.

3. The client with hypothyroidism frequently has a subnormal temperature, so a temperature WNL indicates the medication is effective.

The nurse is assessing a client in an outpatient clinic. Which assessment data are a risk factor for developing pheochromytoma? 3. A family history of adrenal tumors.

3. There is a high incidence of pheochromocytomas in family members with adrenal tumors,

The charge nurse of an intensive care unit is making assignments for the night shift.Which client should be assigned to the most experienced intensive care nurse? 3. The client diagnosed with Addison's disease who is lethargic and has a BP of 80/45, P 124, and R 28.

3. This client has a low blood pressure and tachycardia. This client may be experiencing an addisonian crisis

The nurse is preparing to administer the following medications. Which medication should the nurse question administering? 3. The loop diuretic to the client with a potassium level of 3.3 mEq/L.

3. This potassium level is below normal, which is 3.5 to 5.5 mEq/L.

Which statement made by the client makes the nurse suspect the client is experiencing hyperthyroidism? 4. "I have noticed all my collars are getting tighter."

4. The thyroid gland (in the neck) en larges as a result of the increased need for thyroid hormone production; an enlarged gland is called a goiter.

metabolic syndrome risks

60years old one in 3 adults

BP MS

>130 >85

Triglycerides MS

>150

waist circumference MS

>40 in men >35 in women

Several days before admission, a client reports finding a small lump in the left breast near the nipple. What should the nurse tell the client to do? a) Inform the physician immediately b) Squeeze the nipple to check for drainage c) Check the area after the next menstrual period d) Put a heating pad on the area to reduce inflammation

A - The client should notify the physician immediately because a breast lump may be a sign of breast cancer. The client shouldn't squeeze the nipple to check for drainage until the physician examines the area. The client shouldn't wait until after the next menstrual period to inform the physician of the breast lump because it would have no effect on a breast lump.

RN observes a nursing assistant (NA) caring for a patient after a hypophysectomy. Which action by the NA requires that the RN intervene? a. The NA lowers the HOB to the flat position. b. The NA cautions the patient to avoid coughing. c. The NA cleans the patient's mouth with a swab. d. NA collects a urine specimen for specific gravity

A R: HOB should be elevated about 30 degrees to decrease pressure on the sella turcica and avoid headaches. The other actions by the NA are appropriate after this surgery. (Cognitive Level: Application Text : p. 1293 NProcess: Implementation NCLEX: Safe and Effective Care Enviro)

A patient with symptoms of DI is admitted to the hospital for evaluation and treatment of the condition. An appropriate nursing diagnosis for the patient is a. sleep disturbances related to waking at night to void. b. risk for impaired skin integrity related to generalized edema. c. excess fluid volume related to intake greater than output. d. activity intolerance related to muscle cramps and weakness.

A R: Nocturia occurs as a result of the polyuria caused by diabetes insipidus. Edema will not be expected because dehydration is a concern with polyuria. The pt drinks large amnts of fluid to compensate for losses experienced from diuresis. The pt's fluid and electrolyte status remain normal as long as the patient's oral intake can keep up w fluid losses, muscle cramps and weakness arent concerns.

A patient with hyperthyroidism is treated with radioactive iodine (RAI) at a clinic. Before the patient is discharged, the nurse instructs the pt a. to monitor for symptoms of hypothyroidism, such as easy bruising and cold intolerance. b. to discontinue the antithyroid medications taken before the radioactive therapy. c. that symptoms of hyperthyroidism should be relieved in about a week. d. about radioactive precautions to take with urine, stool, and other body secretions.

A R: There is a high incidence of post-radiation hypothyroidism after RAI, and the pt should be monitored for symptoms of hypothyroidism. RAI has a delayed response, with maximum effect not seen for 2-3 months, and pt will continue to take antithyroid medications during this time. The therapeutic dose of radioactive iodine is low enough that no radiation safety precautions are needed

A patient is hospitalized with possible SIADH. The patient is confused and reports a headache, muscle cramps, and twitching. The nurse would expect the initial laboratory results to include a a. serum sodium of 125 mEq/L (125 mmol/L). b. hematocrit of 52%. c. blood urea nitrogen (BUN) of 22 mg/dl (11.5 mmol/L). d. serum chloride of 110 mEq/L (110 mmol/L).

A R: When water is retained, the serum sodium level will drop below normal, causing the CMs reported by the patient. The hematocrit will decrease because of the dilution caused by water retention. The BUN is not helpful in diagnosis of SIADH and this BUN value is increased. The serum chloride level will usually decrease along with the sodium level. This chloride value is elevated.

A client is diagnosed with terminal inoperable lung cancer tells the nurse that he would like to seek spiritual advice. To provide appropriate spiritual advise the nurse should A. Contact the clients spiritual adviser B. Contact her own spiritual adviser C. Contact clergy of another faith D. Tell the client he may call his spiritual adviser after he's discharged

A. Contact the clients spiritual adviser

Which of the following is not appropriate teaching for clients suffering from GI disturbances secondary to antioplastic therapy A. Eat large meals because it is important to maintain good nutrition status to fight the cancer B. Eat high calorie, high protein foods C. Eat small frequent meals D. Drink Plenty of fluids to prevent dehydration

A. Eat large meals because it is important to maintain good nutrition status to fight the cancer

A client is diagnosed with megoblastic (macrocytic anemia caused by Vitamin B 12 deficiency. The physician begins the client on Cyanocobalamine (B12) 100 mcg IM daily. Which substance influences vitamin B12 absorption A. Intrinsic factor B. Hydrochloric acid C. Histamine D. Liver enzymes

A. Intrinsic Factor

The nurse writes a client problem of activity intolerance for a client diagnosed with anemia of chronic illness secondary to renal failure. Which intervention should the nurse implement A. Pace activities according to tolerance B. Provide supplements high in iron and vitamins C. Administer packed red blood cells D. Monitor vital signs every 4 hours

A. Pace activities according to tolerance

The nurse is admitting a 24 year old African American female client with a diagnosis of rule-out anemia. The client has a history of gastric bypass surgery for obesity four years ago. Current assessment findings include height: 5'5", weight 75kg, P 110, R 27, and BP 104/66, Pale mucous membranes and dyspnea on exertion. Which type of anemia would the nurse suspect the client has developed? A. Pernicious anemia B. Folate deficiency C. Iron Deficiency D. Thalassemia Anemia

A. Pernicious anemia

A home care nurse is instructing the parents of a child with iron deficiency anemia regarding the administration of a liquid oral iron supplement. The nurse tells the mother to A. administer the iron through a straw B. administer the iron at mealtimes C. add the iron to the formula for easy administration D. mix the iron with cereal to administer

A. administer the iron through a straw

In which order will the nurse take these steps to prepare NPH 20 units and regular insulin 2 units using the same syringe? (Put a comma and a space between each answer choice [A, B, C, D, E]). a. Rotate NPH vial. b. Withdraw regular insulin. c. Withdraw 20 units of NPH. d. Inject 20 units of air into NPH vial. e. Inject 2 units of air into regular insulin vial

ANS: A, D, E, B, C When mixing regular insulin with NPH, it is important to avoid contact between the regular insulin and the additives in the NPH that slow the onset, peak, and duration of activity in the longer-acting insulin.

Which question during the assessment of a diabetic patient will help the nurse identify autonomic neuropathy? a. "Do you feel bloated after eating?" b. "Have you seen any skin changes?" c. "Do you need to increase your insulin dosage when you are stressed?" d. "Have you noticed any painful new ulcerations or sores on your feet?"

ANS: A Autonomic neuropathy can cause delayed gastric emptying, which results in a bloated feeling for the patient. The other questions are also appropriate to ask but would not help in identifying autonomic neuropathy

In order to assist an older diabetic patient to engage in moderate daily exercise, which action is most important for the nurse to take? a. Determine what type of activities the patient enjoys. b. Remind the patient that exercise will improve self-esteem. c. Teach the patient about the effects of exercise on glucose level. d. Give the patient a list of activities that are moderate in intensity.

ANS: A Because consistency with exercise is important, assessment for the types of exercise that the patient finds enjoyable is the most important action by the nurse in ensuring adherence to an exercise program. The other actions will also be implemented but are not the most important in improving compliance

Which nursing action can the nurse delegate to unlicensed assistive personnel (UAP) who are working in the diabetic clinic? a. Measure the ankle-brachial index. b. Check for changes in skin pigmentation. c. Assess for unilateral or bilateral foot drop. d. Ask the patient about symptoms of depression.

ANS: A Checking systolic pressure at the ankle and brachial areas and calculating the ankle-brachial index is a procedure that can be done by UAP who have been trained in the procedure. The other assessments require more education and critical thinking and should be done by the registered nurse (RN).

A female patient is scheduled for an oral glucose tolerance test. Which information from the patient's health history is most important for the nurse to communicate to the health care provider? a. The patient uses oral contraceptives. b. The patient runs several days a week. c. The patient has been pregnant three times. d. The patient has a family history of diabetes.

ANS: A Oral contraceptive use may falsely elevate oral glucose tolerance test (OGTT) values. Exercise and a family history of diabetes both can affect blood glucose but will not lead to misleading information from the OGTT. History of previous pregnancies may provide informational about gestational glucose tolerance, but will not lead to misleading information from the OGTT

A 32-year-old patient with diabetes is starting on intensive insulin therapy. Which type of insulin will the nurse discuss using for mealtime coverage? a. Lispro (Humalog) b. Glargine (Lantus) c. Detemir (Levemir) d. NPH (Humulin N)

ANS: A Rapid- or short-acting insulin is used for mealtime coverage for patients receiving intensive insulin therapy. NPH, glargine, or detemir will be used as the basal insulin

Which statement by the patient indicates a need for additional instruction in administering insulin? a. "I need to rotate injection sites among my arms, legs, and abdomen each day." b. "I can buy the 0.5 mL syringes because the line markings will be easier to see." c. "I should draw up the regular insulin first after injecting air into the NPH bottle." d. "I do not need to aspirate the plunger to check for blood before injecting insulin."

ANS: A Rotating sites is no longer recommended because there is more consistent insulin absorption when the same site is used consistently. The other patient statements are accurate and indicate that no additional instruction is needed

A patient who has type 1 diabetes plans to take a swimming class daily at 1:00 PM. The clinic nurse will plan to teach the patient to a. check glucose level before, during, and after swimming. b. delay eating the noon meal until after the swimming class. c. increase the morning dose of neutral protamine Hagedorn (NPH) insulin. d. time the morning insulin injection so that the peak occurs while swimming.

ANS: A The change in exercise will affect blood glucose, and the patient will need to monitor glucose carefully to determine the need for changes in diet and insulin administration. Because exercise tends to decrease blood glucose, patients are advised to eat before exercising. Increasing the morning NPH or timing the insulin to peak during exercise may lead to hypoglycemia, especially with the increased exercise.

A 55-year-old female patient with type 2 diabetes has a nursing diagnosis of imbalanced nutrition: more than body requirements. Which goal is most important for this patient? a. The patient will reach a glycosylated hemoglobin level of less than 7%. b. The patient will follow a diet and exercise plan that results in weight loss. c. The patient will choose a diet that distributes calories throughout the day. d. The patient will state the reasons for eliminating simple sugars in the diet.

ANS: A The complications of diabetes are related to elevated blood glucose, and the most important patient outcome is the reduction of glucose to near-normal levels. The other outcomes also are appropriate but are not as high in priority

A patient receives aspart (NovoLog) insulin at 8:00 AM. Which time will it be most important for the nurse to monitor for symptoms of hypoglycemia? a. 10:00 AM b. 12:00 AM c. 2:00 PM d. 4:00 PM

ANS: A The rapid-acting insulins peak in 1 to 3 hours. The patient is not at a high risk for hypoglycemia at the other listed times, although hypoglycemia may occur

The nurse has been teaching the patient to administer a dose of 10 units of regular insulin and 28 units of NPH insulin. The statement by the patient that indicates a need for additional instruction is, a. "I need to rotate injection sites among my arms, legs, and abdomen each day." b. "I will buy the 0.5 mL syringes because the line markings will be easier to see." c. "I should draw up the regular insulin first after injecting air into the NPH bottle." d. "I do not need to aspirate the plunger to check for blood before injecting insulin."

ANS: A "I need to rotate injection sites among my arms, legs, and abdomen each day." Rotating sites is no longer recommended because there is more consistent insulin absorption when the same site is used consistently. The other patient statements are accurate and indicate that no additional instruction is needed.

The health care provider suspects the Somogyi effect in a patient whose 7:00 AM blood glucose is 220 mg/dL. Which action will the nurse plan to take? a. Check the patient's blood glucose at 3:00 AM. b. Administer a larger dose of long-acting insulin. c. Educate about the need to increase the rapid-acting insulin dose. d. Remind the patient about the need to avoid snacking at bedtime.

ANS: A Check the patient's blood glucose at 3:00 AM If the Somogyi effect is causing the patient's increased morning glucose level, the patient will experience hypoglycemia between 2 and 4 AM. The dose of insulin will be reduced, rather than increased. A bedtime snack is used to prevent hypoglycemic episodes during the night.

A patient with type 2 diabetes has sensory neuropathy of the feet and legs and peripheral arterial disease. Which information will the nurse include in patient teaching? a. Choose flat-soled leather shoes. b. Set heating pads on a low temperature. c. Buy callus remover for corns or calluses. d. Soak the feet in warm water for an hour every day.

ANS: A Choose flat-soled leather shoes. The patient is taught to avoid high heels and that leather shoes are preferred. The feet should be washed, but not soaked, in warm water daily. Heating pad use should be avoided. Commercial callus and corn removers should be avoided. The patient should see a specialist to treat these problems.

Which information from the patient's health history is most important for the nurse to communicate to the health care provider when a patient has an order for an oral glucose tolerance test? a. The patient uses oral contraceptives. b. The patient runs several days a week. c. The patient has a family history of diabetes. d. The patient had a viral illness 2 months ago.

ANS: A The patient uses oral contraceptives. Oral contraceptive use may falsely elevate oral glucose tolerance test (OGTT) values. A viral illness 2 months previously may be associated with the onset of type 1 diabetes but will not falsely affect the OGTT. Exercise and a family history of diabetes both can affect blood glucose but will not lead to misleading information from the OGTT.

A patient who has just been diagnosed with type 2 diabetes has a nursing diagnosis of imbalanced nutrition: more than body requirements. Which patient goal is most important for this patient? a. The patient will have a glycosylated hemoglobin level of less than 7%. b. The patient will have a diet and exercise plan that results in weight loss. c. The patient will choose a diet that distributes calories throughout the day. d. The patient will state the reasons for eliminating simple sugars in the diet.

ANS: A The patient will have a glycosylated hemoglobin level of less than 7%. The complications of diabetes are related to elevated blood glucose, and the most important patient outcome is the reduction of glucose to near-normal levels. The other outcomes also are appropriate but are not as high in priority.

Which patient action indicates good understanding of the nurse's teaching about administration of aspart (NovoLog) insulin? a. The patient avoids injecting the insulin into the upper abdominal area. b. The patient cleans the skin with soap and water before insulin administration. c. The patient stores the insulin in the freezer after administering the prescribed dose. d. The patient pushes the plunger down while removing the syringe from the injection site.

ANS: B Cleaning the skin with soap and water or with alcohol is acceptable. Insulin should not be frozen. The patient should leave the syringe in place for about 5 seconds after injection to be sure that all the insulin has been injected. The upper abdominal area is one of the preferred areas for insulin injection

An unresponsive patient with type 2 diabetes is brought to the emergency department and diagnosed with hyperosmolar hyperglycemic syndrome (HHS). The nurse will anticipate the need to a. give a bolus of 50% dextrose. b. insert a large-bore IV catheter. c. initiate oxygen by nasal cannula. d. administer glargine (Lantus) insulin.

ANS: B HHS is initially treated with large volumes of IV fluids to correct hypovolemia. Regular insulin is administered, not a long-acting insulin. There is no indication that the patient requires oxygen. Dextrose solutions will increase the patient's blood glucose and would be contraindicated

Which laboratory value reported to the nurse by the unlicensed assistive personnel (UAP) indicates the most urgent need for the nurse's assessment of the patient? a. Bedtime glucose of 140 mg/dL b. Noon blood glucose of 52 mg/dL c. Fasting blood glucose of 130 mg/dL d. 2-hr postprandial glucose of 220 mg/dL

ANS: B The nurse should assess the patient with a blood glucose level of 52 mg/dL for symptoms of hypoglycemia and give the patient a carbohydrate-containing beverage such as orange juice. The other values are within an acceptable range or not immediately dangerous for a diabetic patient

The nurse identifies a need for additional teaching when the patient who is self-monitoring blood glucose a. washes the puncture site using warm water and soap. b. chooses a puncture site in the center of the finger pad. c. hangs the arm down for a minute before puncturing the site. d. says the result of 120 mg indicates good blood sugar control.

ANS: B The patient is taught to choose a puncture site at the side of the finger pad because there are fewer nerve endings along the side of the finger pad. The other patient actions indicate that teaching has been effective

Which information will the nurse include when teaching a 50-year-old patient who has type 2 diabetes about glyburide (Micronase, DiaBeta, Glynase)? a. Glyburide decreases glucagon secretion from the pancreas. b. Glyburide stimulates insulin production and release from the pancreas. c. Glyburide should be taken even if the morning blood glucose level is low. d. Glyburide should not be used for 48 hours after receiving IV contrast media.

ANS: B The sulfonylureas stimulate the production and release of insulin from the pancreas. If the glucose level is low, the patient should contact the health care provider before taking the glyburide, because hypoglycemia can occur with this class of medication. Metformin should be held for 48 hours after administration of IV contrast media, but this is not necessary for glyburide. Glucagon secretion is not affected by glyburide

An active 28-year-old male with type 1 diabetes is being seen in the endocrine clinic. Which finding may indicate the need for a change in therapy? a. Hemoglobin A1C level 6.2% b. Blood pressure 146/88 mmHg c. Heart rate at rest 58 beats/minute d. High density lipoprotein (HDL) level 65 mg/dL

ANS: B To decrease the incidence of macrovascular and microvascular problems in patients with diabetes, the goal blood pressure is usually 130/80. An A1C less than 6.5%, a low resting heart rate (consistent with regular aerobic exercise in a young adult), and an HDL level of 65 mg/dL all indicate that the patient's diabetes and risk factors for vascular disease are well controlled.

A diabetic patient who has reported burning foot pain at night receives a new prescription. Which information should the nurse teach the patient about amitriptyline (Elavil)? a. Amitriptyline decreases the depression caused by your foot pain. b. Amitriptyline helps prevent transmission of pain impulses to the brain. c. Amitriptyline corrects some of the blood vessel changes that cause pain. d. Amitriptyline improves sleep and makes you less aware of nighttime pain.

ANS: B Tricyclic antidepressants decrease the transmission of pain impulses to the spinal cord and brain. Tricyclic antidepressants also improve sleep quality and are used for depression, but that is not the major purpose for their use in diabetic neuropathy. The blood vessel changes that contribute to neuropathy are not affected by tricyclic antidepressants

Which question by the nurse will help identify autonomic neuropathy in a diabetic patient? a. "Have you observed any recent skin changes?" b. "Do you notice any bloating feeling after eating?" c. "Do you need to increase your insulin dosage when you are stressed?" d. "Have you noticed any painful new ulcerations or sores on your feet?"

ANS: B "Do you notice any bloating feeling after eating?" Autonomic neuropathy can cause delayed gastric emptying, which results in a bloated feeling for the patient. The other questions also are appropriate to ask, but would not help in identifying autonomic neuropathy.

Which action should the nurse take first when teaching a patient who is newly diagnosed with type 2 diabetes about home management of the disease? a. Ask the patient's family to participate in the diabetes education program. b. Assess the patient's perception of what it means to have diabetes mellitus. c. Demonstrate how to check glucose using capillary blood glucose monitoring. d. Discuss the need for the patient to actively participate in diabetes management.

ANS: B Assess the patient's perception of what it means to have diabetes mellitus. Before planning education, the nurse should assess the patient's interest in and ability to self-manage the diabetes. After assessing the patient, the other nursing actions may be appropriate, but planning needs to be individualized to each patient.

Which action is most important for the nurse to take in order to assist a diabetic patient to engage in moderate daily exercise? a. Remind the patient that exercise will improve self-esteem. b. Determine what type of exercise activities the patient enjoys. c. Give the patient a list of activities that are moderate in intensity. d. Teach the patient about the effects of exercise on glucose level.

ANS: B Determine what type of exercise activities the patient enjoys. Since consistency with exercise is important, assessment for the types of exercise that the patient finds enjoyable is the most important action by the nurse in ensuring adherence to an exercise program. The other actions also will be implemented, but are not the most important in improving compliance.

A pregnant patient who has no personal history of diabetes, but does have a parent who is diabetic is scheduled for the first prenatal visit. Which action will the nurse plan to take on this initial visit? a. Teach about appropriate use of regular insulin. b. Discuss the need for a fasting blood glucose level. c. Schedule an oral glucose tolerance test for the twenty fourth week of pregnancy. d. Provide education about increased risk for fetal problems with gestational diabetes.

ANS: B Discuss the need for a fasting blood glucose level. Patients at high risk for gestational diabetes should be screened for diabetes on the initial prenatal visit. An oral glucose tolerance test also may be used to check for diabetes, but it would be done before the twenty fourth week. The other actions also may be needed (depending on whether the patient develops gestational diabetes), but they are not the first actions that the nurse should take.

Intramuscular glucagon is administered to an unresponsive patient for treatment of hypoglycemia. Which action should the nurse take after the patient regains consciousness? a. Assess the patient for symptoms of hyperglycemia. b. Give the patient a snack of crackers and peanut butter. c. Have the patient drink a glass of orange juice or nonfat milk. d. Administer a continuous infusion of 5% dextrose for 24 hours.

ANS: B Give the patient a snack of crackers and peanut butter. Rebound hypoglycemia can occur after glucagon administration, but having a meal containing complex carbohydrates plus protein and fat will help prevent hypoglycemia. Orange juice and nonfat milk will elevate blood sugar rapidly, but the cheese and crackers will stabilize blood sugar. Administration of glucose intravenously might be used in patients who were unable to take in nutrition orally. The patient should be assessed for symptoms of hypoglycemia after glucagon administration.

Which information will the nurse include when teaching a patient who has type 2 diabetes about glyburide (Micronase, DiaBeta, Glynase)? a. Glyburide decreases glucagon secretion from the pancreas. b. Glyburide stimulates insulin production and release from the pancreas. c. Glyburide should be taken even if the morning blood glucose level is low. d. Glyburide should not be used for 48 hours after receiving IV contrast media.

ANS: B Glyburide stimulates insulin production and release from the pancreas. The sulfonylureas stimulate the production and release of insulin from the pancreas. If the glucose level is low, the patient should contact the health care provider before taking the glyburide, because hypoglycemia can occur with this category of medication. Metformin should be held for 48 hours after administration of IV contrast media, but this is not necessary for glyburide. Glucagon secretion is not affected by glyburide.

A diabetic patient is admitted with ketoacidosis and the health care provider writes these orders. Which order should the nurse implement first? a. Administer regular IV insulin 30 U. b. Infuse 1 liter of normal saline per hour. c. Give sodium bicarbonate 50 mEq IV push. d. Start an infusion of regular insulin at 50 U/hr.

ANS: B Infuse 1 liter of normal saline per hour. The most urgent patient problem is the hypovolemia associated with diabetic ketoacidosis (DKA), and the priority is to infuse IV fluids. The other actions can be accomplished after the infusion of normal saline is initiated.

Which of these laboratory values, noted by the nurse when reviewing the chart of a hospitalized diabetic patient, indicates the need for rapid assessment of the patient? a. Hb A1C of 5.8% b. Noon blood glucose of 52 mg/dL c. Hb A1Cof 6.9% d. Fasting blood glucose of 130 mg/dL

ANS: B Noon blood glucose of 52 mg/dL The nurse should assess the patient with a blood glucose level of 52 mg/dL for symptoms of hypoglycemia, and give the patient some carbohydrate-containing beverage such as orange juice. The other values are within an acceptable range for a diabetic patient.

When the nurse is assessing a patient who is recovering from an episode of diabetic ketoacidosis, the patient reports feeling anxious, nervous, and sweaty. Which action should the nurse take first? a. Administer 1 mg glucagon subcutaneously. b. Obtain a glucose reading using a finger stick. c. Have the patient drink 4 ounces of orange juice. d. Give the scheduled dose of lispro (Humalog) insulin.

ANS: B Obtain a glucose reading using a finger stick. The patient's clinical manifestations are consistent with hypoglycemia and the initial action should be to check the patient's glucose with a finger stick or order a stat blood glucose. If the glucose is low, the patient should ingest a rapid-acting carbohydrate, such as orange juice. Glucagon might be given if the patient's symptoms become worse or if the patient is unconscious. Administration of lispro would drop the patient's glucose further.

A patient is admitted with diabetic ketoacidosis (DKA) and has a serum potassium level of 2.9 mEq/L. Which action prescribed by the health care provider should the nurse take first? a. Infuse regular insulin at 20 U/hr. b. Place the patient on a cardiac monitor. c. Administer IV potassium supplements. d. Obtain urine glucose and ketone levels.

ANS: B Place the patient on a cardiac monitor. Hypokalemia can lead to potentially fatal dysrhythmias such as ventricular tachycardia and ventricular fibrillation, which would be detected with ECG monitoring. Since potassium must be infused over at least 1 hour, the nurse should initiate cardiac monitoring before infusion of potassium. Insulin should not be administered without cardiac monitoring, since insulin infusion will further decrease potassium levels. Urine glucose and ketone levels are not urgently needed to manage the patient's care.

After the nurse has finished teaching a patient about self-administration of the prescribed aspart (NovoLog) insulin, which patient action indicates good understanding of the teaching? a. The patient avoids injecting the insulin into the upper abdominal area. b. The patient cleans the skin with soap and water before insulin administration. c. The patient places the insulin back in the freezer after administering the prescribed insulin dose. d. The patient pushes the plunger down and immediately removes the syringe from the injection site.

ANS: B The patient cleans the skin with soap and water before insulin administration. Cleaning the skin with soap and water or with alcohol is acceptable. Insulin should not be frozen. The patient should leave the syringe in place for about 5 seconds after injection to be sure that all the insulin has been injected. The upper abdominal area is one of the preferred areas for insulin injection.

Which patient action indicates a good understanding of the nurse's teaching about the use of an insulin pump? a. The patient changes the site for the insertion site every week. b. The patient programs the pump to deliver an insulin bolus after eating. c. The patient takes the pump off at bedtime and starts it again each morning. d. The patient states that diet will be less flexible when using the insulin pump.

ANS: B The patient programs the pump to deliver an insulin bolus after eating. In addition to the basal rate of insulin infusion, the patient will adjust the pump to administer a bolus after each meal, with the dosage depending on the oral intake. The insertion site should be changed every 2 or 3 days. There is more flexibility in diet and exercise when an insulin pump is used. The pump will deliver a basal insulin rate 24 hours a day.

A patient with type 1 diabetes has been using self-monitoring of blood glucose (SMBG) as part of diabetes management. During evaluation of the patient's technique of SMBG, the nurse identifies a need for additional teaching when the patient a. washes the puncture site using soap and warm water. b. chooses a puncture site in the center of the finger pad. c. hangs the arm down for a minute before puncturing the site. d. says the result of 130 mg indicates good blood sugar control.

ANS: B chooses a puncture site in the center of the finger pad. The patient is taught to choose a puncture site at the side of the finger pad. The other patient actions indicate that teaching has been effective.

A diagnosis of hyperglycemic hyperosmolar nonketotic coma (HHNC) is made for a patient with type 2 diabetes who is brought to the emergency department in an unresponsive state. The nurse will anticipate the need to a. give 50% dextrose as a bolus. b. insert a large-bore IV catheter. c. initiate oxygen by nasal cannula. d. administer glargine (Lantus) insulin.

ANS: B insert a large-bore IV catheter. HHNC is initially treated with large volumes of IV fluids to correct hypovolemia. Regular insulin is administered, not a long-acting insulin. There is no indication that the patient requires oxygen. Dextrose solutions will increase the patient's blood glucose and would be contraindicated.

A patient with type 2 diabetes that is well-controlled with metformin (Glucophage) develops an allergic rash to an antibiotic and the health care provider prescribes prednisone (Deltasone). The nurse will anticipate that the patient may a. need a diet higher in calories while receiving prednisone. b. require administration of insulin while taking prednisone. c. develop acute hypoglycemia while taking the prednisone. d. have rashes caused by metformin-prednisone interactions.

ANS: B require administration of insulin while taking prednisone. Glucose levels increase when patients are taking corticosteroids, and insulin may be required to control blood glucose. Hypoglycemia is not a side effect of prednisone. Rashes are not an adverse effect caused by taking metformin and prednisone simultaneously. The patient may have an increased appetite when taking prednisone, but will not need a diet that is higher in calories.

To monitor for complications in a patient with type 2 diabetes, which tests will the nurse in the diabetic clinic schedule at least annually (select all that apply)? a. Chest x-ray b. Blood pressure c. Serum creatinine d. Urine for microalbuminuria e. Complete blood count (CBC) f. Monofilament testing of the foot

ANS: B, C, D, F Blood pressure, serum creatinine, urine testing for microalbuminuria, and monofilament testing of the foot are recommended at least annually to screen for possible microvascular and macrovascular complications of diabetes. Chest x-ray and CBC might be ordered if the diabetic patient presents with symptoms of respiratory or infectious problems but are not routinely included in screening

Which statement by a nurse to a patient newly diagnosed with type 2 diabetes is correct? a. Insulin is not used to control blood glucose in patients with type 2 diabetes. b. Complications of type 2 diabetes are less serious than those of type 1 diabetes. c. Changes in diet and exercise may control blood glucose levels in type 2 diabetes. d. Type 2 diabetes is usually diagnosed when the patient is admitted with a hyperglycemic coma.

ANS: C For some patients with type 2 diabetes, changes in lifestyle are sufficient to achieve blood glucose control. Insulin is frequently used for type 2 diabetes, complications are equally severe as for type 1 diabetes, and type 2 diabetes is usually diagnosed with routine laboratory testing or after a patient develops complications such as frequent yeast infections.

The health care provider suspects the Somogyi effect in a 50-year-old patient whose 6:00 AM blood glucose is 230 mg/dL. Which action will the nurse teach the patient to take? a. Avoid snacking at bedtime. b. Increase the rapid-acting insulin dose. c. Check the blood glucose during the night d. Administer a larger dose of long-acting insulin.

ANS: C If the Somogyi effect is causing the patient's increased morning glucose level, the patient will experience hypoglycemia between 2:00 and 4:00 AM. The dose of insulin will be reduced, rather than increased. A bedtime snack is used to prevent hypoglycemic episodes during the night

A 26-year-old female with type 1 diabetes develops a sore throat and runny nose after caring for her sick toddler. The patient calls the clinic for advice about her symptoms and a blood glucose level of 210 mg/dL despite taking her usual glargine (Lantus) and lispro (Humalog) insulin. The nurse advises the patient to a. use only the lispro insulin until the symptoms are resolved. b. limit intake of calories until the glucose is less than 120 mg/dL. c. monitor blood glucose every 4 hours and notify the clinic if it continues to rise. d. decrease intake of carbohydrates until glycosylated hemoglobin is less than 7%.

ANS: C Infection and other stressors increase blood glucose levels and the patient will need to test blood glucose frequently, treat elevations appropriately with lispro insulin, and call the health care provider if glucose levels continue to be elevated. Discontinuing the glargine will contribute to hyperglycemia and may lead to diabetic ketoacidosis (DKA). Decreasing carbohydrate or caloric intake is not appropriate because the patient will need more calories when ill. Glycosylated hemoglobin testing is not used to evaluate short-term alterations in blood glucose

When a patient with type 2 diabetes is admitted for a cholecystectomy, which nursing action can the nurse delegate to a licensed practical/vocational nurse (LPN/LVN)? a. Communicate the blood glucose level and insulin dose to the circulating nurse in surgery. b. Discuss the reason for the use of insulin therapy during the immediate postoperative period. c. Administer the prescribed lispro (Humalog) insulin before transporting the patient to surgery. d. Plan strategies to minimize the risk for hypoglycemia or hyperglycemia during the postoperative period.

ANS: C LPN/LVN education and scope of practice includes administration of insulin. Communication about patient status with other departments, planning, and patient teaching are skills that require RN education and scope of practice.

After the nurse has finished teaching a patient who has a new prescription for exenatide (Byetta), which patient statement indicates that the teaching has been effective? a. "I may feel hungrier than usual when I take this medicine." b. "I will not need to worry about hypoglycemia with the Byetta." c. "I should take my daily aspirin at least an hour before the Byetta." d. "I will take the pill at the same time I eat breakfast in the morning."

ANS: C Since exenatide slows gastric emptying, oral medications should be taken at least an hour before the exenatide to avoid slowing absorption. Exenatide is injected and increases feelings of satiety. Hypoglycemia can occur with this medication

A patient who was admitted with diabetic ketoacidosis secondary to a urinary tract infection has been weaned off an insulin drip 30 minutes ago. The patient reports feeling lightheaded and sweaty. Which action should the nurse take first? a. Infuse dextrose 50% by slow IV push. b. Administer 1 mg glucagon subcutaneously. c. Obtain a glucose reading using a finger stick. d. Have the patient drink 4 ounces of orange juice.

ANS: C The patient's clinical manifestations are consistent with hypoglycemia and the initial action should be to check the patient's glucose with a finger stick or order a stat blood glucose. If the glucose is low, the patient should ingest a rapid-acting carbohydrate, such as orange juice. Glucagon or dextrose 50% might be given if the patient's symptoms become worse or if the patient is unconscious

After change-of-shift report, which patient will the nurse assess first? a. 19-year-old with type 1 diabetes who was admitted with possible dawn phenomenon b. 35-year-old with type 1 diabetes whose most recent blood glucose reading was 230 mg/dL c. 60-year-old with hyperosmolar hyperglycemic syndrome who has poor skin turgor and dry oral mucosa d. 68-year-old with type 2 diabetes who has severe peripheral neuropathy and complains of burning foot pain

ANS: C The patient's diagnosis of HHS and signs of dehydration indicate that the nurse should rapidly assess for signs of shock and determine whether increased fluid infusion is needed. The other patients also need assessment and intervention but do not have life-threatening complications

Which information is most important for the nurse to report to the health care provider before a patient with type 2 diabetes is prepared for a coronary angiogram? a. The patient's most recent HbA1C was 6.5%. b. The patient's admission blood glucose is 128 mg/dL. c. The patient took the prescribed metformin (Glucophage) today. d. The patient took the prescribed captopril (Capoten) this morning.

ANS: C To avoid lactic acidosis, metformin should be discontinued a day or 2 before the coronary arteriogram and should not be used for 48 hours after IV contrast media are administered. The other patient data will also be reported but do not indicate any need to reschedule the procedure

An 18-year-old with newly diagnosed type 1 diabetes has received diet instruction. The nurse determines a need for additional instruction when the patient says, a. "I may have an occasional alcoholic drink if I include it in my meal plan." b. "I will need a bedtime snack because I take an evening dose of NPH insulin." c. "I may eat whatever I want, as long as I use enough insulin to cover the calories." d. "I will eat meals as scheduled, even if I am not hungry, to prevent hypoglycemia."

ANS: C "I may eat whatever I want, as long as I use enough insulin to cover the calories." Most patients with type 1 diabetes need to plan diet choices very carefully. Patients who are using intensified insulin therapy have considerable flexibility in diet choices but still should restrict dietary intake of items such as fat, protein, and alcohol. The other patient statements are correct and indicate good understanding of the diet instruction.

The nurse and LPN/LVN are caring for a type 2 diabetic patient who is admitted for gallbladder surgery. Which nursing action can the nurse delegate to the LPN/LVN? a. Communicate the blood glucose and insulin dose to the circulating nurse in surgery. b. Discuss the reason for the use of insulin therapy during the immediate postoperative period. c. Administer the prescribed lispro (Humalog) insulin before transferring the patient to surgery. d. Plan strategies to minimize the risk for hypo- or hyperglycemia during the postoperative hospitalization.

ANS: C Administer the prescribed lispro (Humalog) insulin before transferring the patient to surgery. LPN/LVN education and scope of practice includes administration of insulin. Communication about patient status with other departments, planning, and patient teaching are skills that require RN education and scope of practice.

A patient with type 2 diabetes is admitted for an outpatient coronary arteriogram. Which information obtained by the nurse is most important to report to the health care provider before the procedure? a. The patient's admission blood glucose is 128 mg/dL. b. The patient's most recent Hb A1C was 6.5%. c. The patient took the prescribed metformin (Glucophage) today. d. The patient took the prescribed captopril (Capoten) this morning.

ANS: C The patient took the prescribed metformin (Glucophage) today. To avoid lactic acidosis, metformin should be discontinued a day or 2 before the coronary arteriogram and should not be used for 48 hours after IV contrast media are administered. The other patient data also will be reported but do not indicate any need to reschedule the procedure.

A patient screened for diabetes at a clinic has a fasting plasma glucose level of 120mg/dL (6.7 mmol/L). The nurse will plan to teach the patient about a. self-monitoring of blood glucose. b. use of low doses of regular insulin. c. lifestyle changes to lower blood glucose. d. effects of oral hypoglycemic medications.

ANS: C lifestyle changes to lower blood glucose. The patient's impaired fasting glucose indicates prediabetes and the patient should be counseled about lifestyle changes to prevent the development of type 2 diabetes. The patient with prediabetes does not require insulin or the oral hypoglycemics for glucose control and does not need to self-monitor blood glucose.

Which action by a patient indicates that the home health nurse's teaching about glargine and regular insulin has been successful? a. The patient administers the glargine 30 minutes before each meal. b. The patient's family prefills the syringes with the mix of insulins weekly. c. The patient draws up the regular insulin and then the glargine in the same syringe. d. The patient disposes of the open vials of glargine and regular insulin after 4 weeks.

ANS: D Insulin can be stored at room temperature for 4 weeks. Glargine should not be mixed with other insulins or prefilled and stored. Short-acting regular insulin is administered before meals, while glargine is given once daily.

Which finding indicates a need to contact the health care provider before the nurse administers metformin (Glucophage)? a. The patient's blood glucose level is 174 mg/dL. b. The patient has gained 2 lb (0.9 kg) since yesterday. c. The patient is scheduled for a chest x-ray in an hour. d. The patient's blood urea nitrogen (BUN) level is 52 mg/dL.

ANS: D The BUN indicates possible renal failure, and metformin should not be used in patients with renal failure. The other findings are not contraindications to the use of metformin

A patient with type 2 diabetes is scheduled for a follow-up visit in the clinic several months from now. Which test will the nurse schedule to evaluate the effectiveness of treatment for the patient? a. Urine dipstick for glucose b. Oral glucose tolerance test c. Fasting blood glucose level d. Glycosylated hemoglobin level

ANS: D The glycosylated hemoglobin (A1C or HbA1C) test shows the overall control of glucose over 90 to 120 days. A fasting blood level indicates only the glucose level at one time. Urine glucose testing is not an accurate reflection of blood glucose level and does not reflect the glucose over a prolonged time. Oral glucose tolerance testing is done to diagnose diabetes, but is not used for monitoring glucose control once diabetes has been diagnosed

The nurse has been teaching a patient with type 2 diabetes about managing blood glucose levels and taking glipizide (Glucotrol). Which patient statement indicates a need for additional teaching? a. "If I overeat at a meal, I will still take the usual dose of medication." b. "Other medications besides the Glucotrol may affect my blood sugar." c. "When I am ill, I may have to take insulin to control my blood sugar." d. "My diabetes won't cause complications because I don't need insulin."

ANS: D The patient should understand that type 2 diabetes places the patient at risk for many complications and that good glucose control is as important when taking oral agents as when using insulin. The other statements are accurate and indicate good understanding of the use of glipizide

A 28-year-old male patient with type 1 diabetes reports how he manages his exercise and glucose control. Which behavior indicates that the nurse should implement additional teaching? a. The patient always carries hard candies when engaging in exercise. b. The patient goes for a vigorous walk when his glucose is 200 mg/dL. c. The patient has a peanut butter sandwich before going for a bicycle ride. d. The patient increases daily exercise when ketones are present in the urine.

ANS: D When the patient is ketotic, exercise may result in an increase in blood glucose level. Type 1 diabetic patients should be taught to avoid exercise when ketosis is present. The other statements are correct

Which patient statement after the nurse has completed teaching a patient with type 2 diabetes about taking glipizide (Glucotrol) indicates a need for additional teaching? a. "Other medications besides the Glucotrol may affect my blood sugar." b. "If I overeat at a meal, I will still take just the usual dose of medication." c. "When I become ill, I may have to take insulin to control my blood sugar." d. "My diabetes is not as likely to cause complications as if I needed to take insulin."

ANS: D "My diabetes is not as likely to cause complications as if I needed to take insulin." The patient should understand that type 2 diabetes places the patient at risk for many complications and that good glucose control is as important when taking oral agents as when using insulin. The other statements are accurate and indicate good understanding of the use of glipizide.

Amitriptyline (Elavil) is prescribed for a diabetic patient who has burning foot pain at night. Which information should the nurse include when teaching the patient about the new medication? a. Amitriptyline will decrease the depression caused by your foot pain. b. Amitriptyline will correct some of the blood vessel changes that cause pain. c. Amitriptyline will improve sleep and make you less aware of nighttime pain. d. Amitriptyline will help prevent the transmission of pain impulses to the brain.

ANS: D Amitriptyline will help prevent the transmission of pain impulses to the brain. Tricyclic antidepressants decrease the transmission of pain impulses to the spinal cord and brain. Tricyclics also improve sleep quality and are used for depression, but that is not the major purpose for their use in diabetic neuropathy. The blood vessel changes that contribute to neuropathy are not affected by tricyclics.

A patient with newly diagnosed type 2 diabetes mellitus asks the nurse what "type 2" means in relation to diabetes. Which statement by the nurse about type 2 diabetes is correct? a. Insulin is not used to control blood glucose in patients with type 2 diabetes. b. Complications of type 2 diabetes are less serious than those of type 1 diabetes. c. Type 2 diabetes is usually diagnosed when the patient is admitted with a hyperglycemic coma. d. Changes in diet and exercise may be sufficient to control blood glucose levels in type 2 diabetes.

ANS: D Changes in diet and exercise may be sufficient to control blood glucose levels in type 2 diabetes. For some patients, changes in lifestyle are sufficient for blood glucose control. Insulin is frequently used for type 2 diabetes, complications are equally severe as for type 1 diabetes, and type 2 diabetes is usually diagnosed with routine laboratory testing or after a patient develops complications such as frequent yeast infections.

To evaluate the effectiveness of treatment for a patient with type 2 diabetes who is scheduled for a follow-up visit in the clinic, which test will the nurse plan to schedule for the patient? a. Urine dipstick for glucose b. Oral glucose tolerance test c. Fasting blood glucose level d. Glycosylated hemoglobin level

ANS: D Glycosylated hemoglobin level The glycosylated hemoglobin (Hb A1C) test shows the overall control of glucose over 90 to 120 days. A fasting blood level indicates only the glucose level at one time. Urine glucose testing is not an accurate reflection of blood glucose level and does not reflect the glucose over a prolonged time. Oral glucose tolerance testing is done to diagnose diabetes, but is not used for monitoring glucose control once diabetes has been diagnosed.

After the home health nurse has taught a patient and family about how to use glargine and regular insulin safely, which action by the patient indicates that the teaching has been successful? a. The patient administers the glargine 30 to 45 minutes before eating each meal. b. The patient's family fills the syringes weekly and stores them in the refrigerator. c. The patient draws up the regular insulin and then the glargine in the same syringe. d. The patient disposes of the open vials of glargine and regular insulin after 4 weeks.

ANS: D The patient disposes of the open vials of glargine and regular insulin after 4 weeks. Insulin can be stored at room temperature for 4 weeks. Glargine should not be mixed with other insulins or prefilled and stored. Short-acting regular insulin is administered before meals, while glargine is given once daily.

Which information about a patient who receives rosiglitazone (Avandia) is most important for the nurse to report immediately to the health care provider? a. The patient's blood pressure is 154/92. b. The patient has a history of emphysema. c. The patient's noon blood glucose is 86 mg/dL. d. The patient has chest pressure when ambulating.

ANS: D The patient has chest pressure when ambulating. Rosiglitazone can cause myocardial ischemia. The nurse should immediately notify the health care provider and expect orders to discontinue the medication. There is no urgent need to discuss the other data with the health care provider.

Which action by a type 1 diabetic patient indicates that the nurse should implement teaching about exercise and glucose control? a. The patient always carries hard candies when engaging in exercise. b. The patient goes for a vigorous walk when the glucose is 200 mg/dL. c. The patient has a peanut butter sandwich before going for a bicycle ride. d. The patient increases daily exercise when ketones are present in the urine.

ANS: D The patient increases daily exercise when ketones are present in the urine. When the patient is ketotic, exercise may result in an increase in blood glucose level. Type 1 diabetic patients should be taught to avoid exercise when ketosis is present. The other statements are correct.

The nurse teaches the diabetic patient who rides a bicycle to work every day to administer morning insulin into the a. arm. b. thigh. c. buttock. d. abdomen.

ANS: D abdomen. Patients should be taught not to administer insulin into a site that will be exercised because exercise will increase the rate of absorption. The thigh, buttock, and arm are all exercised by riding a bicycle.

How does alcohol affect diabetes How to manage the effects

Alcohol inhibits gluconeogenesis (breakdown of glycogen to glucose) by the liver - This can cause severe hypoglycemia in patients on insulin or oral hypoglycemic medications A patient can reduce risk for alcohol-induced hypoglycemia by eating carbohydrates when drinking alcohol.

diabetic nephropathy treatment

Annual screening for albumin; If albuminuria is present, drugs to delay progression: ACE inhibitors (usually Lisinipril), angiotension II receptor antagonists (usually Losartan); Control of hypertension and tight blood glucose control is imperative

12. The nurse is assessing the motor function of an unconscious male client. The nurse would plan to use which plan to use which of the following to test the client's peripheral response to pain? a. Sternal rub b. Nail bed pressure c. Pressure on the orbital rim d. Squeezing of the sternocleidomastoid muscle

Answer B. Motor testing in the unconscious client can be done only by testing response to painful stimuli. Nail bed pressure tests a basic peripheral response. Cerebral responses to pain are tested using sternal rub, placing upward pressure on the orbital rim, or squeezing the clavicle or sternocleidomastoid muscle.

A male client with a nagging cough makes an appointment to see the physician after reading that this symptom is one of the seven warning signs of cancer. What is another warning sign of cancer? a. Persistent nausea b. Rash c. Indigestion d. Chronic ache or pain

Answer C. Indigestion, or difficulty swallowing, is one of the seven warning signs of cancer. The other six are a change in bowel or bladder habits, a sore that does not heal, unusual bleeding or discharge, a thickening or lump in the breast or elsewhere, an obvious change in a wart or mole, and a nagging cough or hoarseness. Persistent nausea may signal stomach cancer but isn't one of the seven major warning signs. Rash and chronic ache or pain seldom indicate cancer.

Laboratory studies are performed for a child suspected of having iron deficiency anemia. The nurse reviews the laboratory results, knowing that which of the following results would indicate this type of anemia? A.An elevated hemoglobin level B.A decreased reticulocyte count C.An elevated RBC count D.Red blood cells that are microcytic and hypochromic

Answer: D The results of a CBC in children with iron deficiency anemia will show decreased hemoglobin levels and microcytic and hypochromic red blood cells. The red blood cell count is decreased. The reticulocyte count is usually normal or slightly elevated.

Pituitary gland hormones

Anterior - Growth hormone (GH) - Thyroid-stimulating hormone (TSH) or thyrotropin - Adrenocorticotropic hormone (ACTH) - Gonadotropic hormones +Follicle-stimulating hormone (FSH) + Luteinizing hormone (LH) - Melanocyte-stimulating hormone (MSH) - Prolactin Posterior - Oxytocin - Antidiuretic hormone (ADH), or vasopressin

A pt is hospitalized with acute adrenal insufficiency. The nurse determines that the pt is responding favorably to treatment upon finding a. decreasing serum sodium. b. decreasing serum potassium. c. decreasing blood glucose. d. increasing urinary output.

B R: CMs of Addison's disease include hyperkalemia and a decrease in potassium level indicates improvement. Decreasing serum sodium and decreasing blood glucose indicate that treatment has not been effective. Changes in urinary output are not an effective way of monitoring treatment for Addison's disease.

When providing postoperative care for a patient who has had bilateral adrenalectomy, which assessment information obtained by the nurse is most important to communicate to HCP? a. The blood glucose is 156 mg/dl. b. The patient's blood pressure is 102/50. c. The patient has 5/10 incisional pain. d. The lungs have bibasilar crackles.

B R: During immediate postoperative period, marked fluctuation in cortisol levels may occur and the nurse must be alert for signs of acute adrenal insufficiency such as hypotension. nurse should also address elevated glucose, incisional pain, and crackles with appropriate collaborative or nursing actions, but prevention and treatment of acute adrenal insufficiency is the priority after adrenalectomy.

A few hours after returning to the surgical nursing unit, a patient who has undergone a subtotal thyroidectomy develops laryngeal stridor and a cramp in the right hand. The nurse anticipates that intervention will include a. administration of IV morphine. b. administration of IV calcium gluconate. c. endotracheal intubation with mechanical ventilation. d. immediate tracheostomy and manual ventilation.

B R: The pt's CMs are consistent with tetany caused by hypocalcemia resulting from damage to the parathyroid glands during surgery. Tracheostomy may be needed if the calcium does not resolve the stridor. There is no indication that morphine is needed. Endotracheal intubation may be done, but only if calcium is not effective in correcting stridor

During preoperative teaching for a patient scheduled for transsphenoidal hypophysectomy for treatment of a pituitary adenoma, the nurse instructs the patient about the need to a. remain on bed rest for the first 48 hours after the surgery. b. avoid brushing the teeth for at least 10 days after the surgery. c. cough and deep-breathe every 2 hrs postoperatively. d. be positioned flat with sandbags at the head postoperatively.

B R: To avoid disruption of the suture line, the patient should avoid brushing the teeth for 10 days after surgery. It is not necessary to remain on bed rest after this surgery. Coughing is discouraged because it may cause leakage of cerebrospinal fluid (CSF) from the suture line. The head of the bed should be elevated 30 degrees to reduce pressure on the sella turcica and decrease the risk for headaches. (Cognitive Level: Application Text Reference: p. 1293 NProcess: Implementation NCLEX: Physiological Integrity)

When teaching a patient with newly diagnosed hypothyroidism about management of the condition, the nurse should a. delay teaching about the condition until the patient has responded to replacement therapy. b. provide written handouts of all instructions for continued reference as the patient improves. c. have a family member teach the patient about the condition when the patient is more alert. d. arrange for daily home visits by home health nurses to repeat the necessary instructions.

B R: Written instructions will be helpful to the patient because initially the hypothyroid patient may be unable to remember to take medications and other aspects of self-care. Teaching should not be delayed, but family members or friends should be included in teaching to assist the patient. The nurse, not a family member, is responsible for patient teaching. Because thyroid replacement does not begin to improve alertness immediately, it is not appropriate to schedule daily home health visits for teaching.

Your client is being discharged with a prescription for levothyroxine 0.1 mg by. You instruct yout client to take the medication daily in the morning The rationale for this instruction is because the medication: a. Causes diuresis b. Causes adrenergic stimulation c. Causes gluconeogensis d. Has decreased absorption when taken with food

B. Causes adrenergic stimulation

A client has sustained a severe head injury and has had intracerebral bleeding in parietal lobes. The nurse should remain particularly alert for which of the following complications A. Visual impairment B. Difficulties with spatial perception and problem solving C. Hearing difficulties D. Imparied Judgement

B. Difficulties with spatial perception and problem solving

The nurse is caring for a comatose patient who has suffered an epidural bleed. Which intervention should the nurse implement to prevent an increase in intracranial pressure A. Suctioning the airway every hour B. Elevating the head of the bed 15 to 30 degrees C. Turning the patient and changing his position every 2 hours D. Maintaining a well-lit room

B. Elevating the head of the bed 15 to 30 degrees

A client is admitted to the trauma unit with a suspected arterial bleed in his head following an injury. He is experiencing periods of confusion and lucidity. As the nurse assesses his status, she will further assess for a. Increased intracranial pressure. b. Epidural hematoma. c. Subdural hematoma. d. Increased blood pressure.

B. Epidural hematomas usually form quickly within 6 hours after injury, as a result of an arterial bleed. They usually cause periods of confusion and lucidity, and may or may not cause loss of consciousness. Epidural hematomas are fatal if left untreated; in fact, subdural hematomas, not epidural hematomas, have the highest mortality of all head injuries (60 to 90 percent).

A diagnosis of a ruptured cerebral aneurysm has been made in a patient with manifestations of a stroke. The nurse anticipates that treatment options that would evaluated for the patient a. hyperventilation therapy b. surgical clipping of the aneurysm c. administration of hyperosmotic agents d. administration of thrombolytic therapy

B: Surgical clipping of they aneurysm- Surgical management with clipping of an aneurysm to decrease re bleeding and vasospasm is an option for a stroke cause by rupture of a cerebral aneurysm. Placement of coils into the lumens of the aneurysm by intercentional radiologists is increasing in popularity. Hyperventilation therapy would increase vasodilation and the potential for hemorrhage. Thrombolytic therapy would be absolutely contraindicated, and if a vessel is patent, osmotic diuretics may leak into tissue, pulling fluid out of the vessel and increasing edema.

qualifications for obesity surgery

BMI 40 kg/m2 or 35 with one or more severe obesity related conditions: DM, HTN, sleep apnea, heart failure

Drug therapy: Oral and Non-insulin injectable agents Biguanides (e.g. metformin) Meglitinides - same as Biguanides Sulfonylurea (e.g. glipizide [Glucotrol]) Dipeptidyl Peptidase-4 (DPP-4) Inhibitors Glucagon-Like Peptide Receptor Agonists

Biguanides (e.g. metformin) - Reduce glucose production by the liver - Enhances insulin sensitivity and glucose transport Meglitinides - same as Biguanides - e.g. repaglinide (Prandin), nateglinide (Starlix) Sulfonylurea (e.g. glipizide [Glucotrol]) - Increase insulin production by pancreas Dipeptidyl Peptidase-4 (DPP-4) Inhibitors - Block DPP-4 = More incretin = More insulin - Incretin = hormone that increase insulin synthesis - Incretin is inactivated by DPP-4 Glucagon-Like Peptide Receptor Agonists - e.g. exenatide (Byetta), liraglutide (Victoza) - stimulate GLP-1 (one of incretin hormones) - Increase insulin synthesis

A patient with Cushing syndrome returns to the surgical unit following an adrenalectomy. During the initial postoperative period, the nurse gives the highest priority to a. monitoring for infection. b. protecting the patient's skin. c. maintaining fluid and electrolyte status. d. preventing severe emotional disturbances.

C R: After adrenalectomy, the pt is at risk for circulatory instability caused by fluctuating hormone levels, and the focus of care is to assess and maintain fluid and electrolyte status through the use of IV fluids and corticosteroids. other goals are also important for pt but arent as immediately life-threatening as circulatory collapse

A patient with an antidiuretic hormone (ADH)-secreting small-cell cancer of the lung is treated with demeclocycline (Declomycin) to control the symptoms of syndrome of inappropriate secretion of antidiuretic hormone (SIADH). The nurse determines that the demeclocycline is effective upon finding that the a. patient's daily weight is stable. b. urine specific gravity is increased. c. patient's urinary output is increased. d. peripheral edema is decreased

C R: Demeclocycline blocks the action of ADH on the renal tubules and increases urine output. A stable body weight and an increase in urine specific gravity indicate that the SIADH is not corrected. Peripheral edema does not occur with SIADH; a sudden weight gain without edema is a common clinical manifestation of this disorder.

A pt with a possible pheochromocytoma is admitted to the hospital for evaluation and diagnostic testing. During an attack, the nurse will monitor for hypertension and a. hypoglycemia. b. bradycardia. c. headache. d. flushing.

C R: The classic CMs of pheochromocytoma are hypertension, tachycardia, severe headache, diaphoresis, and abdominal or chest pain. Elevated blood glucose may also occur due to sympathetic nervous system stimulation. Bradycardia and flushing would not be expected. (Cognitive Level: Application Text Reference: p. 1320 NProcess: Assessment NCLEX: Physiological Integrity)

While assessing a patient who has just arrived in the postanesthesia recovery unit (PACU) after a thyroidectomy, the nurse obtains these data. Which information is most important to communicate to the surgeon? a. The pt is complaining of 7/10 incisional pain. b. The pt's cardiac monitor shows a HR of 112. c. The patient has increasing swelling of the neck. d. The pat's voice is weak and hoarse sounding.

C R: The neck swelling may lead to respiratory difficulty, and rapid intervention is needed to prevent airway obstruction. The incisional pain should be treated but is not unusual after surgery. A heart rate of 112 is not unusual in a pt who has been hyperthyroid and has just arrived in the PACU from surgery. Vocal hoarseness is expected after surgery due to edema.

A pt with Cushing syndrome is admitted to the hospital to have laparoscopic adrenalectomy. During the admission assessment, the patient tells the nurse, "The worst thing about this disease is how terrible I look. I feel awful about it." best response by the nurse is a. "Let me show you how to dress so that the changes are not so noticeable." b. "I do not think you look bad. Your appearance is just altered by your disease." c. "Most of the physical and mental changes caused by the disease will gradually improve after surgery." d. "You really should not worry about how you look in the hospital. We see many worse things."

C Rationale: The most reassuring communication to the patient is that the physical and emotional changes caused by the Cushing syndrome will resolve after hormone levels return to normal postoperatively. The response beginning "Let me show you how to dress" indicates that the changes are permanent and that the patient's appearance needs disguising. The response beginning, "I do not think you look bad" does not acknowledge the patient's feelings and also fails to communicate that the changes will be resolved after surgery. And the response beginning "You really should not worry about how you look in the hospital" implies that the pt's appearance is not good.

A client is hospitalized for the insertion of an internal cervical radiation implant. The nurse initiates what most appropriate activity order for this client? A. Out of bed to chair only B. Ambulate to the bathroom only C. BedRest D. Out of bed ad lib

C. BedRest

when concerned about increased intracranial pressure the nurse recalls that the client presenting with increasing intracranial pressure and cushings triad presents with A increased pulse pressure, decreased mean arterial pressure, and tachycardia B. Decreased pulse pressure, increased mean arterial pressure and tachycardia C. Widening pulse pressure, irregular respirations and bradycardia D. Decreased pulse pressure, decreased systolic blood pressure and increased cerebral blood flow.

C. Widening pulse pressure, irregular respirations and bradycardia

Hypoparathyroidism

Characterized by hypocalcemia from low PTH - Calcium go into bone Etiology/Pathophysiology - Most common cause is accidental removal during neck surgery - Tumors, autoimmune diseases (antibody present) Clinical Manifestations - Related to hypocalcemia - Tetany (sudden decrease in calcium) (muscle spasms, laryngospasms) Collaborative Treatment - Emergency treatment: IV calcium slowly - ECG monitoring - Correct Hypomagnesemia - Vitamin D supplement Nutrition - Eat dark green vegetables, soybeans and tofu - Avoid food with oxalic acid (spinach, rhubarb) because they inhibit absorption of calcium

Hormones definition Common characteristics

Chemical substances produced in the body that control and regulate the activity of certain target cells or organs Common characteristics - Secretion in small amounts at predictable rates - Regulation by feedback system - Binding to specific target cell receptors - "lock and key" type of mechanism - Lipid-soluble hormones needs plasma protein transport - Water-soluble hormones does not need plasma protein

Diabetes Mellitus Pathophysiology

Chronic multi-system disease related to abnormal insulin production, impaired insulin utilization, or both. Pathophysiology of Type 1 - a.k.a juvenile-onset or insulin-dependent - Autoimmune destruction of pancreatic B cells that produce insulin. - *No production of insulin* - Needs external insulin to sustain life Pathophysiology of Type 2 1) Defective B cells: Not produce enough insulin 2) Defective liver: Excess glucose production 3) Defect insulin receptors: Insulin resistance 4) Adipose tissue: Altered adipokines production - Play a role in glucose and fat metabolism

What sign should you assess for decreased calcium?

Chvosteks (face spasm) or trousseaus (arm spasm).

Thyroid Cancer

Clinical manifestations - Painless, palpable nodule or nodules in an enlarged thyroid gland Thyroid scan - Tumors that take up radioactive iodine are "hot" and are nearly always benign - Tumors that don't take up radioactive iodine are "cold" and has higher risk of being malignant Treatment - Surgical removal of tumor - RAI to destroy remaining cancer cells after surgery

When assessing motor function of a patient admitted with a stroke, the nurse notes mild weakness of the arm demonstrated by downward drifting of the arm. The nurse would most accurately document this finding as A) a. Athetosis. B) b. Hypotonia. C) c. Hemiparesis. D) d. Pronator drift.

D Downward drifting of the arm or pronation of the palm is identified as a pronator drift. Hemiparesis is weakness of one side of the body; hypotonia defines a flaccid muscle tone; and athetosis is a slow, writhing, involuntary movement of the extremities.

A pt is admitted with possible SIADH. Which information obtained by nurse is most important to communicate rapidly to health care provider? a. The patient complains of a severe headache. b. The patient complains of severe thirst. c. The patient has a urine specific gravity of 1.025. d. The pt has a serum sodium level of 119 mEq/L.

D R: A serum sodium of less than 120 mEq/L increases risk for complications such as seizures and needs rapid correction. The other data are not unusual for a pt with SIADH and do not indicate the need for rapid action. (Cognitive Level: Application Text Reference: p. 1295 NProcess: Assessment NCLEX: Physiological Integrity)

Which information obtained when caring for a pt who has just been admitted for evaluation of DI will be of greatest concern to the nurse? a. The patient has a urine output of 800 ml/hr. b. The patient's urine specific gravity is 1.003. c. The patient had a recent head injury. d. The patient is confused and lethargic.

D R: Pts with diabetes insipidus compensate for fluid losses by drinking copious amounts of fluids, but a patient who is lethargic will be unable to drink enough fluids and will become hypovolemic. A high urine output, low urine specific gravity, and history of a recent head injury are consistent with DI, but they do not require immediate nursing action to avoid life-threatening complications.

A patient with primary hyperparathyroidism has a serum calcium level of 14 mg/dl (3.5 mmol/L), phosphorus of 1.7 mg/dl (0.55 mmol/L), serum creatinine of 2.2 mg/dl (194 mmol/L), and a high urine calcium. While the patient awaits surgery, the nurse should a. institute seizure precautions such as padded siderails. b. assist the patient to perform range-of-motion exercises QID. c. monitor the patient for positive Chvostek's or Trousseau's sign. d. encourage the pt to drink 4000 ml of fluid daily.

D R: The pt with hypercalcemia is at risk for kidney stones, which may be prevented by a high fluid intake. Seizure precautions and monitoring for Chvostek's or Trousseau's sign are appropriate for hypocalcemic patients. The pt should engage in weight-bearing exercise rather than range-of-motion because weight-bearing decreases calcium loss from bone.

A patient with sudden-onset right-sided weakness has a CT scan and is diagnosed with an intracerebral hemorrhage. Which information about the patient is most important to communicate to the health care provider? a. The patient's speech is difficult to understand. b. The patient's blood pressure is 144/90 mm Hg. c. The patient takes a diuretic because of a history of hypertension. d. The patient has atrial fibrillation and takes warfarin (Coumadin).

D The use of warfarin will have contributed to the intracerebral bleeding and remains a risk factor for further bleeding. Administration of vitamin K is needed to reverse the effects of the warfarin, especially if the patient is to have surgery to correct the bleeding. The history of hypertension is a risk factor for the patient but has no immediate effect on the patient's care. The BP of 144/90 indicates the need for ongoing monitoring but not for any immediate change in therapy. Slurred speech is consistent with a left-sided stroke, and no change in therapy is indicated.

When receiving desmopressin acetate as a nasal spray for treatment of DI, which action would provide optimal control of the patients disease? A: Clear nasal passages after spraying the medication B: Inhale the spray for full drug effect C. If nasal congestion occurs take an over the counter preparation to control mucus D: Take the nasal spray at the same time every day.

D: Take the nasal spray at the same time every day.

dka

DKA - Profound deficiency of insulin. Most likely in DM1. There was not enough insulin, glucose cannot be used for energy so fat stores are broken down as a secondary source of fuel. Ketones is the byproduct and is a problem when it builds up and causes metabolic acidosis. Ketones are excreted in the urine. Protein degrades, nitrogen losses from tissues, etc. If not treated the patient will have loss of sodium, potassium, chloride, magnesium, phosphate, vomiting causes more losses, Hypovolemia and shock and rebal failure can occur. Comatose and death possible. Early symptoms:weakness,lethargy. Tachycardia, orthostatic hypotension. Kussmaul breaths to correct metabolic acidosis (rapid deep). Acetone on the breath -sweet fruity.

Hypopituitarism

Decrease in one or more pituitary hormones Etiology - Tumor, autoimmune, infection, trauma Low GH - Subtle symptoms, truncal obesity, decreased muscle mass and strength, weakness, fatigue Low FSH and LH - Irregular menstrual, less libido, smaller breasts - Testicular atrophy, diminished spermatogenesis, loss of libido, impotence, decrease facial hair/mass Low TSH - Mild form of primary hypothyroidism: - Fatigue, cold intolerance, constipation, weight gain Low ACTH - Deficiency of cortisol: - weakness, fatigue, headache, dry and pale skin, hypoglycemia, lower immune system

Disorders of posterior pituitary gland DI

Diabetes Insipidus (DI) - too little ADH - Increase urine output - Increase plasma osmolality (hypernatremia) - Polydipisa, polyuria - Low urine specific gravity <1.005 Diagnostic studies - Excrete dilute urine > 200mL/hr - Urine specific gravity <1.005 - Water deprivation test Collaborative Management - Maintain fluid and electrolyte balance - Fluid replaced orally or IV (Hypotonic solution) - Central DI: DDAVP (demopressin) = ADH hormone - Nephrogenic DI: Low-sodium, thiazide diuretics (reduce flow to ADH-sensitive distal nephrons), or Indomethacin (NSAID that increase renal responsiveness to ADH)

Diagnostic Studies for Cushing's Syndrome

Elevated plasma cortisol 24-hour urine collection for free cortisol - Normal 80-120 mcg/24hr; Elevated = Cushings Low-dose dexamethasone suppression test - If 24-hour cortisol urine result is borderline

Maintaining a Patent Feeding Tube

Flush the tube with 20 to 30mL of water (or the amount prescribed by the health care provider or dietitian): • At least every 4 hours during a continuous tube feeding • Before and after each intermi ent tube feeding • Before and after drug administration (use warm water) • After checking residual volume • If the tube becomes clogged, use 30mL of water for flushing, applying gentle pressure with a 50-mL piston syringe. • Avoid the use of a carbonated beverage, except for existing clogs when water is not effective. Do not use cranberry juice. • Whenever possible, use liquid medications instead of crushed tablets unless liquid forms cause diarrhea; make sure that the drug is compatible with the feeding solution. • Do not mix drugs with the feeding product before giving. Crush tablets as finely as possible and dissolve in warm water. (Check to see which tablets are safe to crush. For example, do not crush slow-acting [SA] or slow-release [SR] drugs.) • Consider use of automatic-flush feeding pump such as Flexiflo or Kangaroo.

- Gonadotropic hormones +Follicle-stimulating hormone (FSH) + Luteinizing hormone (LH)

Follicle-stimulating hormone (FSH) - (female) Stimulate growth of ovarian follicle - (female) Increase estrogen production - (male) Stimulate testes: increase spermatogenesis Luteinizing hormone (LH) = stimulate sex hormones - (male) Stimulate production of testosterone - (female) Triggers ovulation (14 days) => corpus luteum created, which produces progesterone

factors that may increase blood sugar

GLUCOCORTICOIDS, THIAZIDE DIURETICS, AND ESTROGEN stress, surgery or infection

What are the components of the food pyramid from greatest need to smallest?

Grains, veggies, fruits, milk, and meat&beans

What is characteristic of HHNK?

High blood sugar, extreme dehydration, increased serum osmolarity, diuresis, electrolyte loss, warm flushed skin, poor turgor, sunken eyeballs, and dry mucous membranes.

Place the insulins in order of onset? NPH, Lantus, regular, humalog, lente

Humalog, regular, lente, NPH, and lantus

GLITAZONES

INCREASE CELLULAR UTILIZATION OF GLUCOSE reduces resistance to insulin

Immediate postop considerations for bariatric surgeries

Immediate postop -airway, pain, HOB 35-40 degrees to allow full expansion lungs and decrease abd pressure. Body stores anesthetics in adipose tissue. Pt may become more sedated after surgery as adipose cells let go of anesthesia into bloodstream. Turn and ambulate early. Respiratory complications due to shallow and rapid breathing. Risk for DVT. Frequently observe abd wound and protect againt straining. Pain mangement. IF THE PATIENT VOMITS WITH NGT IN NOTIFY THE SURGEON_IT MAY NEED REPOSITION. Clear liquid diet when fully awake. Home on high protein liquid meals. Vomiting common.

increased risk of breast cancer

Increased age is the primary risk factor for developing breast cancer in both women and men. Several other factors are known to increase the risk of developing breast cancer; many are not modifiable, such as family history, early menarche, and late menopause. Modifiable risk factors include, but are not limited to, postmenopausal obesity, use of postmenopausal hormone replacement therapy (HRT), alcohol consumption, nulliparity, and lack of breast- feeding (ACS,

Hyperparathyroidism

Increased secretion of parathyroid hormone (PTH) - Take calcium out of bones - Excessive hypercalcemia and hypophosphatemia Pathophysiology - Primary: Due to increased secretion of PTH (due to tumor, radiation, life-long lithium therapy) - Secondary: hypocalcemia compensatory response - Tertiary: No negative feedback from calcium level Clinical manifestations - Osteoporosis, fractures, kidney stones - R/T hypercalcemia (muscle weakness, loss of appetite, constipation, fatigue, short attention span)

Thyroiditis

Inflammation of thyroid - Frequent cause of goiter Hashimoto's thyroiditis - Chronic Autoimmune - Destruction of thyroid tissues by antibodies - Hypothyroidism - T3 and T4 are low, TSH is high, antibodies present Silent, painless thyroiditis (May be early Hashimoto's) - Can occur in 6 month postpartum women - T3/T4 initially elevated, but depressed overtime - Suppression of radioactive iodine uptake (RAIU) S/S - Pain localized in thyroid - Pain may radiate to throat, ears, or jaw. - Systemic s/s: fever, chills, sweats, fatigue Treatment - May go away in weeks or months w/o treatment - Antibiotics, or surgical drainage - Symptom management

What is metabolic syndrome?

Insulin resistance syndrome. Excess insulin is needed for glucose to respond causing fat stores and prevents fat breakdown.

What is acromegaly? What causes acromegaly? When does it occur?

It is large hands, ear, feet, etc. Hyperpituitary gland causes acromegaly. It occurs after 18 years of age bc bone thickness increases and cartilage is overproduced. Before 18 years of age=overproduction of growth hormone=giant.

What are the four types of intermediate acting insulins?

Lente, NPH, Novolin L, and Humulin N (onset 1-5 hrs, peak 6-15 hrs, duration 14-24 hrs, adverse reaction occurs early evening-weakness/fatigue).

bse

Lie on your back and place your right arm behind your head. Lying down spreads the breast tissue evenly over the chest wall, making it easier to feel all the breast tissue. 2. Use the finger pads of the three middle fingers on your left hand to feel for lumps in the right breast. Use overlapping dime-size circular motions of the finger pads to feel the breast tissue. 3. Use three different levels of pressure to feel all the breast tissue. Light pressure is needed to feel the tissue closest to the skin; medium pressure to feel a li le deeper; and firm pressure to feel the tissue closest to the chest and ribs. It is normal to feel a firm ridge in the lower curve of each breast. 4. Move around the breast in an up-and-down pa ern, starting at an imaginary line drawn straight down your side from the underarm and moving across the breast to the middle of the chest bone (sternum or breastbone). Be sure to check the entire breast area, going down until you feel only ribs and up to the neck. 5. Repeat the examination on your left breast, pu ing your left arm behind your head and using the finger pads of your right hand to do the examination. 6. While standing in front of a mirror with your hands pressing firmly down on your hips, look at your breasts for any changes of size, shape, contour, or dimpling and look at your nipples and breast skin for redness or scaling. (The pressing down-on-the-hips position contracts the chest wall muscles and enhances any breast changes.) 7. Examine each underarm while si ing up or standing and with your arm only slightly raised so you can easily feel in this area. Raising your arm straight up tightens the tissue in this area and makes it harder to examine.

Hyperosmolar Hyperglycemic Nonketotic Syndrome (HHNS)

Life-threatening syndrome that can occur in patients with diabetes who is able to produce enough insulin to prevent DKA but not enough to prevent severe hyperglycemia, osmotic diuresis, and extracellular fluid depletion - High blood sugar, but non-acidic (No DKA) Common causes - UTI, pneumonia, sepsis, acute illnesses - Often related to impaired thirst mechanism or functional inability to replace fluids.

Insulin therapy complication

Lipodystrophy - atrophy of subcutaneous tissue on frequently used sites Somogyi Effect and Dawn phenomenon - Hyperglycemia in the morning due to hypoglycemia levels at night or early mornings that stimulates the counter-regulatory hormones which increases production of glucose

If there is a psych patient complaining of bad nightmares what could be the cause?

Low blood sugar

oral meds that decrease liver glucose, and decrease intestinal absorption of glucose

METFORMIN

What are the S&S of cushings syndrome? select all that apply

Moon face, hyperglycemia, purple striae, buffalo hump, GI distress, Na and fluid retention, everything increased except K (can have salt substitutes), and osteoporosis.

Basal-bolus regimen

Most closely mimics endogenous insulin production Rapid- or short-acting (bolus) insulin before meals Intermediate- or long-acting (basal) background insulin once or twice a day

Diabetic dermopathy

Most common diabetic skin lesion, characterized by reddish brown, round or oval patches. - Initially scaly, then they flatten out and become indented.

Prolactinomas Signs/Symptoms

Most common pituitary adenomas Women - Galactorrhea, anovulation, infertility, oligomenorrhea, amenorrhea, decreased libido, hirsutisms Men - Impotence and decrease sperm and libido Both - Compression of optic chiasm cause visual problem and signs of ICP (headache, nausea, vomiting)

Can Cushing's patients have steroids?

NO because their blood sugar is already elevated.

Can Addison's pts have salt substitutes?

NO because they are straight potassium.

treatment for malnutrition

Order daily calorie count and diary. Promote high calorie, high protein foods (gravy, milkshakes, veggies with butter or fried, whipped cream). Supplements between meals. Appetite stimulant like Megace or marinol. Parenteral if pt cannot tolerate po.

Pineal Gland hormones Thyroid gland Parathyroid Pancreas Adrenal Cortex Adrenal medulla Ovary Testes

Pineal = Melatonin: Circardian rhythm Thyroid gland hormones - T3/T4: Metabolism - Calcitonin: Lowers blood Ca Parathyroid = PTH: Raise blood Ca Pancreas - Insulin: Lowers blood sugar - Glucagon: Raises blood sugar Adrenal Cortex - Glucocorticoids: Anti-inflammatory Adrenal medulla = Epinephrine: Fight or flight Ovary = Estrogen: Female sex characteristics Testes = Testosterone: Male sex characteristics

Diabetic retinopathy

Process of microvascular damage to the retina as a result of chronic hyperglycemia, nephropathy, and HTN in patients with diabetes. - Most common cause of adult blindness Pathophysiology - Retinal capillaries become occluded, causing microaneurysms and fluid or blood leaks out - Patient sees black or red spots or lines - Retinal detachment can occur D.M. create greater risk for glaucoma and cataract as well. Collaborative care - Person with diabetes should have yearly dilated eye exam - Prevent with glycemic and HTN control

Adrenal androgens (sex steroids)

Produced by Adrenal Cortex Converts to sex hormones - Testosterone in men - Estrogen in women (Main source post-menopause)

Aldosterone

Produced by Adrenal Cortex Increase sodium retention = water retention Increase excretion of potassium and hydrogen ions

Calcitonin

Produced by Thyroid gland (stimulated by high calcium in blood) *Calciton-in (calcium go into bone)* Helps calcium go blood to bone - Decreases calcium blood level Increases renal excretion of Ca - Decrease calcium blood level Increases renal excretion of phosphorus

What are characteristics of an adrenal crisis?

Profound fatigue, dehydration, vascular collapse, renal shutdown, decreased serum Na, increased serum K.

Non-drug therapy for Hyperthyrodisim

Radioactive Iodine Therapy - Effects not seen for up to 3 months - Antithyroid drug therapy until effects are seen - Limit radiation exposure to others Surgical - Thyroidectomy - Indicated for large goiter causing tracheal compression, or been unresponsive to drug therapy, or thyroid cancer - Talking is difficult for a short time after surgery - IV calcium salt if tetany (twitching, paresthesia) - Assess q2h for 24 hours for hemorrhage or tracheal compression (irregular breathing, neck swelling, frequent swallowing, sensation of fullness, choking, blood on dressings) Nutritional - High-calorie diet (4000-5000 cal/day) - Avoid highly seasoned or high-fiber foods (It can stimulate an already hyperactive GI tract) - Avoid caffeine (Already restless and hyper)

Mealtime Insulin (Bolus)

Rapid-acting (bolus) Lispro, aspart, glulisine Onset of action 15 minutes Injected within 15 minutes of mealtime Short-acting (bolus) Regular with onset of action 30 to 60 minutes Injected 30 to 45 minutes before meal Onset of action 30 to 60 minutes

What are the S&S of hypthyroidism (AKA mixedema)?

SLOW AND LOW, LETHARGY, CONSTIPATION, MENSTRUAL DISTURBANCES, DULL-BLANK EXPRESSION, INTOLERANCE TO COLD, RECEDING HAIRLINE, ANOREXIA, BRITTLE HAIR AND NAILS.

Corticosteroids (e.g. cortisol, hydrocortisone)

Secreted by adrenal cortex - Increased by stress, burns, infection, fever, anxiety, hypoglycemia 1) Increase glucose to be used during stress - Inhibit insulin - Glycogenesis in liver - Inhibit protein synthesis (use its glucose for stress) 2) Move glycerol and fats into abdomen 3) Inhibits inflammatory/immune response

Catecholamines (epinephrine and norepinephrine)

Secreted by adrenal medulla Increases in response to stress - Increase effects of sympathetic nervous system

signs a a hernia is strangulated,

Signs of strangulation are abdominal distention, nausea, vomiting, pain, fever, and tachycardia.

simple carb

Simple carbohydrates are found naturally in foods such as fruits, milk, and milk products. They are also found in processed and refined sugars such as candy, table sugar, syrups, and soft drinks.Feb 22, 2018

Regulation of hormonal secretions

Simple feedback, negative feedback - Increased blood levels of a hormone causing inhibition of same hormones or stimulating release of a counter hormone. Positive feedback - Rising hormone level causes another gland to release a hormone that then stimulates further release of the first hormone. - Increase estradiol > ^FSH > ^estradiol Nervous system - SNS, PSNS and catecholamines Rhythms (e.g. circadian rhythm) - hormone levels fluctuate predictably in cycles

What changes would you see in someone who has developed Type II diabetes?

Slow onset, weight gain, and eye problems.

Collaborative management of hyperaldosteronism

Surgical removal of adenoma (adrenalectomy) Before surgery, normalize potassium levels and BP - Potassium-sparing diuretics - Antihypertensive medications - Oral potassium supplements - Sodium restrictions After surgery - Monitor BP and potassium levels

Collaborative Management of disorders of adrenal medulla

Surgical removal of tumor Control BP and prevent HTN crisis - Alpha and Beta-adrenergic receptor blockers - phenoxybenzamine (Dibenzyline) is given 7-10 days preoperatively to reduce BP and alleviate other symptoms of catecholamine excess - Beta blockers to decrease tachycardia and other dysrhythmias

A Client undergoes a biopsy of a suspicious lesion. The biopsy report classifies the lesion according to the TNM staging system as follows: TIS, NO, MO. What does this classification mean? A Can't assess tumor or regional lymph nodes and no evidence of metastasis B No evidence of primary tumor, no abnormal regional lymph nodes, and no evidence of distant metastasis C Carcinoma in situ, no demonstrable metastasis of the regional lymph nodes, and ascending degrees of distant metastasis D Carcinoma in situ, no abnormal regional lymph nodes, and no evidence of distant metastasis

TIS, N0, M0 denotes carcinoma in situ, no abnormal regional lymph nodes, and no evidence of distant metastasis. No evidence of primary tumor, no abnormal regional lymph nodes, and no evidence of distant metastasis is classified as T0, N0, M0. If the tumor and regional lymph nodes can't be assessed and no evidence of metastasis exists, the lesion is classified as TX, NX, M0. A progressive increase in tumor size, no demonstrable metastasis of the regional lymph nodes, and ascending degrees of distant metastasis is classified as T1, T2, T3, or T4; N0; and M1, M2, or M3.

signs and symptoms of dumping syndrome

Tachycardia, nausea, diarrhea, and abdominal cramping are common symptoms of dumping syndrome.

Collaborative Care of hypothyroidism

Thyroid hormone replacement (levothyroxine) - Low initial dose to avoid increase in HR/BP - Increases myocardial oxygen demand (watch out for angina and cardiac dysrhythmias) - It make takes up to 8 weeks for full effect - Lifelong therapy is required Teaching - Warm environment (Intolerance to cold) - Use soap sparingly and lotion to skin (weak skin) - Minimize constipation - Lifelong thyroid hormone replacement - *Take thyroid hormone in morning before food*

anterior pituitary

Thyroid-stimulating hormone (TSH), also known as thyrotropin Adrenocorticotropic hormone (ACTH, corticotropin) Luteinizing hormone (LH), also known as Leydig cell-stimulating hormone (LCSH) Follicle-stimulating hormone (FSH) Prolactin (PRL) Growth hormone (GH) Melanocyte-stimulating hormone (MSH)

checking ph to determine location of tube

To perform this procedure, aspirate a sample of the GI content, observe its color, and test its pH. When aspirating fluid, wait at least 1 hour after drug administration and then flush the tube with 20mL of air to clear it. Collect the aspirate and test it with pH paper. The pH of gastric fluid ranges from 0 to 4.0. If the tube has moved down into the intestines, the pH will be between 7.0 and 8.0. If the tube is in the lungs, the pH will be greater than 6.0. The pH may also be as high as 6.0 if the patient takes certain drugs, such as H2 blockers (e.g., ranitidine [Zantac] and famotidine [Pepcid]). Because these drugs affect pH, bilirubin testing or capnometry may be a more reliable and valid method for predicting tube location.

What should you have available post thyroidectomy?

Tracheostomy set in case an emergency airway is needed. (tight sutures so possible compromise)

What are the two types of long acting insulins?

Ultralente and Humulin U-ultralente (onset 2-4 hrs, peak 8-24, duration 24-36, adverse reaction occurs early morning-HA/confusion).

Which medication order should the nurse question in the client diagnosed with untreated hypothyroidism? 3. Sedatives

Untreated hypothyroidism is characterized by an increased susceptibility to the effects of most hypnotic and sedative agents; therefore, the nurse should question this medication.

What will need life long replacement following transphenoidal hypophysectomy?

Vasopressin (suppresses pee), Cortisone, Thyroid, and Sex hormones

HHS treatment

Very similar to DKA; Fluids, more volume than DKA, delivered more slowly K+ may be needed check for cardiac issues deadly

cause of diarrhea

Viral most common -48 hours and mild usually but can cause death. (rotavirus and norovirus) highly contagious ( usually not admitted to hospital) Bacterial- (E.coli, shigella, salmonella- contaminated food, staphylococcus, campylobacter, C.diff) Parasitic- (Giardia-contaminated water, Entamoeba, Cryptosporidium)

Can Cushing's patients have salt substitutes?

Yes bc they are low in Potassium

• Pancreas transplantation can be used _

_as a treatment option for patients with type 1 diabetes mellitus. if renal failure not present: hx of acute, severe metabolic complications incapacitating clinical or emotional issues consistent failure of insulin based managment

Hyperosmolar Hyperglycemic Syndrome (HHS)

_is a life-threatening syndrome that can occur in the patient with diabetes who is able to produce enough insulin to prevent DKA but not enough to prevent severe hyperglycemia, osmotic diuresis, and extracellular fluid depletion.

Drug therapy: Oral and Non-insulin injectable agents a-glucosidase inhibitors (e.g. Acarbose [Precose]) Sodium-Glucose Co-transporter 2 (SGLT2) Inhibitors Thiazolidineodiones (e.g. pioglitazone [Actos]) Dopamine Receptor Agonist Amylin Analog (e.g. pramlintide [Symlin])

a-glucosidase inhibitors (e.g. Acarbose [Precose]) - "starch blockers", slow down absorption of carbs Sodium-Glucose Co-transporter 2 (SGLT2) Inhibitors - (e.g. canagliflozin [Invokana]) - Blocks reabsorption of glucose by kidneys - Increase glucose excretion Thiazolidineodiones (e.g. pioglitazone [Actos]) - "insulin sensitizers"; improve insulin sensitivity - rarely used because of cardiovascular s/e Dopamine Receptor Agonist - (e.g. bromocriptine [Cycloset]) - Help improve glycemic control, Unknown MOA Amylin Analog (e.g. pramlintide [Symlin]) - Amylin = hormone secreted by pancreas that reduces glucagon secretion, slow gastric emptying, and increases satiety. - Can only be injected subcut into thigh or abdomen

A patient is admitted to the hospital with a diagnosis of Cushing syndrome. On physical assessment of the patient, the nurse would expect to find a. HTN, peripheral edema, and petechiae

a. HTN, peripheral edema, and petechiae rationale- The effects of glucocorticoid excess include weight gain from accumulation and redistribution of adipose tissue, sodium and water retention, glucose intolerance, protein wasting, loss of bone structure, loss of collagen, and capillary fragility.

a patient on antithyroid medications should be advised to avoid which foods? a. Soy products and seafood b. Bananas and oranges c. Dairy products d. Processed meats and cheeses

a. Soy products and seafood

Preoperative instructions for the patient scheduled for a subtotal thyroidectomy includes teaching the patient a. how to support the head with the hands when moving

a. how to support the head with the hands when moving

diabetic nephropathy

accumulation of damage to the glomerulus capillaries due to the chronic high blood sugars of diabetes mellitus

complications of diarrhea

anemia from iron and folate deficiency, increased hematocrit, BUN and creatinine may show fluid deficit. check skin in perineal area

diabetes insipidus

antidiuretic hormone is not secreted adequately, or the kidney is resistant to its effect

A 22 year old male is admitted with a traumatic brain injury due to an ATV crash he suffered while driving intoxicated. The pt also develops complication of increased intracranial pressure and syndrome of inappropriate antidiuretic hormone. Fluid management for clients diagnosed with SIADH would include a. Rapid intravenous fluid infusion b. Fluid restriction c. Increased oral fluid intake d. Admin of dextrose intravenous fluid

b. Fluid restriction

When developing a plan of care for a pt with SIADH, which interventions will the nurse include? b. Offer patient hard candies to suck on.

b. Offer patient hard candies to suck on. R: Sucking on hard candies decreases thirst for patient on a fluid restriction.

A client has undergone transsphenoidal surgery. Which assessments would help identify the complication of DI? a. polyuria, polydipsia, polyphagia b. Urine output, tented skin turgor, urine specific gravity c. nervousness, weakness, warm, sweaty flushed skin d. constipation, cold intolerance, sluggishness

b. Urine output, tented skin turgor, urine specific gravity

A1C

blood test that measures glycosylated hemoglobin (HbA1c) to assess glucose control can measure from 3 months ahead

A pt with Cushing syndrome is admitted to the hospital to have laparoscopic adrenalectomy. During the admission assessment, the patient tells the nurse, "The worst thing about this disease is how terrible I look. I feel awful about it." best response by the nurse is c. "Most of the physical and mental changes caused by the disease will gradually improve after surgery."

c. "Most of the physical and mental changes caused by the disease will gradually improve after surgery." Rationale: The most reassuring communication to the patient is that the physical and emotional changes caused by the Cushing syndrome will resolve after hormone levels return to normal postoperatively.

A female client with the beta-thalassaemia trait plans to marry a man of Italian ancestry who also has the trait. Which client statement indicates that she understands the teaching provided by the nurse? a. Thalassemia is treated with iron supplements b. I need to learn how to give myself b12 injections c. I'll see a genetic counselor before starting a family d. if my fiance was of Middle Eastern decent, I wouldn't worry about having children

c. I'll see a genetic counselor before starting a family Two people with the beta-thalassaemia trait have a 25% chance of having a child with thalassaemia major, a potentially life-threatening disease.

A patient with Addison's disease comes to the emergency department with complaints of N/V/D, and fever. The nurse would expect collaborative care to include c. IV administration of hydrocortisone

c. IV administration of hydrocortisone rationale- vomiting and diarrhea are early indicators of addisonian crisis and fever indicates an infection, which s causing additional stress for the patient. treatment of a crisis requires immediate glucocorticoid replacement,

Which of the following would the nurse include as a non-pharmacological nursing intervention for chronic pain related to metastatic cancer? a. Referring the client to a therapist for hypnosis b. Administering pain meds as prescribed c. Imagery and reduction of external stimuli D. Applying a TENS unit

c. Imagery and reduction of external stimuli

A nurse is caring for a client who is scheduled for a radioactive iodine therapy (RAI) Which circumstances would warrant the immediate notification of the physician? a. Thyroid-stimulating hormone (TSH) level is decreased and t4 is elevated b. Taking Propranolol daily c. Pregnancy test comes back positive d. states an allergy to latex

c. Pregnancy test comes back positive

While assessing a patient who has just arrived in the postanesthesia recovery unit (PACU) after a thyroidectomy, the nurse obtains these data. Which information is most important to communicate to the surgeon? c. The patient has increasing swelling of the neck.

c. The patient has increasing swelling in the neck

To prevent complications in the patient with Cushing syndrome, the nurse monitors the patient for c. cardiac arrhythmias

c. cardiac arrhythmias rationale- electrolyte changes that occur in Cushing syndrome include sodium retention and potassium excretion by the kidney, resulting in hypokalemia

A 72-year-old patient is diagnosed with hypothyroidism, and levothyroxine (Synthroid) is prescribed. During initiation of thyroid replacement for the patient, it is most important for the nurse to assess c. cardiac function.

c. cardiac function. R: In older patients, initiation of levothyroxine therapy can increase myocardial oxygen demand and cause angina or dysrhythmias.

A patient is scheduled for bilateral adrenalectomy. During the postoperative period, the nurse would expect administration of corticosteroids to be c. increased to promote an adequate response to the stress of surgery

c. increased to promote an adequate response to the stress of surgery rationale- although the patient with Cushing syndrome has excess corticosteroids, removal of the glands and the stress of surgery require that high doses of cortisone

symptoms of autonomic neuropathy

changes in blood pressure and heart rate constipation bladder and sexual dysfunction

illness and diabetes

check BG q4 acutely ill pt with BG > 240 should check urine for ketone q 3-4

autonomic neuropathy treamet

cholinergic agonist drug catherization

pituatary

controls water reabsorption in kidney (ADH) sexual development and function

If fluid intake is limited in a client with diabetes insipidus , which of the following complications will he be at risk for developing? a. Hypertension and bradycardia b. Glucosuria and weight gain c. Peripheral edema and hyperglycemia d. Severe dehydration and hypernatremia

d. Severe dehydration and hypernatremia

When teaching a patient newly diagnosed with Graves' disease about the disorder, the nurse explains that d. antithyroid medications may take several weeks to have an effect.

d. antithyroid medications may take several weeks to have an effect. R: Improvement usually begins in 1-2 wks w good results at 4-6 weeks.

Causes of primary hypothyroidism in adults include d. autoimmune-induced atrophy of the gland

d. autoimmune-induced atrophy of the gland rationale- both Graves disease and Hasimotos thyroiditis are autoimmune disorders that eventually destroy the thyroid gland, leading to primary hypothyroidism.

Physical changes of hypothyroidism that must be monitored when replacement therapy is started include d. decreased cardiac contractility and coronary atherosclerosis

d. decreased cardiac contractility and coronary atherosclerosis rationale- hypothyroidism affects the heart in many ways, causing cardiomyopathy, coronary atherosclerosis, bradycardia, pericardial effusions, and weakened cardiac contractility.

A patient with primary hyperparathyroidism has a serum calcium level of 14 mg/dl (3.5 mmol/L), phosphorus of 1.7 mg/dl (0.55 mmol/L), serum creatinine of 2.2 mg/dl (194 mmol/L), and a high urine calcium. While the patient awaits surgery, the nurse should d. encourage the pt to drink 4000 ml of fluid daily.

d. encourage the pt to drink 4000 ml of fluid daily.

In a patient with central diabetes insipidus, administration of aqueous vasopressin during a water deprivation test will result in a d. increase in urine osmolality

d. increase in urine osmolality When vasopressin is administered, the symptoms are reversed, with water retention, decreased urinary output that increases urine osmolality, and an increase in blood pressure

A patient with hypoparathyroidism receives instructions from the nurse regarding symptoms of hypocalcemia and hypercalcemia. The nurse teaches the patient that if mild symptoms of hypocalcemia occur, the patient should d. rebreathe with a paper bag and then seek medical assistance.

d. rebreathe with a paper bag and then seek medical assistance.

A pt who uses every-other-day prednisone therapy for rheumatoid arthritis complains of not feeling as well on the non-prednisone days and asks nurse about taking prednisone daily instead. The best response to the pt is that d. there is less effect on normal adrenal function when prednisone is taken every other day.

d. there is less effect on normal adrenal function when prednisone is taken every other day.

Nonproliferative diabetic retinopathy

damaged capillaries leak blood--->lipids, fluid seep into retina--->hemorrhage, macular edema treat: blood sugar control, macular laser

pancreas and aging

decreased hormone production altered hormone metabolism and activity decreased responsiveness of tissue **increased glucose intolerance with decreased sensitivity to insulin

onset of diabetes 1

develop when pancreas can no longer produce sufficient insulin onset rapid impending or actual ketoacidosis

intervention of constipation

diet high in fiver and fluids decrease sugar or milk monitor hypernatremia and hyperphosphatemia enemas

diabetic retinopathy

disease of the retina in diabetics characterized by capillary leakage, bleeding, and new vessel formation (neovascularization) leading to scarring and loss of vision

Islet cell autoantibody testing

distinguish between autoimmune type 1 diabetes and diabetes from other causes

angiopathy in diabetes

end-organ disease from damage of blood vessels #1 cause of diabetes related deaths

improper use of insulin

entry of glucose into cell is blocked=hyperglycemia

Ovary

estrogen progesterone

factors that lower blood sugar

excercising ( check before, during, and after ) aspirin, alcohol, sulfonamides, oral contraceptives, monoamine oxidase inhibitor

interventions of hypoglycemia

give 10- 15 g of a fast acting simple carbohydrate retest the blood glucose level in 15 minutes and repeat the treatments if symptoms do not resolve once symptoms resolve, give a snack containg protein and carb ( milk and crackers) unless client plans to eat within 60 minutes

HLA

human leukocyte antigen

Hypoglycemia symptoms

hunger, fatigue, weakness, sweating, headache, dizziness, low bp, cold or clammy skin, numbness(fingers,toes,mouth), tachycardia, tremors, unsteady gait, slurred speech, vision changes, coma, seizures

signs of diabetes

hyperglycemia, blurred vision, slow wound healing, vaginal infection, weakness, signs of inadequate circulation

diabetic ketoacidosis

hyperglycemia, ketosis, metabolic acidosis and dehydration most likely in type 1 may be seen in type 2 severe illness and stress

somogyi phenomenom

hypoglycemia occurs at 2 to 3 am by 7 am pt becomes hyperglycemic treated by decreasing the eveinging dose of intermediate acting insulin or giving bedside snack

DKA treatment

if pt has UTI or pneumonia needs to be hospitalized IV at high flow rate (200hr.) with insulin R (it doesnt matter what solution its in)

severe hypoglycemia

if unconscious administer glucagon subcut, or intramuscularly administer second dose in 10 min if clinet remain unconscious give small meal once pt awakes notify physician treat with 50% dextrose in water

prediabetes ruled out with

impaired glucose tolerance impaired fasting glucose

cause of constipation

in fluids, fiber, activity, ignoring the urge, drug (pain meds) slow motility, Neurological conditions -DM, Parkinson's, MS, depression, laxative dependence.

protein

individual goals high protein not recommended for weight loss

rule of 15

ingesting 15g of carbs 4-6oz of soda, fruit juice, 1tbsp of honey, 1tsp of jelly recheck q15 after 2 or 3 tries contact HCP

Pancrease

insulin and glucagon

type 1 diabetes mellitus

insulin dependent 5 to 10% of type of diabetes more common in youth onset symptoms

insulin resistance syndrome

insulin receptors locates in skeletal muscle, fat and liver cells receptors are unresponsive and/or insufficient

type 2 diabetes patho

insulin resistance decreased insulin production by the pancreas body doesnt produce enough insulin or doesnt use effectively of both

metabolic syndrome patho

insulin resistance related to excessive visceral fat

referring syndrome of ten

is a potentially life-threatening metabolic complication that can occur when nutrition is restarted for a patient who is in a starvation state. When a patient is starved for nutrition, the body breaks down fat and protein, rather than carbohydrates, for energy. Protein catabolism leads to muscle and cell loss, often in major organs such as the heart, liver, and lungs. The body's cells lose valuable electrolytes, including potassium and phosphate, into the plasma. Insulin secretion decreases in response to these changes. When refeeding begins, insulin production resumes; and the cells take up glucose and electrolytes from the bloodstream, thus depleting serum levels.

pancreatic islet cell transplation

islets are harvested from pancreas 2 or more pancreas and inserted into portal vein of liver via catheter experimental

assessment of hyperpituitarism of growth hormone of the anterior pituitary gland

large hands thickening and protrusion of the jaw arthritic changes diaphoresis hypertension deepening of voice Intervention: patient support, hypophysectomy

post op intervention of surgery for breast cancer

lay pt is semi fouwler with affected arm elevated above the level of the heart maintain suction of drain and record amt of drainage post sign for no iv, blood pressure, etc on effected arm

metabolic syndrome management

life style modifications reduce major risk factors

parathyroid

located on thyroid gland; controls the calcium levels in your body, and normals the bone growth. and phosphorus metabolism

Pancrease

located posteriorly to stomach influences carbohydrates metabolism, indirectly influences fat and protein metabolism produces insulin and glucagon

Symptoms of polyneuropathy

loss of sensation abnormal sensations pain

Hyperglycemia symptoms

lots of eating, peeing, drinking. blurred vision, fatigue, weight loss, abdominal cramps fruity breath, headache, nausea

hypoglycemia

low blood sugar <70

two catergories of angiopathy

macrovascular microvascular

Long acting insulin

maintain BG in between meals and overnight

Macrovascular complications

medium to large vessels Coronary heart disease Cerebrovascular disease Peripheral vascular disease

metabolic syndrome

metablic factors known to increase riks for developing type 2 DM and cardiovascular disease 1. abdominal obesity of 40 inches or more for men, and 35 inches for women 2. hyperglycemia: fasting blood glucose level of more than 100 or on drug treatment for elevated blood glucose 3. hypertension: systolic bp of 130mm or diastolic of 85 or drug for treatment for hypertension hyperlipidemia: triglycerides of greater of 150

intervention of diabetes insipedus

monitor vitals monitor electrolytes maintain adequate fluids monitor intake/output, weight, and specific gravity administer chlorpropamide vasopressin and desmopressin administer

symptoms of malnutrition

most obvious hair and nails, skin, mouth crusting and ulceration, atrophy of muscles, CNS irritability or confusion. Decreased wound healing and anemic due nutritional deficiencies (iron and folic acid- needed to construct RBCs)

Diabetic neuropathy

nerve damage from impoverished blood flow can lead to erectile dysfunction, incontinence, poor wound healing, and loss of sensation from area most common: sensory neuropathy

diabetes type 2

non-insulin dependent adult onset diabetes -accounts for 80 to 90% of diabetes

A client, age 42, visits the gynecologist. After examining her, the physician suspects cervical cancer. The nurse reviews the client's history for risk factors for this disease. Which history finding is a risk factor for cervical cancer? a. Onset of sporadic sexual activity at age 17 b. Spontaneous abortion at age 19 c. Pregnancy complicated with eclampsia at age 27 d. Human papillomavirus infection at age 32

nswer D. Like other viral and bacterial venereal infections, human papillomavirus is a risk factor for cervical cancer. Other risk factors for this disease include frequent sexual intercourse before age 16, multiple sex partners, and multiple pregnancies. A spontaneous abortion and pregnancy complicated by eclampsia aren't risk factors for cervical cancer.

metabolic syndrome is caused by

obesity hypertension abnormal lipid levels high blood glucose

risk factors for type 2 diabetes

obesity, rapid weight gain, family hx, sedentary lifestyle, increasing age, and other components of metabolic syndrome

symptom most prevelent in diabetes type 1

polyuria polydipsia polyphagia weight loss

Assessment of diabetes insipidus

polyuria of 4-24 l/day polydipsia dehydration inability to concentrate urine low urine specipfic gravity of 1.006 or lower tachycardia

s/s of diabetes type 1

polyuria: excessive voiding polydipsia: excessive thirst polyphagia: excessive hunger

labs that indicate malutrition

prealbumin half life 2 days. Albumin 22 days. But these can be low during inflammatory response . Measure the CRP to see if there is inflammation. Liver enzymes elevated, decreased vitamins. Measure height, weight and skin fold thicknes. Handgrip strength and timed gait and chair stands measure strength.

Loss of Protective Sensation (LOPS)

prevents pt from being aware of injury screening with microfilment

adipokines

protein hormones made and released by adipose tissue (fat) cells **cause chronic inflammation(insulin resistance) seen in type 2

interventions for diarrhea

provide pedialyte for mode, for severe make npo to give bowel at rest and provide electrolyte replacement through iv

Human Leukocyte Antigen (HLA)

related to diabetes type 1 when exposed to virus= b-cells(insulin producers) are destroyed

risks for hypoglycemia

repeated episodes of hypoglycemia age pt who use beta blockers

Microvascular complications

retinopathy(eye) nephropathy(kidneys) neuropathy(nerves)

metabolic syndrome

risks factors that increase chance of developing cardiovascular disease, stroke and diabetes mellitus

Treatment for hypoglycemia

rule of 15 avoid high fats(milk, ice cream, cookies) delay or slow absorption

insulin most likely to cause hypoglycemia?

short acting -longer duration of action

assessment of SIADH

signs of fluid overload changes in level of consciousness and mental status weight gain hypertension tachycardia hyponatremia anorexia, nausea, and vomitting

insulin dependent tissues

skeletal muscle and adipose tissue helps transport glucose to cells

What should you do for someone with HHNK?

#1=Fluids, #2= Insulin, and electrolytes administered.

diabetic neuropathy patho

accumulation of sorbtiol and fructose in nerves cause reduced nerve conduction and demyelinization ischemic damage

critically ill pts and diabetes

BG > 180=insulin drip pt should increase intake of noncaloric fluids(water , sugar free gelo, decaff beverage

sensory neuropathy

Loss of protective sensation in lower extremities Major risk for amputation (hands and feet) referred to sock and glove syndrome

How long do prefilled syringes of humalog last?

1 week

bmi

(BMI) for adults should be between 18.5 and 25 ; older adults should have a BMI between 23 and 27. A BMI of 27 to 30 indicates overweight, over 30 indicates obesity, and 40 and greater indicates morbid obesity.

DKA lab findings

***-Hyperglycemia > 250mg/dl -Bicarbonate low (<15 mEq/L) -Ph low (< 7.3) -Ketonemia -Ketonuria (moderate to large) -Na level low or normal related to intracellular/extracellular shifts -K level normal or elevated, actual level is depletion

HHS lab findings

***Glucose- >600 mg/dL Osmolarity- >320 mOsm/L pH- >7.4 HcO3- >20 mEq/L NA- normal or low BUN- elevated Creatinine- elevated

sensory neuropathy treatment

**Control of glucose is the only treatment, Drug therapy can help control symptoms; drugs can be used for pain: 1) capsaicin (topical), 2) tricyclic antidepressants,e.g, amitriptyline (Elavil) 3) SSRI, e.g., duloxetine (Cymbalta), 4) antiseizure (gabapentin)

onset of type 2 diabetes

- Gradual onset - Person may go many years with undetected hyperglycemia - Osmotic fluid/electrolyte loss from hyperglycemia may become severe (hyperosmolar coma)

The nurse manager of a medical-surgical unit is asked to determine if the unit should adopt a new care delivery system. Which behavior is an example of an autocratic style of leadership? 1. Call a meeting and educate the staff on the new delivery system being used.

. An autocratic style is one in which the person in charge makes the decision without consulting anyone else.

The nurse is performing discharge teaching for a client diagnosed with Cushing's disease. Which statement by the client demonstrates an understanding of the instructions? 1. "I will be sure to notify my health-care provider if I start to run a fever."

. Cushing's syndrome/disease predis- poses the client to develop infections as a result of the immunosuppressive nature of the disease.

B cells in pancreas

.produce insulin in response to raising glycemia

The client is diagnosed with hypothyroidism. Which signs/symptoms should the nurse expect the client to exhibit? 1. Complaints of extreme fatigue and hair loss.

1 . A decrease in thyroid hormone causes decreased metabolism, which leads to fatigue and hair loss.

Which laboratory data make the nurse suspect the client with primaryhyperparathyroidism is experiencing a complication? 1. A serum creatinine level of 2.8 mg/dL.

1 . A serum creatinine level of 2.8 mg/dL indicates the client is in renal failure, which is a complication of hyperparathy- roidism.

How to limit radiation exposure to others after radioactive iodine therapy

1) Use private toilet facilities if possible, flush 2-3 times after each use 2) Separate laundering towels, bed linens, and clothes daily at home 3) Not preparing food for others that requires prolonged handling with bare hands 4) Avoid being close to pregnant women or children for 7 days after therapy

post up surgery intervention for bariatric surgery

1. clear liquids once bowel sounds return and pt is passing gas 2. liquids and puree for 6 weeks

the three glucose related emergencies in patients with diabetes are

1. dka 2. hyperglycemic hyperosmolar state hypoglycemia

diagnosis thru impaired glucose tolerance(IGT)

2 hour oral glucose tolerance test 140-199

During the admission assessment of a client with advanced ovarian cancer, the nurse recognizes which symptom as typical of the disease? 1) Diarrhea. 2) Hypermenorrhea. 3) Abnormal bleeding. 4) Abdominal distention.

4) Abdominal distention.

What hemoglobin A1C percent is normal?

4-6%, 4%=65, 6%=135, 7%=glucose of 170

82. The nurse is providing an in-service on thyroid disorders. One of the attendees asks the nurse, "Why don't the people in the United States get goiters as often?" Which statement by the nurse is the best response? 1. "It is because of the screening techniques used in the United States." 2. "It is a genetic predisposition rare in North Americans." 3. "The medications available in the United States decrease goiters." 4. "Iodized salt helps prevent the development of goiters in the United States."

4. Almost all of the iodine entering the body is retained in the thyroid gland. A deficiency in iodine will cause the thyroid gland to work hard and enlarge, which is called a goiter. Goiters are commonly seen in geographical regions having an iodine deficiency. Most table salt in the United States has iodine added.

The client is three (3) days postoperative unilateral adrenalectomy. Which discharge instructions should the nurse teach? 4. Tell the client to notify the HCP if the incision is inflamed.

4. Any inflammation of the incision indicates an infection

The nurse is admitting a client to rule out aldosteronism. Which assessment data support the client's diagnosis? 4. Blood pressure.

4. Blood pressure is affected by aldosteronism, with hypertension being the most prominent and universal sign of aldosteronism.

The nurse is developing a plan of care for the client diagnosed with acquired immunodeficiency syndrome (AIDS) who has developed an infection in the adrenal gland. Which client problem is highest priority? 4. Fluid volume deficit.

4. Fluid volume deficit (dehydration) can lead to circulatory impairment and hyperkalemia.

average amount of insulin secreted daily

40 to 50U 0.6U/kg

normal glucose range

70-110 mg/dL

When assessing a patient with a traumatic brain injury, the nurse notes uncoordinated movement of the extremities. The nurse would document this as A) a. Ataxia. B) b. Apraxia. C) c. Anisocoria. D) d. Anosognosia.

A) a. Ataxia.

diagnositic tests for diabetes

A1C(7% or higher) fasting plasma glucose 8-hour no cal intake (>126) two hour plasma glucose(>200) random blood glucose(>200)

Which information will the nurse include in teaching a female patient who has peripheral arterial disease, type 2 diabetes, and sensory neuropathy of the feet and legs? a. Choose flat-soled leather shoes. b. Set heating pads on a low temperature. c. Use callus remover for corns or calluses. d. Soak feet in warm water for an hour each day.

ANS: A The patient is taught to avoid high heels and that leather shoes are preferred. The feet should be washed, but not soaked, in warm water daily. Heating pad use should be avoided. Commercial callus and corn removers should be avoided. The patient should see a specialist to treat these problems

When teaching a diabetic patient who has just been started on intensive insulin therapy about mealtime coverage, which type of insulin will the nurse need to discuss? a. glargine (Lantus) b. lispro (Humalog) c. detemir (Levemir) d. NPH (Humulin N)

ANS: B Rapid- or short-acting insulin is used for mealtime coverage for patients receiving intensive insulin therapy. NPH, glargine, or detemir will be used as the basal insulin.

A patient with type 1 diabetes who uses glargine (Lantus) and lispro (Humalog) insulin develops a sore throat, cough, and fever. When the patient calls the clinic to report the symptoms and a blood glucose level of 210 mg/dL, the nurse advises the patient to a. use only the lispro insulin until the symptoms of infection are resolved. b. monitor blood glucose every 4 hours and notify the clinic if it continues to rise. c. decrease intake of carbohydrates until glycosylated hemoglobin is less than 7%. d. limit intake of calorie-containing liquids until the glucose is less than 120 mg/dL.

ANS: B monitor blood glucose every 4 hours and notify the clinic if it continues to rise. Infection and other stressors increase blood glucose levels and the patient will need to test blood glucose frequently, treat elevations appropriately with lispro insulin, and call the health care provider if glucose levels continue to be elevated. Discontinuing the glargine will contribute to hyperglycemia and may lead to diabetic ketoacidosis (DKA). Decreasing carbohydrate or caloric intake is not appropriate because the patient will need more calories when ill. Glycosylated hemoglobins are not used to test for short-term alterations in blood glucose.

A 48-year-old male patient screened for diabetes at a clinic has a fasting plasma glucose level of 120 mg/dL (6.7 mmol/L). The nurse will plan to teach the patient about a. self-monitoring of blood glucose. b. using low doses of regular insulin. c. lifestyle changes to lower blood glucose. d. effects of oral hypoglycemic medications.

ANS: C The patient's impaired fasting glucose indicates prediabetes, and the patient should be counseled about lifestyle changes to prevent the development of type 2 diabetes. The patient with prediabetes does not require insulin or oral hypoglycemics for glucose control and does not need to self-monitor blood glucose

A hospitalized diabetic patient who received 34 U of NPH insulin at 7:00 AM is away from the nursing unit, awaiting diagnostic testing when lunch trays are distributed. To prevent hypoglycemia, the best action by the nurse is to a. save the lunch tray to be provided upon the patient's return to the unit. b. call the diagnostic testing area and ask that a 5% dextrose IV be started. c. ensure that the patient drinks a glass of milk or orange juice at noon in the diagnostic testing area. d. request that the patient be returned to the unit to eat lunch if testing will not be completed promptly.

ANS: D request that the patient be returned to the unit to eat lunch if testing will not be Consistency for mealtimes assists with regulation of blood glucose, so the best option is for the patient to have lunch at the usual time. Waiting to eat until after the procedure is likely to cause hypoglycemia. Administration of an IV solution is unnecessarily invasive for the patient. A glass of milk or juice will keep the patient from becoming hypoglycemic but will cause a rapid rise in blood glucose because of the rapid absorption of the simple carbohydrate in these items.

What should be taught to a person with Addison's disease? check all that apply

Adrenal insufficiency=cause (adrenalectomy or coming off steroids too fast), hirsutism (hair distribution changes, avoid stress (infection, rm changes, or arguing), STEROIDS rest of life in am, Diet: Salt and decreased K (no salt substitutes), Immunocompromised.

What is an iport?

An injection port for ppl who have to give daily insulin injections where needle remain above surface and med delivered through cannula into subQ tissue.

When should you give lantus and why?

At bedtime to prevent somogyi effect (rebound hyperglycemia).

Lipodystrophy

Atrophy or hypertrophy of subcutaneous fat wasting or thickening of skin 6 months without use of site skin returns to normal

"Polydipsia and polyuria related to diabetes mellitus are primarily due to: "a. The release of ketones from cells during fat metabolism b. Fluid shifts resulting from the osmotic effect of hyperglycemia c. Damage to the kidneys from exposure to high levels of glucose d. Changes in RBCs resulting from attachment of excessive glucose to hemoglobin"

B

The first nursing action indicated when a patient returns to the surgical nursing unit following a thyroidectomy is to a. check the dressing for bleeding. b. assess respiratory rate and effort. c. support the patient's head with pillows. d. take the blood pressure and pulse.

B R: Airway obstruction is a possible complication after thyroidectomy because of swelling or bleeding at the site or tetany, and priority nursing action is to assess airway. The other actions are also part of the standard nursing care post-thyroidectomy but are not as high in priority.

A client is receiving continuous bladder irrigation postoperatively after having a transurethral resection of the prostate cancer. The most common complication with this surgery is? A. Infection B. Hemorrhage C. Urinary Retention D. Pain

B. Hemorrhage

A client is admitted to the hospital with a very low neutrophil count 10 days after her last course of chemotherapy ended. Recognizing this as a common side effect of chemotherapy, the nurse knows to assess for sign of A, Bleeding B. Infection C. Fatigue D. Malnutrition

B. Infection

A client with pancreatic cancer is receiving morphine for complaints of severe pain. The nurse is aware that the antidote for morphine is A. Romazicon (flumazicon) B. Naloxone (Narcan) C. Acetylcysteine (mucomyst) D. Charcoal with sorbitol

B. Naloxone (Narcan)

Intervention of diverticulitis

Bed rest Npo No lifting coughing or bending Increase fluids by 2500-3000

What are the S&S of Addisons?

Bronze pigmentation of skin, changes in distribution of body hair (hirsutism), weakness, weight loss, hypoglycemia, hypotension.

A nursing assessment of a patient with Cushing syndrome reveals that the patient has truncal obesity and thin arms and legs. An additional manifestation of Cushing syndrome that the nurse would expect to find is a. chronically low blood pressure. b. decreased axillary and pubic hair. c. purplish red streaks on the abdomen. d. bronzed appearance of the skin.

C Rationale: Purplish-red striae on the abdomen are a common clinical manifestation of Cushing syndrome. Hypotension and bronzed-appearing skin are manifestations of Addison's disease. Decreased axillary and pubic hair occur with androgen deficiency.

Which of the following nursing interventions is most appropriate when communicating with a patient suffering from aphasia post epidural stroke? A) a. Present several thoughts at once so that the patient can connect the ideas. B) b. Ask open-ended questions to provide the patient the opportunity to speak. C) c. Use simple, short sentences accompanied by visual cues to enhance comprehension. D) d. Finish the patient's sentences so as to minimize frustration associated with slow speech.

C) c. Use simple, short sentences accompanied by visual cues to enhance comprehension.

Truncal obesity, purple abdominal striae, and thin extremities may indicate which disease and hormone dysfunction?

Cushing syndrome - Hypercortisolism

After being seen in the oncology clinic, a client is admitted to the hospital with severe bone marrow depression. The client's cancer therapy consisted of radiation and chemotherapy. Which nursing diagnosis takes priority when developing this client's plan of care A. Risk for injury B. Imbalanced nutrition: less than body requirements C. Risk for infection D. Anxiety

C. Risk for infection

The nurse is caring for a 37 year old admitted to the ICU. The nurse on the preceding shift reports that a neurological examination was preformed on the client and that she had abnormal Dolls eyes test. Which of the following descriptions best described a normal occulocephalic response to the dolls eyes test? A. Bright light in one eye causes constriction of the opposite eye B. The eyes follow the direction og a quick turn of the head C. The eyes move up in the opposite direction from which the head is being turned D. The eyes develop nystagmus following instillation of cold water into the ear

C. The eyes move up in the opposite direction from which the head is being turned

transmission of the epstein barr virus in infectious mononucleosis is thought to occur A. During intercourse B. Through Blood transfusion C. Through Saliva D. Through superficial tears

C. Through Saliva

What electrolyte levels should you evaluate?

Calcium levels. Parathyroids may have been damaged or accidentally removed. Low levels of Calcium could be an EMERGENCY. Have calcium gluconate available.

What is a person with hyperthyroidism (graves disease) at risk for?

Corneal abrasions from exopthalmia (may need artificial tears and cover with tape at night.

Hypothalamus hormone

Corticotropin-releasing hormone (CRH) Thyrotropin-releasing hormone (TRH) Gonadotropin-releasing hormone (GnRH) Growth hormone-releasing hormone (GHRH) Growth hormone-inhibiting hormone (somatostatin GHIH) Prolactin-inhibiting hormone (PIH) Melanocyte-inhibiting hormone (MIH)

Nursing considerations for hormone diagnostic tests - Cortisol blood studies - Aldosterone blood studies - Adrenocorticotropic hormone (ACTH) - ACTH stimulation with cosyntropin

Cortisol blood draw - Diurnal, levels are higher in AM than PM - Stress and high activity produce elevated results Aldosterone blood draw - Inform patient that required position, supine or sitting/standing must be maintained for 2 hour before specimen is drawn ACTH blood draw - NPO after midnight, blood draw 6-8am - Diurnal levels; same with cortisol: higher in AM ACTH stimulation with cosyntropin - Baseline cortisol level before inject cosyntropin

After receiving change-of-shift report about these pts, which patient should nurse assess first? a. A 22-year-old admitted with SIADH who has a serum sodium level of 130 mEq/L. b. A 31-year-old who has iatrogenic Cushing's syndrome with a capillary blood glucose level of 244 mg/dl. c. A 53-year-old who has Addison's disease and is due for a scheduled dose of hydrocortisone (Solu-Cortef). d. A 70-year-old who recently started levothyroxine (Synthroid) to treat hypothyroidism and has an irregular pulse of 134

D R: Initiation of thyroid replacement in older adults may cause angina and cardiac dysrhythmias. The pt's high pulse rate needs rapid investigation by the nurse to assess for and intervene with any cardiac problems. The other pts also require nursing assessment and/or actions but are not at risk for life-threatening complications. (Cognitive Level: Application Text Reference: p. 1306 Nursing Process: Planning NCLEX: Physiological Integrity)

A 35 years old client has been receiving chemotherapy to treat cancer. Which assessment finding suggests that the client has developed stomatitis (inflammation of the mouth)? A White, cottage cheese-like patches on the tongue B Yellow tooth discoloration C Rust-colored sputum D Red, open sores on the oral mucosa

D Red, open sores on the oral mucosa The tissue-destructive effects of cancer chemotherapy typically cause stomatitis, resulting in ulcers on the oral mucosa that appear as red, open sores. White, cottage cheese-like patches on the tongue suggest a candidal infection, another common adverse effect of chemotherapy. Yellow tooth discoloration may result from antibiotic therapy, not cancer chemotherapy. Rust-colored sputum suggests a respiratory disorder, such as pneumonia.

Which of the following best indicates your client understands the teaching done by the nurse about alopecia A. My hair will never come back B. Hair loss is not something I need to worry about while I am taking my chemotherapy because it happens so rarely C. Alopecia is only something women have to worry about D. If I lose my air, I can yse wraps, scarves or hates to improve the way I look until it grows back after my treatments are done

D. If I lose my air, I can yse wraps, scarves or hates to improve the way I look until it grows back after my treatments are done

A 25 year old is receiving chemotherapy. Lab results indicate a platelet count of 20,000/mm3. Based on this information, the nurse should include which measure in the plan of care? A. Provide a diet high in iron B. Use hypoallergenic soap when bathing the patient C. Change the patients position every 2 hours D. Inspect the patients skin for ecchymosis and petechiae

D. Inspect the patients skin for ecchymosis and petechiae

A nurse is interviewing a client about his past medical history. Which pre-existing condition may lead the nurse to suspect that a client has colorectal cancer? A. Duodenal ulcers B. Hemorrhoids C. Weight Gain D. Polyps

D. Polyps

What are the S&S of DIABETES INSIPIDUS?

DECREASED ADH (DRY AS PRUNE-NO REABSORPTION OF H20 FROM KIDNEYS), INCREASED OUTPUT= DECREASED SG<1.005= DECREASED URINE OSMOLARITY, DEHYDRATION=INCREASED SERUM Na=INCREASED SERUM OSMOLALITY>300 MOS/KG.

Collaborative Care of Diabetes

Glucose-lowering agents - Insulin - Oral agents - Non-insulin injectable agents Balance diet, activity, medications Client and family education Prevent complications

Multiple Endocrine neoplasia

Inherited condition characterized by hormone-secreting tumors Type 1 - Commonly show signs of hyperactive pituitary gland Type 2 - Commonly show signs of medullary thyroid carcinoma and pheocromocytoma (tumor of adrenal gland) Treatment - Medication to block effects of excess hormones - Surgical removal

Diverticulitis

Inflammation of herniation of intestinal mucosa

What is characteristic of type II diabetes?

Insufficient insulin production, adults after 40 mostly, was adult onset dm, familial but not dt obesity, may need insulin

Intraductal papilloma

Mass in duct that results in nipple discharge; mass is usually not palpable Women 40 to 55 years of age

proliferative diabetic retinopathy

Neovascularization is new vessel formation that looks like radiating spokes.

Neuropathy

Nerve damage - Most common is loss of protective sensations - Increase injury to lower extremities Sensory neuropathy - loss of sensation, pain, paresthesias Autonomic Neuropathy - hypoglycemic unawareness, bowel incontinence and diarrhea, and urinary retention - Gastroparesis (delayed gastric emptying) can produce anorexia, nausea, vomiting, GERD, fullness - Cardio: postural hypotension, resting tachycardia, painless myocardial infarction - First common symptom in men is ED

internal feeding

Place patients receiving tube feeding in a semi-Fowler's position at all times to prevent aspiration; check residual contents every 4 hours or as designated per facility policy.

Acromegaly Diagnostic Studies

Plasma insulin-like growth factor 1 (IGF-1) - as GH level rises, so do IGF-1 Oral glucose tolerance test (OGTT) - Normally, GH concentration falls during an OGTT, but in acromegaly, GH levels do not fall

What are the 3 Ps of DM?

Polyphagia, polydipsia, and polyuria (others= fatigue and increase UTIs)

On what shift do problems occur with short acting? Intermediate acting? Long acting?

Short acting=first shift, intermediate acting=second shift, and long acting=3rd shift.

priority after bariatric surgery

The priority for immediate care of postoperative bariatric surgery patients is airway management. Patients with short and thick necks often have compromised airways and need aggressive respiratory support—possibly mechanical ventilation in the critical care unit.

A child is admitted with thyrotoxic crisis. Which manifestations should a nurse expect to observe during assessment (Select All) a. Agitiation b. Hypothermia c. Bradycardia d. Nausea e. Vomiting

a. Agitiation d. Nausea e. Vomiting

When caring for a patient with primary hyperaldosteronism, the nurse would question a physician's order for the use of a. Lasix

a. Lasix

When caring for a client receiving external beam radiation for a glioblastoma, the nurse should be alert for which of the following S/E a. N/V Headache b. Diarrhea and stomatits c. Severe diarrhea and headache d. Cystitis and alopecia

a. N/V Headache

Patients who have been receiving corticosteroid therapy for a prolonged period and suddenly stop the drug will experience an adrenal crisis because their own adrenal glands will not be producing any adrenal hormones. Your assessment of a patient with adrenal crisis will include a. Physiological exhaustion, shock, fluid shifts b. Acne development and hypertension c. water retention and increased speed of wound healing d. Hyperglycemia and water retention

a. Physiological exhaustion, shock, fluid shifts

During care of a patient with syndrome of inappropriate ADH (SIADH), the nurse should a. monitor neurologic status Q2H or more often if needed

a. monitor neurologic status Q2H or more often if needed Rationale- the pt with SIADH has marked dilution hyponatremia and should be monitored for decreased neurologic function and convulsions every 2 hours.

Following a transsphenoidal resection of a pituitary tumor, an important N assessment is a. monitoring hourly urine output.

a. monitoring hourly urine output. Rationale- After pituitary surgery, the pt is at risk for diabetes insipidus

A patient is hospitalized with possible SIADH. The patient is confused and reports a headache, muscle cramps, and twitching. The nurse would expect the initial laboratory results to include a a. serum sodium of 125 mEq/L (125 mmol/L).

a. serum sodium of 125 mEq/L (125 mmol/L).

tests for diagnosis of diabetes

a1c of greater that 6.5% fasting plasma glucose: 2 separate tests of 126 or greater or random reading of 200 mg/dl

constipation assessment

abdominal pain and cramping palpable movable fecal massess normal or decreased bowel sounds nausea, vomiting, anorexia

What blood sugar levels are common with HHNK?

above 600

A pt is admitted to the hospital in addisonian crisis 1 month after a diagnosis of Addison's disease. The nurse identifies the nursing diagnosis of ineffective therapeutic regimen management related to lack of knowledge of management of condition when the patient says, b. "I had the stomach flu earlier this week and couldn't take the hydrocortisone."

b. "I had the stomach flu earlier this week and couldn't take the hydrocortisone."

A pt with suspected acromegaly is seen at the clinic. To assist in making the diagnosis, which question should the nurse ask? b. "Do you have to wear larger shoes now?"

b. "Do you have to wear larger shoes now?"

High serum sodium and glucose level, low potassium and calcium count, associated with which of the following disease processes a. Addisons Disease b. Cushings Syndrome c. Graves Disease d. Mycedema

b. Cushings Syndrome

While hospitalized for a fractured femur, a 68-year-old pt is diagnosed with hypothyroidism. Which of these medications on the original admission orders will the nurse need to consult with the HCP about before it is administered? b. Diazepam (Valium)

b. Diazepam (Valium) R: Worsening of mental status and myxedema coma can be precipitated by the use of sedatives, especially in older adults.

A pt is taking high doses of prednisone to control the symptoms of an acute exacerbation of systemic lupus erythematosus. When teaching the pt about use of prednisone, which information is most important for the nurse to include? b. Do not stop taking the prednisone suddenly; it should be decreased gradually.

b. Do not stop taking the prednisone suddenly; it should be decreased gradually. R: Acute adrenal insufficiency may occur if exogenous glucocorticoids are suddenly stopped.

A client diagnosed with hyperparathyroidism is prescribed the biphosphonate alendronate (Fosamax). Which discharge instruction should the nurse discuss with the client a. The medication must be taken with the breakfast meal only b. Remain upright for at least 30 minutes after taking the medication c. The tablet should be chewed thoroughly before swallowing d. Stress the importance of having monthly hormone levels

b. Remain upright for at least 30 minutes after taking the medication

When providing postoperative care for a patient who has had bilateral adrenalectomy, which assessment information obtained by the nurse is most important to communicate to HCP? b. The patient's blood pressure is 102/50.

b. The patient's blood pressure is 102/50.

A few hours after returning to the surgical nursing unit, a patient who has undergone a subtotal thyroidectomy develops laryngeal stridor and a cramp in the right hand. The nurse anticipates that intervention will include b. administration of IV calcium gluconate.

b. administration of IV calcium gluconate.

The first nursing action indicated when a patient returns to the surgical nursing unit following a thyroidectomy is to b. assess respiratory rate and effort.

b. assess respiratory rate and effort

During preoperative teaching for a patient scheduled for transsphenoidal hypophysectomy for treatment of a pituitary adenoma, the nurse instructs the patient about the need to b. avoid brushing the teeth for at least 10 days after the surgery.

b. avoid brushing the teeth for at least 10 days after the surgery.

A pt is hospitalized with acute adrenal insufficiency. The nurse determines that the pt is responding favorably to treatment upon finding b. decreasing serum potassium.

b. decreasing serum potassium. R: CMs of Addison's disease include hyperkalemia

The nurse identifies a nursing dx of risk for injury: corneal ulceration related to inability to close the eyelids secondary to exophthalmos for a patient with Graves' disease. An appropriate nursing intervention for this problem is to b. elevate the head of the patient's bed to reduce periorbital fluid.

b. elevate the head of the patient's bed to reduce periorbital fluid.

When the patient with parathyroid disease experiences symptoms of hypocalcemia, a measure that can be used to temporarily raise serum calcium levels is to b. have the patient rebreathe in a paper bag

b. have the patient rebreathe in a paper bag

An appropriate nursing intervention for the patient with hyperparathyroidism is to b. increase fluid intake to 3000 to 4000ml/day

b. increase fluid intake to 3000 to 4000ml/day Rationale-A high fluid intake is indicated in hyperparathyroidism to dilute hypercalcemia and flush the kidneys so that calcium stone formation is reduced

A patient with SIADH is treated with water restriction and administration of IV fluids. The nurses evaluates that treatment has been effective when the patient experiences b. increased urine output, increased serum sodium, and decreased urine specific gravity

b. increased urine output, increased serum sodium, and decreased urine specific gravity

The most important nursing intervention during the medical and surgical treatment of the patient with a pheochromocytoma is b. monitoring blood pressure

b. monitoring blood pressure38 rationale- a pheochromocytoma is a catecholamine-producing tumor of the adrenal medulla, which may cause severe, episodic HTN;

A patient with hypothyroidism is treated with Synthroid. When teaching the patient about the therapy, the nurse b. provides written instruction for all information related to the medication therapy

b. provides written instruction for all information related to the medication therapy rationale- because of the mental sluggishness, inattentiveness, and memory loss that occur with hypothyroidism, it is important to provide written instructions

A nurse is preparing to discharge a child with leukemia observes a family member who has a cold sharing a meal with the child. How should the nurse approach the situation? a. instruct the family members not to share food because it isn't healthful. b. offer a face mask to the person with the cold and use this as an opportunity for further teaching. c. tell the family members to be careful to avoid the child if they're sick d. post isolation signs on the child's door and carefully assess the health status of all visitors.

b. offer a face mask to the person with the cold and use this as an opportunity for further teaching.

When teaching a patient with newly diagnosed hypothyroidism about management of the condition, the nurse should b. provide written handouts of all instructions for continued reference as the patient improves.

b. provide written handouts of all instructions for continued reference as the patient improves. R: Written instructions will be helpful to the patient because initially the hypothyroid patient may be unable to remember to take medications and other aspects of self-care.

When caring for a patient with nephrogenic DI, the nurse would expect treatment to include b. thiazide diuretics

b. thiazide diuretics Rationale- in nephrogenic Di the kidney is unable to respond to ADH, so vasopressin or hormone analogs are not effective.

During the nursing assessment of a patient with Graves' disease, the nurse notes a bounding, rapid pulse and systolic hypertension. Based on these assessment data, which question is important for the nurse to ask the patient? c. "Do you ever have any chest pain?"

c. "Do you ever have any chest pain?"

A client is newly diagnosed with Graves disease. In managing this illness, a clinic nurse should instruct the client to do which of the following? a. Sleep with an extra blanket b. Increase fiber in the diet to prevent constipation c. Eat frequent, small meals to meet the body's energy requirements d. Exercise just prior to bedtime to promote a restful sleep

c. Eat frequent, small meals to meet the body's energy requirements

A patient with oat cell lung carcinoma presenting to the ED with altered level of consciousness, nausea and vomiting is diagnosed with SIADH. The nurse would.... a. increased serum osmolality b. Hyperuricemia c. Hyponatremia d. Elevated hematocrit

c. Hyponatremia

What should be the nurses priority when admitting a client suspected of Addisons disease? a. Preparing the client for a CT Scan b. Administer meds to treat N/V c. Obtain EEG to assess for cardiac dysrythmias d. admin pain meds for complaints of abdominal pain commonly noted with Addisons

c. Obtain EEG to assess for cardiac dysrythmias

Your client has been diagnosed with diabetes insipidus after cranial surgery to remove a brain tumor. In addition to accurate fluid management and assessment of daily weight, you anticipate the administration of: a. Glipizide 10mg. po daily b. Regular insulin IV on a scale titrated according to hourly glucose levels c. Vasopressin 10 units subcutaneously daily d. Lasix 10 mg daily

c. Vasopressin 10 units subcutaneously daily

As you prepare to administer your clients dosage of levothyroxine you notice the clients heart rate is 164 beats per minute. You will: a. Administer the medication as prescribed b. Anticipate a dose increase c. Withhold the dosage and notify the physician d. Preform cardioversion to sloe the heart rate

c. Withhold the dosage and notify the physician

A pt seen at clinic for an upper respiratory infection reports receiving subcutaneous somatotropin (Genotropin) when asked by the nurse about current medications. The nurse questions the pt further about a hx of c. a pituitary tumor.

c. a pituitary tumor.

A patient with DI is treated with nasal desmopression. The nurse recognize that the drug is not having an adequate therapeutic effect the the patient experiences c. a urine specific gravity of 1.002

c. a urine specific gravity of 1.002 rationale- normal urine specific gravity is 1.003 to 1.030, and urine with a specific gravity of 1.002 is very dilute

A patient is admitted to the hospital in thyrotoxic crisis. On physical assessment of the patient, the nurse would expect to find c. elevated temperature and signs of heart failure

c. elevated temperature and signs of heart failure rationale- a hyperthyroid crisis results in marked manifestations of hyperthyroidism, with fever tachycardia, heart failure, shock, hyperthermia, agitation, N/V/D, delirium, and coma.

A pt with a possible pheochromocytoma is admitted to the hospital for evaluation and diagnostic testing. During an attack, the nurse will monitor for hypertension and c. headache.

c. headache. R: The classic CMs of pheochromocytoma are hypertension, tachycardia, severe headache, diaphoresis, and abdominal or chest pain.

A nursing assessment of a patient with Cushing syndrome reveals that the patient has truncal obesity and thin arms and legs. An additional manifestation of Cushing syndrome that the nurse would expect to find is c. purplish red streaks on the abdomen.

c. purplish red streaks on the abdomen.

A patient has an adrenocortical adenoma causing hyperaldosteronism and is scheduled for laparoscopic surgery to remove the tumor. During care before surgery, the nurse should c. monitor the blood pressure every 4 hours.

c. monitor the blood pressure every 4 hours. R: HTN caused by Na retention is a common complication of hyperaldosteronism.

A patient with Graves' disease is prepared for surgery with drug therapy consisting of 4 weeks of propylthiouracil (PTU) and 10 days of iodine before surgery. When teaching the patient about the drugs, the nurse explains that the drugs are given preoperatively to c. normalize metabolism and decrease the size and vascularity of the gland.

c. normalize metabolism and decrease the size and vascularity of the gland.

After a patient with a pituitary adenoma has had a hypophysectomy, the nurse will plan to do discharge teaching about the need for c. oral corticosteroids to replace endogenous cortisol

c. oral corticosteroids to replace endogenous cortisol R: ADH, cortisol, and thyroid hormone replacement will be needed for life after hypophysectomy.

A patient with an antidiuretic hormone (ADH)-secreting small-cell cancer of the lung is treated with demeclocycline (Declomycin) to control the symptoms of syndrome of inappropriate secretion of antidiuretic hormone (SIADH). The nurse determines that the demeclocycline is effective upon finding that the c. patient's urinary output is increased.

c. patient's urinary output is increased.

After receiving change-of-shift report about these pts, which patient should nurse assess first? d. A 70-year-old who recently started levothyroxine (Synthroid) to treat hypothyroidism and has an irregular pulse of 134.

d. A 70-year-old who recently started levothyroxine (Synthroid) to treat hypothyroidism and has an irregular pulse of 134. R: Initiation of thyroid replacement in older adults may cause angina and cardiac dysrhythmias.

The client with hyperthyroidism is prescribed the thioamide propylthiouricil (PTU) which lab data should the nurse monitor? a. ABG's b. Serum potassium c. RBC's d. WBC's

d. WBC's

A patient suspected of having acromegaly has an elevated plasma growth hormone level. In acromegaly, the nurse would also expect the patient's diagnostic results to include d. a serum somatomedin C (Insulin-like-growth-factor) of more than 300

d. a serum somatomedin C (Insulin-like-growth-factor) of more than 300 rationale- a normal response to growth hormone secretion is stimulation of the liver to produce somatomedin C which stimulates growth of bones and soft tissue.

Hormones that oppose insulin

glucagon, cortisol, epinephrine, norepinephrine, growth hormone = called counterregulatory hormones due to their effects in the liver, glycogen degradation, and release of glucose into the blood

insulin facilitates

glucose transport into cells transports of amino acids across muscle membranes

Herbs that lower blood sugar:

• Cinnamon (mild) • Bitter melon (stronger depending on dose), aleo, chromium, garlic, ginseng

assessments after bariatric

• Measure and record abdominal girth daily, as requested. • In collaboration with the dietitian, provide six small feedings and plenty of fluids to prevent dehydration. • Observe for signs and symptoms of dumping syndrome (caused by food entering the small intestine instead of the stomach) after gastric bypass, such as tachycardia, nausea, diarrhea, and abdominal cramping.

lab to monitor while on renters feeding

• Monitor laboratory values, especially blood urea nitrogen (BUN), serum electrolytes, hematocrit, prealbumin, and glucose.

Acromegaly Clinical Manifestations

*Too much growth hormone* Etiology - benign pituitary tumor (adenoma) Clinical manifestations - Big hands/feet/joint pain = difficult movement - Big tongue = difficult speech - Big vocal cords (hypertrophy) = voice deepens - Increase airway soft tissue = sleep apnea - Big skin = thick, leathery, oily skin - Menstrual disturbances - Tumor pressure on optic nerves = visual changes - Tumor pressure in head = headache - GH antagonizes insulin = hyperglycemia - *Increase hat, gloves, shoe sizes*

Diagnostic Studies in adrenocortical insufficiency

- Depressed serum and urinary cortisol levels - Increased ACTH levels in Primary - Decreased ACTH levels in Secondary ACTH stimulation test - Primary adrenal insufficiency is confirmed when cortisol levels fail to rise over basal levels with an ACTH stimulation test - Positive response to ACTH stimulation test indicates functioning adrenal gland and could be related to pituitary gland

Water deprivation test

- Deprive water 8-12 hrs - Give demopressin acetate (DDAVP) - Patients with central DI exhibit dramatic increase in urine osmolality, from 100 to 600 mOsm/kg and significant decrease in urine volume. Urine will be concentrated. (Kidneys still works and responds to ADH) - Patient with neurogenic DI will not be able to increase osmolality to great than 300 mOsm/kg. Urine will not be concentrated. (Kidneys does not work and does not respond to ADH)

Storage of insulin

- Insulin vials and pens may be left at room temperature for up to 4 weeks - Prolonged exposure to direct sunlight should be avoided - Unopened insulin stored in refrigerator

Insulin pump therapy

- Programmed to deliver a continuous infusion of rapid-acting insulin 24 hours a day - At mealtime, user programs the pump to deliver a bolus of additional insulin - Insertion site should be changed every 2-3 days to avoid site infection and promote good insulin absorption - Insulin pump users must check their blood glucose level at least 4 times per day

Diagnostic studies of Diabetes mellitus

1) A1C of 6.5% or higher 2) Fasting glucose > 126 mg/dL 3) Two-hour plasma glucose level > 200 mg/dL during an Oral Glucose Tolerance Test (OGTT) using glucose load of 75g 4) Classic symptoms of hyperglycemia: (polyuria, polydipsia, polyphagia), unexplained weight loss or hyperglycemic crisis

Explain A1C

A1C measures amount of glycosylated hemoglobin as a percentage of total hemoglobin - When glucose level is elevated, the extra glucose attaches to RBCs for remainder of the RBC's life - A1C represent % of RBC that has glucose attached to it, which explain how glucose has been elevated Diseases affecting RBCs (e.g. iron deficiency anemia, sickle cell anemia) can influence A1C results Estimated average glucose (eAG) can be calculated eAG = 28.7 x A1C - 46.7 A1C goal for patients with diabetes is < 7%

The nurse has administered 4 oz of orange juice to an alert patient whose blood glucose was 62 mg/dL. Fifteen minutes later, the blood glucose is 67 mg/dL. Which action should the nurse take next? a. Give the patient 4 to 6 oz more orange juice. b. Administer the PRN glucagon (Glucagon) 1 mg IM. c. Have the patient eat some peanut butter with crackers. d. Notify the health care provider about the hypoglycemia.

ANS: A The "rule of 15" indicates that administration of quickly acting carbohydrates should be done 2 to 3 times for a conscious patient whose glucose remains less than 70 mg/dL before notifying the health care provider. More complex carbohydrates and fats may be used once the glucose has stabilized. Glucagon should be used if the patient's level of consciousness decreases so that oral carbohydrates can no longer be given

A 38-year-old patient who has type 1 diabetes plans to swim laps daily at 1:00 PM. The clinic nurse will plan to teach the patient to a. check glucose level before, during, and after swimming. b. delay eating the noon meal until after the swimming class. c. increase the morning dose of neutral protamine Hagedorn (NPH) insulin. d. time the morning insulin injection so that the peak occurs while swimming.

ANS: A The change in exercise will affect blood glucose, and the patient will need to monitor glucose carefully to determine the need for changes in diet and insulin administration. Because exercise tends to decrease blood glucose, patients are advised to eat before exercising. Increasing the morning NPH or timing the insulin to peak during exercise may lead to hypoglycemia, especially with the increased exercise

A 34-year-old has a new diagnosis of type 2 diabetes. The nurse will discuss the need to schedule a dilated eye exam a. every 2 years. b. as soon as possible. c. when the patient is 39 years old. d. within the first year after diagnosis.

ANS: B Because many patients have some diabetic retinopathy when they are first diagnosed with type 2 diabetes, a dilated eye exam is recommended at the time of diagnosis and annually thereafter. Patients with type 1 diabetes should have dilated eye exams starting 5 years after they are diagnosed and then annually.

Which action should the nurse take after a 36-year-old patient treated with intramuscular glucagon for hypoglycemia regains consciousness? a. Assess the patient for symptoms of hyperglycemia. b. Give the patient a snack of peanut butter and crackers. c. Have the patient drink a glass of orange juice or nonfat milk. d. Administer a continuous infusion of 5% dextrose for 24 hours.

ANS: B Rebound hypoglycemia can occur after glucagon administration, but having a meal containing complex carbohydrates plus protein and fat will help prevent hypoglycemia. Orange juice and nonfat milk will elevate blood glucose rapidly, but the cheese and crackers will stabilize blood glucose. Administration of IV glucose might be used in patients who were unable to take in nutrition orally. The patient should be assessed for symptoms of hypoglycemia after glucagon administration

Thyrotoxicosis (thyrotoxic crisis or thyroid storm)

Acute, severe, and rare condition that occurs when excessive amounts of thyroid hormones are released into the circulation. - Can be from stressors (infection, trauma, surgery) in a patient with preexisting hyperthyroidism - Prevalent in patients undergoing thyroidectomy S/S - Extreme version of s/s in hyperthyroidism - Severe tachycardia, heart failure, shock, hyperthermia (up to 105.3*F) - Restlessness, seizures, abdominal pain - Vomiting, diarrhea, delirium, coma Treatment: - Anti-thyroid drugs: PTU and Tapazole - Iodine - Monitor for cardiac dysrhythmia - Provide a calm, quiet, cooled room - artificial tears to moisten dry eyes

Disorders of adrenal medulla (Pheochromocytoma)

Caused by tumor in adrenal medulla affecting the chromaffin cells - Result in excess production of catecholamines Complications - Most dangerous is severe hypertension Clinical manifestations: Constant "fight-or-flight" - Episodic hypertension - Classic triad of pheochromocytoma (Severe pounding headache, tachycardia, profuse sweating) - Palpitations - Unexplained abdominal or chest pains Diagnostic Test - Best test is Vanillymandelic acid (VMA) - Measurement of urinary fractionated metanephrines (catecholamine metabolites) - *Avoid palpating abdomen* of patient with suspected pheochromocytoma, since it may cause sudden release of catecholamines and severe HTN

Types of diabetes insipidus

Central (neurogenic) DI - Result from interference with ADH synthesis, transport, or release - Caused by tumor, head injuries, brain surgery, CNS infection Nephrogenic DI - Result from inadequate renal response to ADH despite presence of adequate ADH - Caused by renal damage, renal disease or renal drug therapy (especially lithium) Primary DI - Excessive water intake

Hyperaldosteronism (Conn's syndrome)

Characterized by excessive aldosterone secretion Main effects of aldosterone - Sodium retention - Potassium and hydrogen ion excretion Excessive aldosterone = - Hypertension with hypokalemic alkalosis Etiology/pathophysiology - Primary is caused mostly by adrenal adenoma - Secondary is caused by renal artery stenosis, renin-secreting tumors, and chronic kidney disease Clinical manifestations - Sodium retention (hypernatremia: HTN, headache) - Edema does not usually occur because rate of sodium excretion increases which prevents more severe sodium retention - Potassium wasting (hypokalemia: generalized muscle weakness, fatigue, cardiac dysrhythmias, glucose intolerance, metabolic alkalosis)

Hyperparathyroidism Collaborative Care

Collaborative Care - Surgery is most effective: removal of parathyroid + Autotransplantation of normal PTH tissue + Calcium supplement for life if autotransplant fail - High fluid, moderate calcium intake - Continue ambulation, avoid immobility - Biphosphonates: inhibit osteoclast - Calcimimetic agents (e.g. cinacalcet [Semsipar]) After surgery, monitor for tetany (sudden decrease in calcium) (muscle spasms, laryngospasms) - IV calcium

Myxedema coma

Complication of hypothyroidism - Can be precipitated by infection, drugs, exposure to cold, or trauma S/S - Cold temperature - Hypotension - Hypoventilation - Hypoglycemia - Lactic acidosis Treatment - Vital functions supported (ABCs) - IV thyroid hormone replacement - All meds should be IV because of paralytic ileus - Prevent/treat hypothermia

Goiter

Enlarged thyroid gland - Can be either overactive or underactive thyroidism Etiology - Lack of iodine in diet (most common worldwide) - Over/under production of thyroid hormones - Nodules that develop in the gland itself - Goitrogens (PTU, methimazole, Large iodine dose) Toxic nodular goiters - hyperthyroidism (Commonly found in Graves' disease)

Hypothyroidism Etiology/pathophysiology Clinical manifestations Complications

Etiology/Pathophysiology - Iodine deficiency: Most common cause in world - Atrophy of thyroid gland: Common cause in U.S. - Amiodarone (contains Iodine) and lithium (blocks hormone production) can be the cause Clinical Manifestations - Systemic slowing of body processes - Fatigue, lethargic, slowed speech, somnolence - Cold intolerance, constipation, cold/dry skin - Depressed, weight gain, impaired memory - Decreased CO = low exercise tolerance, SOB - *Myxedema* - puffiness, facial and periorbital edema, and masklike affect Diagnostic Testings - TSH and free T4 levels - Presence of thyroid antibodies (e.g. TPO) Complications: - Myxedema coma

Cushing syndrome

Excess corticosteroids (e.g. prednisone) *Most common is excess meds* Clinical manifestations - Weight gain - Accumulation of adipose tissue in trunk, face, C-spine area (Moon face, Buffalo hump) - Purplish red striae on abdomen, breasts, buttocks - Muscle wasting in extremities (thin skinny arms) - Weaker and thinner skin (fragile, easily bruise) - Menstrual disorders and hirsutism in women - Slow healing wounds - Acne, thinning of hair - Hyperglycemia due to glucose intolerance and increase gluconeogenesis by liver - HTN - Unexplained hypokalemia

Collaborative Care in adrenocortical insufficiency

Hormone replacement therapy - Glucocorticoid (Hydrocortisone) 2/3 dose in morning, 1/3 at night - *Corticosteroids administer with food* - Mineralocorticoid (fludrocortisone) in morning - Timing reflects normal circadian rhythm - During stressful situations, glucocorticoid dosage is increased to prevent addisonian crisis - Salt additives for excess heat or humidity

Adrenal insufficiency (addisonian crisis)

Life threatening emergency Triggered by - Stress (infection, surgery, psychologic distress) - Sudden withdrawal of corticosteroid hormone - Adrenal surgery - Sudden pituitary gland destruction Clinical manifestations - Hypotension, tachycardia, dehydration - Hyponatremia, hyperkalemia, hypoglycemia - Fever, weakness, confusion - Circulatory collapse often unresponsive to usual treatment (vasopressors and fluid replacements) - Severe vomiting, diarrhea, pain in abdomen, legs Treatment - Directed toward shock management - High-dose hydrocortisone replacement - Large volumes of 0.9% saline 5% dextrose to reverse hypotension and electrolyte imbalance until BP returns to normal

Hyperthyroidism/Grave's Disease

Pathophysiology - Patient develops antibodies to TSH receptors - They stimulate thyroid gland to produce T3 / T4 - Characterized by remissions and exacerbations - Lead to death of thyroid tissue = hypothyroidism Clinical Manifestations - Related to hyper-metabolism and tissue sensitivity - *Exophthalmos* - Weight loss, increased nervousness, restlessness - HTN, bounding rapid pulse, increase respiration - Increase appetite, intolerance to heat

Acromegaly Nursing Care

Post-op transsphenoidal hypophysectomy - HOB 30 (decrease headache common post-op) - Oral care q4h to keep surgical site clean - Avoid tooth brushing for at least 10 days - Avoid vigorous cough, sneeze, straining to prevent any CSF leak. Notify and test any drainage - Observe for diabetes insipidus (gland damage) - Life long hormone therapy

Function of insulin

Promotes glucose transport from bloodstream across cell membrane to the cytoplasm of the cell *keys to the cell's lock that allow glucose to get in*

Clinical manifestations of Hyperaldosteronisms Diagnostic tests

Sodium retention - Hypernatremia: HTN, headache - Edema does not usually occur because rate of sodium excretion increases which prevents more severe sodium retention Potassium wasting - Hypokalemia: generalized muscle weakness, fatigue, cardiac dysrhythmias, glucose intolerance, metabolic alkalosis Hyperaldosteronism should be suspected in all HTN patients with hypokalemia who are not being treated with diuretics Diagnostic tests - Elevated aldosterone levels - Decreased serum potassium levels - Decreased plasma renin activity

Acromegaly Collaborative Care

Surgery: Hypophysectomy - Hormone therapy needed throughout life Radiation - Full effects may not be noted for months-years - May lead to hypopituitarism = Lifelong hormone Drug therapy - octreotide (Sandostatin) + Reduce GH levels to normal + GH level measures q 2 weeks, then q 6 month - Dopamine agonists (bromocriptine [Parlodel]) + Reduce secretion of GH from the tumor - GH antagonists (e.g. pegvisomant [Somavert]) + Reduce GH effect by blocking hepatic IGF-1

Collaborative Care of Cushing's Syndrome

Treatment - Surgical removal or tumor - Gradual discontinuance of corticosteroids or lower dosage - avoid adrenal insufficency Drug therapy: - ketoconazole (Nizoral) - aminoglutethimide (Cytadren) - mitotane (Lysodren) Nursing - High-protein diet - Manage hypokalemia: supplements or food - After surgery, stay on bedrest until BP stable - High doses of corticosteroids are administered IV during surgery and for several days afterward - 24-48 hour after surgery is critical for circulatory instability

A 27-year-old patient admitted with diabetic ketoacidosis (DKA) has a serum glucose level of 732 mg/dL and serum potassium level of 3.1 mEq/L. Which action prescribed by the health care provider should the nurse take first? a. Place the patient on a cardiac monitor. b. Administer IV potassium supplements. c. Obtain urine glucose and ketone levels. d. Start an insulin infusion at 0.1 units/kg/hr.

ANS: A Hypokalemia can lead to potentially fatal dysrhythmias such as ventricular tachycardia and ventricular fibrillation, which would be detected with electrocardiogram (ECG) monitoring. Because potassium must be infused over at least 1 hour, the nurse should initiate cardiac monitoring before infusion of potassium. Insulin should not be administered without cardiac monitoring because insulin infusion will further decrease potassium levels. Urine glucose and ketone levels are not urgently needed to manage the patient's care

Which patient action indicates a good understanding of the nurse's teaching about the use of an insulin pump? a. The patient programs the pump for an insulin bolus after eating. b. The patient changes the location of the insertion site every week. c. The patient takes the pump off at bedtime and starts it again each morning. d. The patient plans for a diet that is less flexible when using the insulin pump.

ANS: A In addition to the basal rate of insulin infusion, the patient will adjust the pump to administer a bolus after each meal, with the dosage depending on the oral intake. The insertion site should be changed every 2 or 3 days. There is more flexibility in diet and exercise when an insulin pump is used. The pump will deliver a basal insulin rate 24 hours a day.

A 54-year-old patient is admitted with diabetic ketoacidosis. Which admission order should the nurse implement first? a. Infuse 1 liter of normal saline per hour. b. Give sodium bicarbonate 50 mEq IV push. c. Administer regular insulin 10 U by IV push. d. Start a regular insulin infusion at 0.1 units/kg/hr.

ANS: A The most urgent patient problem is the hypovolemia associated with diabetic ketoacidosis (DKA), and the priority is to infuse IV fluids. The other actions can be done after the infusion of normal saline is initiated

After change-of-shift report, which patient should the nurse assess first? a. 19-year-old with type 1 diabetes who has a hemoglobin A1C of 12% b. 23-year-old with type 1 diabetes who has a blood glucose of 40 mg/dL c. 40-year-old who is pregnant and whose oral glucose tolerance test is 202 mg/dL d. 50-year-old who uses exenatide (Byetta) and is complaining of acute abdominal pain

ANS: B Because the brain requires glucose to function, untreated hypoglycemia can cause unconsciousness, seizures, and death. The nurse will rapidly assess and treat the patient with low blood glucose. The other patients also have symptoms that require assessments and/or interventions, but they are not at immediate risk for life-threatening complications

The nurse is preparing to teach a 43-year-old man who is newly diagnosed with type 2 diabetes about home management of the disease. Which action should the nurse take first? a. Ask the patient's family to participate in the diabetes education program. b. Assess the patient's perception of what it means to have diabetes mellitus. c. Demonstrate how to check glucose using capillary blood glucose monitoring. d. Discuss the need for the patient to actively participate in diabetes management.

ANS: B Before planning teaching, the nurse should assess the patient's interest in and ability to self-manage the diabetes. After assessing the patient, the other nursing actions may be appropriate, but planning needs to be individualized to each patient.

The nurse is taking a health history from a 29-year-old pregnant patient at the first prenatal visit. The patient reports no personal history of diabetes but has a parent who is diabetic. Which action will the nurse plan to take first? a. Teach the patient about administering regular insulin. b. Schedule the patient for a fasting blood glucose level. c. Discuss an oral glucose tolerance test for the twenty-fourth week of pregnancy. d. Provide teaching about an increased risk for fetal problems with gestational diabetes.

ANS: B Patients at high risk for gestational diabetes should be screened for diabetes on the initial prenatal visit. An oral glucose tolerance test may also be used to check for diabetes, but it would be done before the twenty-fourth week. The other actions may also be needed (depending on whether the patient develops gestational diabetes), but they are not the first actions that the nurse should take

When a patient who takes metformin (Glucophage) to manage type 2 diabetes develops an allergic rash from an unknown cause, the health care provider prescribes prednisone (Deltasone). The nurse will anticipate that the patient may a. need a diet higher in calories while receiving prednisone. b. develop acute hypoglycemia while taking the prednisone. c. require administration of insulin while taking prednisone. d. have rashes caused by metformin-prednisone interactions.

ANS: C Glucose levels increase when patients are taking corticosteroids, and insulin may be required to control blood glucose. Hypoglycemia is not a side effect of prednisone. Rashes are not an adverse effect caused by taking metformin and prednisone simultaneously. The patient may have an increased appetite when taking prednisone, but will not need a diet that is higher in calories

The nurse determines a need for additional instruction when the patient with newly diagnosed type 1 diabetes says which of the following? a. "I can have an occasional alcoholic drink if I include it in my meal plan." b. "I will need a bedtime snack because I take an evening dose of NPH insulin." c. "I can choose any foods, as long as I use enough insulin to cover the calories." d. "I will eat something at meal times to prevent hypoglycemia, even if I am not hungry."

ANS: C Most patients with type 1 diabetes need to plan diet choices very carefully. Patients who are using intensified insulin therapy have considerable flexibility in diet choices but still should restrict dietary intake of items such as fat, protein, and alcohol. The other patient statements are correct and indicate good understanding of the diet instruction

The nurse is assessing a 22-year-old patient experiencing the onset of symptoms of type 1 diabetes. Which question is most appropriate for the nurse to ask? a. "Are you anorexic?" b. "Is your urine dark colored?" c. "Have you lost weight lately?" d. "Do you crave sugary drinks?"

ANS: C Weight loss occurs because the body is no longer able to absorb glucose and starts to break down protein and fat for energy. The patient is thirsty but does not necessarily crave sugar-containing fluids. Increased appetite is a classic symptom of type 1 diabetes. With the classic symptom of polyuria, urine will be very dilute

A hospitalized diabetic patient received 38 U of NPH insulin at 7:00 AM. At 1:00 PM, the patient has been away from the nursing unit for 2 hours, missing the lunch delivery while awaiting a chest x-ray. To prevent hypoglycemia, the best action by the nurse is to a. save the lunch tray for the patient's later return to the unit. b. ask that diagnostic testing area staff to start a 5% dextrose IV. c. send a glass of milk or orange juice to the patient in the diagnostic testing area. d. request that if testing is further delayed, the patient be returned to the unit to eat.

ANS: D Consistency for mealtimes assists with regulation of blood glucose, so the best option is for the patient to have lunch at the usual time. Waiting to eat until after the procedure is likely to cause hypoglycemia. Administration of an IV solution is unnecessarily invasive for the patient. A glass of milk or juice will keep the patient from becoming hypoglycemic but will cause a rapid rise in blood glucose because of the rapid absorption of the simple carbohydrate in these items

The nurse is interviewing a new patient with diabetes who receives rosiglitazone (Avandia) through a restricted access medication program. What is most important for the nurse to report immediately to the health care provider? a. The patient's blood pressure is 154/92. b. The patient has a history of emphysema. c. The patient's blood glucose is 86 mg/dL. d. The patient has chest pressure when walking.

ANS: D Rosiglitazone can cause myocardial ischemia. The nurse should immediately notify the health care provider and expect orders to discontinue the medication. There is no urgent need to discuss the other data with the health care provider


Related study sets

Anthropology Chapter One Test Quiz Questions

View Set

Chapter 23: The Agency Relationship- Creation, Duties, and Termination

View Set